2ちゃんねる ■掲示板に戻る■ 全部 1- 最新50    

■ このスレッドは過去ログ倉庫に格納されています

大学物理質問スレ part.1

1 :ご冗談でしょう?名無しさん:2020/01/10(金) 15:21:58 ID:a24XZavZ.net
まずは>>1をよく読みましょう

・質問する前に教科書や参考書をよく読みましょう。
・質問者は何が分からないのか、どこまで考えたのかを明記しましょう。
 問題の丸投げはダメです。丸投げに答えるのもダメ。ヒントを示す程度に留めましょう。
・質問者はあらゆる回答者に敬意を表しましょう。
 質問に対する返答には、何かしらの返答を。(荒らしはスルーでおながい)
・回答者がわかるように問題を書くようにしましょう。
 問題の写し間違いに気をつけましょう。
 問題の途中だけとか説明なく習慣的でない記号を使うとかはやめてね。

■書き方
・数式の例 (ちょっとした疑問や質問スレのテンプレも参考に)
 ベキ乗 x^2
 平方根 √(a+b)
 分数式 ((x+1)/(x+2))
 三角関数 sin(θ)
・図
 図が必要な場合、画像としてupするか、文字で書くことになります。
 文字で書く場合は、ずれに注意してください。
 MSPゴシックで表示できるエディタや2ch専用ブラウザを使いましょう。
 また、連続する半角空白は単一の空白として表示されるので注意。

2 :ご冗談でしょう?名無しさん:2020/01/10(金) 20:19:42.13 ID:???.net
ホール測定は磁場で電流が流れているだけなのに電場が生まれるのはなぜですか?

3 :ご冗談でしょう?名無しさん:2020/01/11(土) 00:59:15 ID:bk35M6Ft.net
>>2
ホール効果のことなら
ローレンツ力によって物質中の電子が偏ることによって電場が生まれるのでは?

4 :ご冗談でしょう?名無しさん:2020/01/11(土) 19:28:19.19 ID:???.net
それ本当?

5 :ご冗談でしょう?名無しさん:2020/01/11(土) 21:27:41.24 ID:4Rsz+WnR.net
ホール効果な。
p型でもn型でも移動しているのは電子である。
正孔の実態は電子の移動だからである。

電流の方向が同じなら
p型でもn型でも移動する電子の方向は同じであり、
ローレンツ力は電流に対して働くのだからこれもp型とn型で同じ方向のはずである。

しかし不思議なことに、そしてよく知られているように
電流の方向と磁場の方向が同じなのに
p型とn型では発生するホール電圧が逆極性になる!
移動しているのは実際にはどちらも電子なのにだ。

これは、p型半導体には正孔なんかではなく陽電子が存在し、
流入する普通の電子と電荷交換を行って電流を形成しているのであり、
このことを知っている人間ははほとんどいないのである。
負の質量とか笑わせんなよクソザルが。

くっくっく

6 :ご冗談でしょう?名無しさん:2020/01/11(土) 21:32:16.27 ID:???.net
>>5
effectiveな質量なだけね
本当に負であるわけではないんやよ

7 :ご冗談でしょう?名無しさん:2020/01/11(土) 22:11:53 ID:???.net
>>5
質問の答えは?

8 :ご冗談でしょう?名無しさん:2020/01/11(土) 23:42:12.91 ID:OJxBRyOe.net
>>7
電池と一緒だぞ。
ローレンツ力によって電子がその方向にドリフトして
片側の電子密度が濃くなり、片側が薄くなる。つまり、電界ができる。
この電界とローレンツ力が釣り合ったところでそのドリフトはなくなって、
本来の電流の方向へまっすぐ進むようになる。

電池と同じで内部のローレンツ力は外部には直接現れず、
現れるのは電子の濃淡による電位である。

この場合も当然ながら電位を測定する回路で∫E・ds>0。
内訳は、外部の電位測定回路にて部分積分∫E・ds>0であり、
ホール素子内では釣り合っているのでE=0で部分積分はゼロ。
2つ足して電位測定回路全体の周回積分は∫E・ds>0

アホザルには
これが意味不明なんだろうな。
実に哀れだわ。

くっくっく

9 :ご冗談でしょう?名無しさん:2020/01/11(土) 23:48:16.38 ID:OJxBRyOe.net
ああ、電位じゃなくて電位差な。

くっくっく

10 :ご冗談でしょう?名無しさん:2020/01/11(土) 23:49:24.09 ID:???.net
NGWord:くっくっく

11 :ご冗談でしょう?名無しさん:2020/01/11(土) 23:50:34.87 ID:???.net
俺も
NGWord:くっくっく

12 :ご冗談でしょう?名無しさん:2020/01/11(土) 23:51:21.75 ID:OJxBRyOe.net
電池では化学力だが
ホール素子ではローレンツ力になる。
両端が帯電するのは同じ。
だからホール素子は電池と一緒ってことだ。

くっくっく

13 :ご冗談でしょう?名無しさん:2020/01/11(土) 23:52:07.09 ID:???.net
磁場に垂直な面に抵抗を入れて片側の電荷をもう片側に移動させても電位差が生まれますか?

14 :ご冗談でしょう?名無しさん:2020/01/12(日) 12:55:08.42 ID:???.net
p型半導体に陽電子があるだって?

15 :ご冗談でしょう?名無しさん:2020/01/12(日) 14:48:40.64 ID:???.net
「ローレンツ力が同じ方向」から「電荷の移動方向は同じ」まではいいけど
電荷の正負で電場が逆になる事を気づかんとはね

16 :ご冗談でしょう?名無しさん:2020/01/12(日) 16:27:03.23 ID:???.net
>>14
相間くっくっくにとって電磁気力は遠隔作用だから、電場、磁場は仮想にすぎない
俺様説で通すには、なんちゃって陽電子をでっち上げて説明するしかない。

17 :ご冗談でしょう?名無しさん:2020/01/12(日) 18:44:35.69 ID:BdNZCl7+.net
>>5

爺さん、それ違うぞ、多分。
ホール電圧のことあんまり考えたことないんで
自信ないんだが。

先ず、確認したいのは、
電流、磁場が同じ時、電子が電流を担っている場合と、
プラスの電荷(仮想的)が電流を担っている場合を考えてみよう。
@動く方向は逆であるが、ローレンツ力は同じ方向である。
電荷が違うのでホール電圧は逆になる。
A半導体の電荷の実在はp型でもn型でも電子である。
だからホール電圧の向きは同じになるはずである。

実験と比べると@は正解Aは不正解。

すると、Aの考え方はどこか違っているのだろう。

俺の考えはn型では電子は伝導帯を自由電子っぽく動く
のに対してp型では電子の動きが違う。単純なモデルで
考えても、電子が動かなくてもホールは波のように動ける。
だから、プラスの電荷(仮想的)が動いているような
現象がおこるのであろう。

これからよく考えてみるが。

18 :ご冗談でしょう?名無しさん:2020/01/12(日) 19:09:32.82 ID:???.net
>>5
陽電子と電子が一緒の空間にいたら対消滅してγ線が出るので、p型半導体に電気流してγ線を観測した実験データをもってきてください

19 :ご冗談でしょう?名無しさん:2020/01/12(日) 19:47:40.59 ID:???.net
>>17
レス文からみると無理して妄想しなくてもよい
量子力学で数量的に説明済みだから、知りたければ勉強すればよい。
 古典力学的なアナロジーで運動エネルギーを持つ正電荷が移動するとみなしてもよい
ことが解るだろ。

20 :ご冗談でしょう?名無しさん:2020/01/12(日) 20:47:14.77 ID:BdNZCl7+.net
>>19

アホだね。

物理的表象がないんだね。

21 :ご冗談でしょう?名無しさん:2020/01/12(日) 21:08:40.92 ID:BdNZCl7+.net
物理学的描像がないと言うべきか。

22 :ご冗談でしょう?名無しさん:2020/01/12(日) 22:28:34.66 ID:BdNZCl7+.net
>>17

今のところであるが、

不純物半導体では電子と正孔の密度が違う(有効質量も違うが)ため
差し引いて正孔のみあるいは電子のみが移動していると考えて良い。

と言う辺りに絞って検証している。

23 :ご冗談でしょう?名無しさん:2020/01/12(日) 22:56:30.11 ID:???.net
>>19
>古典力学的なアナロジーで運動エネルギーを持つ正電荷が移動するとみなしてもよい

の物理的な意味は一般学生にもイメージし易い様に物質中のエネルギーバンドの絵を描いたり、
負電荷(電子)や正電荷(正孔)の古典粒子モデルの絵で電荷移動を説明しても間違いではない
と量子力学の物性論からお墨付きが与えられたという意味だ、有り難く思え。

24 :ご冗談でしょう?名無しさん:2020/01/13(月) 13:39:20.51 ID:E9jz7Zc+.net
>>22

今のところであるが

p型、3価の不純物が電子をつかまえて、ホールができても、つかまえた電子は
伝導帯にないから、ホールができるだけ。

n型、5価の不純物から伝導帯に上がっても、残った順位はホールとして機能しない。

それで、キャリア密度の差が生じる。

25 :ご冗談でしょう?名無しさん:2020/01/13(月) 14:11:29.42 ID:???.net
なぜ誘電率と屈折率の自乗は比例するのですか?
外場に対する分極の大きさと電磁波の速度がどう関係するのでしょうか。

26 :ご冗談でしょう?名無しさん:2020/01/13(月) 14:59:53.44 ID:???.net
>>25
マクスウェル方程式から電磁波の速度を導出してみれば分ると思う

27 :ご冗談でしょう?名無しさん:2020/01/13(月) 15:02:58.06 ID:???.net
数式ではなく物理的な描像が知りたいです。

28 :ご冗談でしょう?名無しさん:2020/01/13(月) 16:53:15.37 ID:E9jz7Zc+.net
>>24

これはキャリア発生の事について言及しただけで、p型における電子の移動が
自由電子の移動とどう違うかの説明にはなっていない。

n型において移動するのは自由電子、p型において移動するのは価電子帯の電子だ。

p型においては電流方向に移動しなくて、その場の3価電子のアクセプタ準位に
はまり込む(物が多い)と考えれば電場の変化はプラスに荷電した粒子の移動
のようになるのだろう。波のようだ。

29 :ご冗談でしょう?名無しさん:2020/01/13(月) 16:56:45.93 ID:E9jz7Zc+.net
このイメージで、半導体の物理学を考えてみることにする。

30 :ご冗談でしょう?名無しさん:2020/01/13(月) 23:33:17.60 ID:y/SraET7.net
解析力学でrとv(=dr/dt)は独立変数として扱いますけどそれは何故ですか?
v=dr/dtとの関係があるから従属変数のように思うのですが

31 :ご冗談でしょう?名無しさん:2020/01/14(火) 00:25:37.36 ID:???.net
導線に可視光周波数の交流を流せないのでしょうか。

32 :ご冗談でしょう?名無しさん:2020/01/14(火) 01:20:05.58 ID:???.net
>>30
ラグランジアンの中ではあるひとつのtの関数ではないんや
つまり各tでどんな値がはいっても良い
そして運動の決定には位置と速度が必要

33 :ご冗談でしょう?名無しさん:2020/01/14(火) 03:33:35.35 ID:???.net
音波に理論的に限界な周波数はありますか?

34 :ご冗談でしょう?名無しさん:2020/01/14(火) 11:39:59 ID:???.net
分子間距離から波長限界が決まり
分子運動速度から音速が決まる
波長と音速から周波数が決まる

35 :ご冗談でしょう?名無しさん:2020/01/14(火) 12:10:23 ID:???.net
何言ってんのこいつ

36 :ご冗談でしょう?名無しさん:2020/01/14(火) 15:44:47.36 ID:???.net
反強磁性は磁性を持たないから反磁性が表に出てくる?

37 :ご冗談でしょう?名無しさん:2020/01/14(火) 22:55:30.83 ID:???.net
何言ってんのこいつ

38 :ご冗談でしょう?名無しさん:2020/01/14(火) 22:56:26.34 ID:???.net
>>37
これで意味がわからないなら答えられないだろ

39 :ご冗談でしょう?名無しさん:2020/01/15(水) 00:34:25 ID:???.net
>>36
磁性をもつ とは?
表に出てくる とは?

40 :ご冗談でしょう?名無しさん:2020/01/15(水) 00:46:23 ID:???.net
>>39
答えれない奴の質問じゃん

41 :ご冗談でしょう?名無しさん:2020/01/15(水) 00:49:56 ID:???.net
それかアスペルガー症候群

42 :ご冗談でしょう?名無しさん:2020/01/15(水) 13:43:18.11 ID:???.net
反強磁性も磁性にきまっとんやん

43 :ご冗談でしょう?名無しさん:2020/01/15(水) 13:49:05.17 ID:???.net
アスペルガー症候群には質問の意図が読み取れない

44 :ご冗談でしょう?名無しさん:2020/01/15(水) 14:55:44.60 ID:???.net
フントの規則の3番目の全角運動量の最大とか最小というのはどういう意味なのでしょうか。
1番目と2番目でスピンと磁気量子数が確定するので全角運動量も確定して選択の余地はないような気がするのですが分かりません。

45 :ご冗談でしょう?名無しさん:2020/01/15(水) 15:01:16.82 ID:???.net
>>36
なんか出来の悪い学生みたいだな
ちゃんと言葉を選んで書けよ

46 :ご冗談でしょう?名無しさん:2020/01/15(水) 15:08:18.14 ID:???.net
>>36
古めの磁性の本には,反強磁性の帯磁率や磁化曲線の話は出てるだろ

47 :ご冗談でしょう?名無しさん:2020/01/15(水) 15:19:22.10 ID:???.net
よく読んだら反強磁性帯には常磁性は現れないって書いてました。ありがとうございました

48 :ご冗談でしょう?名無しさん:2020/01/15(水) 15:24:31.07 ID:???.net
上のレスに釣られて常磁性と書いてしまった

49 :ご冗談でしょう?名無しさん:2020/01/15(水) 15:42:24 ID:???.net
そうなのか
知らなかった

50 :ご冗談でしょう?名無しさん:2020/01/15(水) 16:26:59 ID:???.net
反磁性体にも常磁性程度の磁化率があって常磁性体に反磁性は埋もれるって書いてる

51 :ご冗談でしょう?名無しさん:2020/01/15(水) 22:03:11.96 ID:???.net
>>47
>>50

本が変だぞ

52 :ご冗談でしょう?名無しさん:2020/01/15(水) 22:05:58.96 ID:???.net
ありがと
適当書いて指摘くるか待ってた

53 :ご冗談でしょう?名無しさん:2020/01/16(木) 02:34:32.97 ID:???.net
>>44
1と2はそれぞれスピンと軌道の角運動量が決まるだけで
相互関係は何も言ってないぞ

54 :ご冗談でしょう?名無しさん:2020/01/16(木) 03:22:19.30 ID:???.net
>>53
電子を磁気量子数とスピンの表に埋めたら電子配置は確定する気がするのですが、まだ自由度が残っているのですか?

55 :ご冗談でしょう?名無しさん:2020/01/16(木) 08:10:45.72 ID:???.net
>>54
気がする,とか言ってないで
手を動かしてみろ

56 :ご冗談でしょう?名無しさん:2020/01/16(木) 09:55:41 ID:bG2t2PBY.net
>>33
>>34
分散関係によって上限が決められます

57 :ご冗談でしょう?名無しさん:2020/01/16(木) 10:48:36 ID:???.net
音波と音響モードは別物
波数0の音響モード振動が音波

58 :ご冗談でしょう?名無しさん:2020/01/16(木) 10:55:56 ID:nbtSLAYW.net
波の数が0の振動って並進運動と何が違うんですか?

59 :ご冗談でしょう?名無しさん:2020/01/16(木) 12:46:52.49 ID:7eOLE1VN.net
>>55
手を動かすとは?
私のやったものだと表を埋めてみても1と2で確定します。

60 :ご冗談でしょう?名無しさん:2020/01/16(木) 12:49:25.90 ID:???.net
>>58
>>57はすべての音波の波長は無限大だと主張するキチガイだよ

61 :ご冗談でしょう?名無しさん:2020/01/16(木) 13:00:35.11 ID:7eOLE1VN.net
例えばこうなります。
https://i.imgur.com/PcucSaJ.jpg

半数より少ないのでJの「最大(L+S)」を計算するとJ=L+S=2となりますが、そもそも最大とか最小というのが分かりません。

角運動量の合成とかいう話が関係するのかと思ったのですが、あれは合成する二つの運動量のそれぞれの値に関してすべてのパターンを取っているので結果が複数になるのは分かりますが、この問題だと既にmやsの具体的な値を足し合わせているので結果は1パターンしかない気がします。

正しい認識を教えてください。

62 :ご冗談でしょう?名無しさん:2020/01/16(木) 13:51:08 ID:???.net
>>61
角運動量の合成はそのままの足し算じゃなくない?

63 :ご冗談でしょう?名無しさん:2020/01/16(木) 14:09:52 ID:7eOLE1VN.net
確定した角運動量ベクトルを同士を合成するとその大きさでとりうる全ての角運動量ベクトルのどれになるかが分からなくなって、もとのベクトルのz成分は合成後のベクトルの大きさの算出に使われるだけで合成後のz成分はランダムということですか?

64 :ご冗談でしょう?名無しさん:2020/01/16(木) 14:20:34 ID:???.net
最大てのは絶対値が最大だぞ

65 :ご冗談でしょう?名無しさん:2020/01/16(木) 14:25:25 ID:7eOLE1VN.net
確定した運動量同士を合成しても合成後のベクトルは大きさも向きもバラバラの幾つもの状態の重ね合わせということですか?
電子配置が確定しても全角運動量は何通りもあるのですか?

66 :ご冗談でしょう?名無しさん:2020/01/16(木) 16:43:20.00 ID:???.net
>>65
確定した角運動量と磁気量子?は違うんちゃうん
角運動量演算子Lx,Ly,Lzの固有状態としてL^2の固有値l(l+1)(間違ってるかも)のlが最大のものってだけで
Lzの固有値mの最大じゃないんじゃないの?

67 :ご冗談でしょう?名無しさん:2020/01/16(木) 17:30:41.27 ID:WsMivrkr.net
d軌道はl=2なので大きさも確定していませんか?
大きさもz成分も確定してるベクトル同士を合成しても結果は確定しないということですか?
>>61のように電子を配置したときでも全角運動量の大きさとz成分が確定しないのですか?

68 :ご冗談でしょう?名無しさん:2020/01/16(木) 20:04:29.81 ID:???.net
>>57
音速340m/s
可聴周波数20Hz-20000Hz
波長にすると17m-1.7cm
固体原子の振幅はpmオーダー
原子の感覚からすれば無限大に近似

69 :ご冗談でしょう?名無しさん:2020/01/16(木) 20:42:43.85 ID:???.net
>>67
よくある誤解だな
フントルールの中には「z成分」とはどこにも書いてないよ

70 :ご冗談でしょう?名無しさん:2020/01/16(木) 20:44:53.36 ID:???.net
どうでもいいけど340m/sとか言うなら固体じゃなくて気体の平均自由行程持ってこいよ

71 :ご冗談でしょう?名無しさん:2020/01/16(木) 21:15:51 ID:7eOLE1VN.net
>>69
よくわからないです。

72 :ご冗談でしょう?名無しさん:2020/01/16(木) 21:20:56 ID:???.net
>>71
z成分がどうして確定するのですか
角運動量lに属す状態が多いものがエネルギー低いって書いてるんだから
もし角運動量lならその状態は2l+1こもつって習わなかったですか?

73 :ご冗談でしょう?名無しさん:2020/01/16(木) 21:31:01 ID:7eOLE1VN.net
属すのが多いとは?

74 :ご冗談でしょう?名無しさん:2020/01/16(木) 21:41:13 ID:???.net
>>68
>>70
物質名 縦波 [m/s] 横波 [m/s]
乾燥空気 331.45
水蒸気(100℃) 473
水 1500
海水 1513
氷 3230  1600
水素 1269.5
ヘリウム 970
窒素 337
酸素 317.2
塩素 205.3
アルゴン 319
水銀 1450
グリセリン 1986
ベンゼン 1295
エタノール 1207
四塩化炭素 930
二酸化炭素 258
ベリリウム 12890 8880
アルミニウム 6420 3040
鉄 5950 3240
金 3240 1220
鉛 1960 690
溶融水晶 5968 3764
ポリスチレン 2350 1120
軟質ポリエチレン 1950 540
天然ゴム 1500 120

75 :ご冗談でしょう?名無しさん:2020/01/16(木) 21:43:32 ID:???.net
>>73
Lx,Ly,Lzの交換関係はsu(2)代数ってやつです
ある角運動量がL^2とLzで同時固有状態をとるのは計算できます
そのとき状態はlとmでラベルできます
そのときl=l0(整数か半整数)なら
mは-l0からl0を1とびで値をもてます
つまりあるl0のときのそれの多重度は2l0+1です

それでフントの規則は
スピンに関しては多重度が最大が最低エネルギーでだからこそl0(スピンの場合は普通sを使う)が最大の場合ということですね
軌道角運動量は多重度はなんでも角運動量が最大が最低と言っているのでこれまたl0が最大のときですね

ここで別にどちらのmについても確定してませんよね?

76 :ご冗談でしょう?名無しさん:2020/01/16(木) 21:45:20 ID:7eOLE1VN.net
すみません、明日別の試験があるのでまた余裕があるときによく考えてみます。
ありがとうございます。

77 :ご冗談でしょう?名無しさん:2020/01/16(木) 21:45:53 ID:???.net
ほとんど「聞こえない」のに音響モード

78 :ご冗談でしょう?名無しさん:2020/01/16(木) 22:00:05 ID:???.net
>>72
>>状態が多いものがエネルギー低い
こういう説明は初めて聞いた
良くある説明なの?

79 :ご冗談でしょう?名無しさん:2020/01/16(木) 22:11:20 ID:???.net
>>78
フントの規則の1こめを多重度を状態の数っていっただけ

80 :ご冗談でしょう?名無しさん:2020/01/16(木) 22:34:16.95 ID:???.net
音は目に見える振動(ex.ギターの弦、ウーハー)
格子振動は見えない

81 :ご冗談でしょう?名無しさん:2020/01/16(木) 22:34:38.25 ID:???.net
>>77
だったら何なんだよ?

82 :ご冗談でしょう?名無しさん:2020/01/16(木) 22:41:51.95 ID:???.net
>>79
スピンの大きさが最大なのが重要で
多重度が大きいためではなかったような気がするんだが
違うかな

83 :ご冗談でしょう?名無しさん:2020/01/16(木) 22:44:40.37 ID:???.net
>>82
全スピンは最大多重度
全軌道角運動量はその最大量子数
であるときって書いとるで

84 :ご冗談でしょう?名無しさん:2020/01/16(木) 23:36:02 ID:???.net
エネルギーが下がる理由は
縮重度とは関係ないような気がするんだが

85 :ご冗談でしょう?名無しさん:2020/01/16(木) 23:41:39 ID:???.net
>>84
wikipedia参照しただけだから許して

86 :ご冗談でしょう?名無しさん:2020/01/17(金) 00:32:16.01 ID:???.net
同じ状態が多かったらクーロンポテンシャルエネルギーが大きいのさ

87 :ご冗談でしょう?名無しさん:2020/01/17(金) 08:56:29 ID:???.net
>>80
アホ

88 :ご冗談でしょう?名無しさん:2020/01/25(土) 04:34:48 ID:???.net
http://o.5ch.net/1lx5g.png

89 :ご冗談でしょう?名無しさん:2020/02/06(木) 03:09:37 ID:???.net
直線電流Iの生む磁場B(= μ0I/2πr)に対して、∇×Bはどんな値をとるのでしょうか。
マクスウェル方程式には電流密度iを用いて∇×B = μ0iの式がありますが、考えている直線電流は面積を持たない真の直線上を流れる電流ですから、電流密度をどのように与えればよいのか分かりません。

90 :ご冗談でしょう?名無しさん:2020/02/06(木) 05:49:27 ID:s0lLiIw1.net
微分形の解なら、デルタ関数で表すしかないのでは?

91 :ご冗談でしょう?名無しさん:2020/02/08(土) 03:00:09.24 ID:HnRgs7QX.net
>>89
現実を考えろよ。
面積ゼロの電流など存在せん。

ビオサバールの法則が元なんだから
面積ゼロなどありえんわ。

くっくっく

92 :ご冗談でしょう?名無しさん:2020/02/09(日) 00:45:19 ID:???.net
任意の分布による結果は1点に対する式の積分で得られる
どちらが基本かは言うまでもない

93 :ご冗談でしょう?名無しさん:2020/02/10(月) 01:40:44 ID:jiwCXM/o.net
>>89
あのさあ、その電流密度とやらの単位を
目の前の紙に書いてみてみてみ?

94 :ご冗談でしょう?名無しさん:2020/02/10(月) 01:46:25 ID:jiwCXM/o.net
>>92
この「格言」さん(たった今命名)、今日まで
ずっと九九九の分身かと疑ってたんだが、
どうやら別人のようね。

95 :ご冗談でしょう?名無しさん:2020/02/10(月) 01:52:51 ID:jiwCXM/o.net
>>91
いちへいべいあたりのでんりゅうが、
めんせきぢぁろぅのいってんにしうちう
してるとかんがえるの。このていどのびせき、
ちうがくくらいですませてくるやつおおいぞ。

96 :ご冗談でしょう?名無しさん:2020/02/10(月) 11:20:27.03 ID:???.net
>>89
今では分数計算ができない(忘れた)大学生が(文系?)もいるから問題外として
 電磁気理論の最初に点電荷同士のクーロン力を刷り込むのも問題だ、後で矛盾が判る。
点電荷の解釈は、有限の大きさの電荷の中心位置としたほうが電磁場での理解可能になる。

 電磁場の物理量は他の連続物理量(密度など)と同じく可測であり、簡単に言えば順番
に数えられ、総和、積分が確定する物理単位で扱う。
つまり、空間上の任意の点や線そのものは可測でない。

97 :ご冗談でしょう?名無しさん:2020/02/10(月) 12:30:39 ID:???.net
ぷっ

98 :ご冗談でしょう?名無しさん:2020/02/10(月) 13:31:05.13 ID:???.net
>>96
甘やかし過ぎじゃない?

99 :ご冗談でしょう?名無しさん:2020/02/10(月) 23:13:16.44 ID:???.net
>>96
可測って何?
可算の間違いにしては連続量とか言ってるし意味不明なんだけど

100 :ご冗談でしょう?名無しさん:2020/02/10(月) 23:29:07.83 ID:???.net
>>99
ある適当な測度空間において、測度が定義されない集合のことを可測でないといいます

101 :ご冗談でしょう?名無しさん:2020/02/10(月) 23:31:29.83 ID:???.net
ま、普通のユークリッド空間に通常の測度入れた場合は点も線も可測だと思いますけど

>>96の可測はそもそもそういう意味ではないでしょうけどね

102 :ご冗談でしょう?名無しさん:2020/02/10(月) 23:33:26.41 ID:???.net
線については病的な例があるかもだが、空間上の任意の点は可測だろ

103 :ご冗談でしょう?名無しさん:2020/02/11(火) 00:00:20.73 ID:???.net
まあ>>96は背伸びしたかったんだろ

104 :ご冗談でしょう?名無しさん:2020/02/11(火) 00:28:15.22 ID:???.net
ペアノ曲線は病的過ぎて可測だな

105 :ご冗談でしょう?名無しさん:2020/02/20(木) 16:01:42 ID:S2YlVA+R.net
ゲージ場、ゲージ対称性、ゲージ変換とか色々あるけど結局ゲージ理論ていうのは物理的に何が言いたいんですか?

106 :ご冗談でしょう?名無しさん:2020/02/20(木) 16:25:12 ID:???.net
粉末X線解析で面感覚を求める時は何故ブラッグの式のnを1と置くのですか?

107 :ご冗談でしょう?名無しさん:2020/02/20(木) 17:43:36.63 ID:???.net
>>105
名前そのものやん
ゲージ
例えばものさしの長さを局所的に変えても物理は変わらないことを要請している

108 :ご冗談でしょう?名無しさん:2020/02/20(木) 22:06:20 ID:???.net
>>105
積分定数やTorやExtに振り回されずBRSTコホモロジーを摂れって要請。

109 :ご冗談でしょう?名無しさん:2020/02/21(金) 12:47:15 ID:???.net
>>105
ゲージ対称性があればゲージ場がありゲージ粒子が存在して力を媒介する
というゲージ原理が現実だってことさ

110 :ご冗談でしょう?名無しさん:2020/02/25(火) 18:27:21 ID:???.net
重力波を観測する国内初の大型施設「かぐら」が2月25日、本格観測を開始
米国の観測施設と同時に外部天体の重力波を観測できれば重力波検証実験の信憑性が上がる
しかし、日本のような地殻変動が頻発する最悪環境で外乱・バックグラウンドノイズを
キャンセルできるなら凄い技術だ。

111 :ご冗談でしょう?名無しさん:2020/02/26(水) 14:21:25 ID:znCW5kEZ.net
微分幾何学入門
ttp://x0000.net/topic.aspx?id=3694-0

アルファ・ラボ|学術掲示板群
(理系 文系 工学 語学)
ttp://x0000.net/

112 :ご冗談でしょう?名無しさん:2020/03/01(日) 22:29:22 ID:???.net
この運動方程式ってどうなるんですか?
https://i.imgur.com/OQWGJzE.jpg

113 :ご冗談でしょう?名無しさん:2020/03/01(日) 23:16:30 ID:???.net
ゴースト場ってなんですか?

114 :ご冗談でしょう?名無しさん:2020/03/02(月) 13:26:03 ID:???.net
計算の都合で入れただけだから観測不可能ってことさ

115 :ご冗談でしょう?名無しさん:2020/03/04(水) 20:40:04.83 ID:???.net
これ教えて下さい。
一辺の長さがaの正方形と正三角形があり、
その周りに糸を下図のように密着させてFの力で引っ張る。
ただし、摩擦力はないものとする。
このとき、各図形にかかる力の分布(どの部分にどの大きさでかかることになるか)を答えよ。
http://o.5ch.net/1mkm4.png

116 :ご冗談でしょう?名無しさん:2020/03/04(水) 20:54:48.93 ID:???.net
あと、これもお願いします。
滑車の場合で、摩擦なしです。
半径をaとします。

よろしくお願いします。
http://o.5ch.net/1mkma.png

117 :ご冗談でしょう?名無しさん:2020/03/04(水) 20:58:55.58 ID:???.net
こんなの高校物理スレ行け

118 :ご冗談でしょう?名無しさん:2020/03/04(水) 20:59:46.41 ID:???.net
>>115
少なくとも理想化すれば角にしか力はかからない

119 :ご冗談でしょう?名無しさん:2020/03/04(水) 21:05:41.09 ID:???.net
>>118
できれば具体的に教えて下さい。
よろしくお願いします。

120 :ご冗談でしょう?名無しさん:2020/03/04(水) 22:17:17.73 ID:???.net
>>119
角さえあれば同じになるでしょ
ピンとはればその直線方向にしか力がはたらかない
もちろん曲がっているところは
釣り合いの式から力がはたらく

121 :ご冗談でしょう?名無しさん:2020/03/04(水) 22:43:39.95 ID:???.net
滑車に角?

122 :ご冗談でしょう?名無しさん:2020/03/05(木) 12:58:56 ID:???.net
滑車の場合は高校範囲じゃねーな

123 :ご冗談でしょう?名無しさん:2020/03/05(木) 13:02:02 ID:???.net
図形の点位置を示す記号さえ付けてないんじゃ
まともに聞いてるとは思えんわな

124 :ご冗談でしょう?名無しさん:2020/03/05(木) 14:12:32 ID:???.net
ランダウ力学§4の∂L/∂(v^2)*2vsが時間についての完全導関数になるような∂L/∂(v^2)は定数しかないってどうやって保証できるんですか?

125 :ご冗談でしょう?名無しさん:2020/03/05(木) 14:14:15 ID:OOzIGH9R.net
sじゃなくてεだ(どっちにしても定数だけど)

126 :ご冗談でしょう?名無しさん:2020/03/05(木) 16:08:45 ID:???.net
>>124
完全導関数作ってみたらええねん
でその差がでるけどそれを消すには定数しかないよって

127 :ご冗談でしょう?名無しさん:2020/03/05(木) 19:03:18.08 ID:g9rfwfO8.net
>>126
差って∂L/∂(v^2)*2vεを部分積分して積分の形にならない項の事ですよね?∂L/∂(v^2)が定数でなかったときその項が何かを積分した形にはならないことの根拠が分からないんですが

128 :ご冗談でしょう?名無しさん:2020/03/05(木) 19:57:04 ID:???.net
>>127
だからその項のせいで全微分にならないんでしょ?
でその項はその微分なんだからそれがゼロなら消えてくれるんでしょ?
だったら微分がぜろでそれは定数であってほしいんでしょ?

129 :ご冗談でしょう?名無しさん:2020/03/05(木) 21:07:51.36 ID:???.net
>>124
∂L/∂(v^2)*2vεが完全導関数になるなら、その定義から v を含まない q と t だけの関数 f(q,t) が有って
∂L/∂(v^2)*2vε= df(q,t)/dt = ∂f(q,t)/∂q * dq/dt + ∂f(q,t)/∂t が成り立つ。
dq/dt=v に注意して両辺を比較すると
 ∂L/∂(v^2)*2ε= ∂f(q,t)/∂q …@
 0 = ∂f(q,t)/∂t …A

@について見てみると、左辺はv^2以外の変数を含まず、右辺はvを含まない。
だから@の両辺は定数だと分かる。すなわち∂L/∂(v^2)も定数。

130 :ご冗談でしょう?名無しさん:2020/03/05(木) 21:18:10 ID:???.net
>>122
答えよろしく。
四角形、三角形、円の場合ね。

131 :ご冗談でしょう?名無しさん:2020/03/05(木) 23:08:55 ID:???.net
>>130
くっくっく以外答えられんだろ。

132 :ご冗談でしょう?名無しさん:2020/03/05(木) 23:14:21 ID:???.net
ではひとつ、くっくっく大明神の降臨を待ってみるか?

133 :ご冗談でしょう?名無しさん:2020/03/05(木) 23:26:16 ID:7M1R3Qbc.net
どうせ迷答にすぎない

134 :ご冗談でしょう?名無しさん:2020/03/05(木) 23:34:03 ID:???.net
誰か>>115の3つの場合について具体的に教えて下さい。
高校生2年の姪に聞かれて困っています(名門高校に通っています)。
よろしくお願いします。

135 :ご冗談でしょう?名無しさん:2020/03/05(木) 23:37:13 ID:???.net
ヤフー知恵袋で聞いたほうが早いでしょうか?
もう少し待ってみます。
どうかよろしくお願いします。

136 :ご冗談でしょう?名無しさん:2020/03/06(金) 00:23:51.24 ID:???.net
こんな答えをご希望か?
https://i.imgur.com/uTpogCg.png

137 :ご冗談でしょう?名無しさん:2020/03/06(金) 01:30:07 ID:0s+VqjmM.net
>>129
『∂L/∂(v^2)*2vε= df(q,t)/dt = ∂f(q,t)/∂q * dq/dt + ∂f(q,t)/∂t

       ↓

 ∂L/∂(v^2)*2ε= ∂f(q,t)/∂q …?
 0 = ∂f(q,t)/∂t …?       』

これって正しいんですかね?

138 :ご冗談でしょう?名無しさん:2020/03/06(金) 01:45:15 ID:0s+VqjmM.net
『g(v)・2v・ε=d/dt(f)   ( g(v)=∂L/∂(v^2)  )
   ↓
g(v)=const    』

は言えないんじゃないでしょうか?

f=g(v)・2q・ε-∫d/dt(g(v))・2q・εdt

がqとtのみの関数でなければ作用の変分を取るときに消えないのでg(v)=const

というのが正しいんじゃないでしょうか?

ランダウ先生に逆らって申し訳ないけど。

139 :ご冗談でしょう?名無しさん:2020/03/06(金) 01:59:11.15 ID:0s+VqjmM.net
よく見たら、p5で『座標と時間の任意関数の時間についての完全導関数』って書いてあった。
だけどp8の論理は間違ってる、あるいは言葉が不足しているので間違っている、といえるだろう。

140 :ご冗談でしょう?名無しさん:2020/03/06(金) 02:01:51.81 ID:0s+VqjmM.net
g(v)・2v・εが時間についての完全導関数、という要請だけでは

g(v)=const

はいえない。

時間についての完全導関数かつq,tだけの関数

という要請が必要である。

141 :ご冗談でしょう?名無しさん:2020/03/06(金) 02:10:26.18 ID:0s+VqjmM.net
>>137
>>129
∂L/∂(v^2)*2vεー ∂f(q,t)/∂q * v = ∂f(q,t)/∂t <>0

が成立するので、論理的に間違いであることが理解される。

142 :ご冗談でしょう?名無しさん:2020/03/06(金) 03:34:30.55 ID:60uoeioZ.net
>>136
ほう、正解だな。
黙って見ておったが
これほどの解答ができるヤツがおるとは思わなんだわ。
まあ、もうちょっと気を利かせて

・ひっぱられた糸はまっすぐになろうとする。
 それを阻害する箇所すなわち曲がったところに力が働き、それは張力である。
・合力の作用線は各図の真ん中を通る。
・正方形の上辺の平行な張力は打ち消しあうので、剛体としての合力はF+F=2Fが作用線を通る。
・1つめの正三角形の合力は計算すればちゃんと下向き2Fになって作用線を通る。
・2つめの正三角形の合力は2Fより小さい。それはFが斜め向きだから。

とこれぐらいのことは書いてやれ。
さて、滑車の場合だがヒントは2つめの正三角形にある。

円は無限の正多角形で近似できるので、ほぼ180度の角度を持つ二等辺三角形が
無限に連なったものとして考えればよい。つまり、力の線密度[N/m]だな。
これがどの方向に向いていて、大きさはいくらなのかを考えればよい。
意外と簡単だ。こういう問題を受験で出せ。
あとは考えてみろ。

くっくっく

143 :ご冗談でしょう?名無しさん:2020/03/06(金) 04:29:17.91 ID:???.net
>>137
うん。見直してみると単純に両辺を比較して導くのはすこし論理の飛躍があった。
だから以下のようにすればいい。
(正確な記述のため内積を『*』、普通の積を「・」で表す。)

1行目の式
∂L/∂(v^2)・2v*ε= df(q,t)/dt = ∂f(q,t)/∂q * dq/dt + ∂f(q,t)/∂t
より
∂L/∂(v^2)・2ε*v = ∂f(q,t)/∂q * v + ∂f(q,t)/∂t …B

この等式Bは任意の v に対して成立する等式なのだから、v=0 とおいて 0=∂f(q,t)/∂t …A を得る。

Aが成り立てば、等式Bの右辺第2項が落ちる。
∂L/∂(v^2)・2ε*v = ∂f(q,t)/∂q * v

この式に v=(v,0,0)、(0,v,0)、(0,0,v) を代入して両辺を v で割れば、それぞれ
∂L/∂(v^2)・2εx = ∂f(q,t)/∂x
∂L/∂(v^2)・2εy = ∂f(q,t)/∂y
∂L/∂(v^2)・2εz = ∂f(q,t)/∂z
を得る。但しε=(εx,εy,εz)、∂/∂q=(∂/∂x,∂/∂y,∂/∂z) とした。

この3式を成分とするベクトルの等式が
∂L/∂(v^2)・2ε= ∂f(q,t)/∂q …@

144 :ご冗談でしょう?名無しさん:2020/03/06(金) 05:13:59.06 ID:0s+VqjmM.net
>>143
>>∂L/∂(v^2)・2ε*v = ∂f(q,t)/∂q * v + ∂f(q,t)/∂t …B
>>
>>この等式Bは任意の v に対して成立する等式なのだから、
>>v=0 とおいて 0=∂f(q,t)/∂t …A を得る。

∂f(q,t)/∂t=0  (v=0)
∂f(q,t)/∂t=0とは限らない (v<>0)

v<>0で相対運動する系L’について考えてるんですけど・・・

145 :ご冗談でしょう?名無しさん:2020/03/06(金) 05:31:58.41 ID:???.net
>>144

Bはvに関する恒等式ではないとでも?

あと慣性系Kと慣性系K'の相対速度はεだろ。

146 :ご冗談でしょう?名無しさん:2020/03/06(金) 05:35:43.23 ID:0s+VqjmM.net
g(v)・2v・ε=d(f)/dt   ( g(v)=∂L/∂(v^2)  )

∫g(v)・2v・εdt=∫d(f)/dtdt=f

fが任意の関数だったらg(v)は何でも良さそうだよね。

fに、qとtだけの関数、と言う制限を付けて初めて

g(v)=const

が出てきそうだよね。

147 :ご冗談でしょう?名無しさん:2020/03/06(金) 05:41:20.71 ID:???.net
>fに、qとtだけの関数、と言う制限を付けて

この条件は最初から付いてる。外して良い条件じゃない。

148 :ご冗談でしょう?名無しさん:2020/03/06(金) 05:42:25.49 ID:0s+VqjmM.net
>>145

>>あと慣性系Kと慣性系K'の相対速度はεだろ。

間違えました。

149 :ご冗談でしょう?名無しさん:2020/03/06(金) 05:46:54.77 ID:0s+VqjmM.net
∂f(q,t)/∂t=0  (v=0)
∂f(q,t)/∂t=0とは限らない (v<>0)

× v<>0で相対運動する系L’について考えてるんですけど・・・
○ v=0、つまり静止している質量に対して成り立つにしても、v<>0
で運動している質量についてはどうなの?

150 :ご冗談でしょう?名無しさん:2020/03/06(金) 05:49:09.59 ID:0s+VqjmM.net
>>fに、qとtだけの関数、と言う制限を付けて

>この条件は最初から付いてる。外して良い条件じゃない。

最初から最後まで付けてないとg(v)=constは言えない。

151 :ご冗談でしょう?名無しさん:2020/03/06(金) 05:53:25.55 ID:???.net
>>149
f(q,t) が v を含まないから ∂f(q,t)/∂t も v を含まない。だから v に依存して∂f(q,t)/∂t の値が変わることはない。
v=0 のときに∂f(q,t)/∂t=0 となることが示されたなら、v がどんな値であっても∂f(q,t)/∂t=0

152 :ご冗談でしょう?名無しさん:2020/03/06(金) 05:55:06.60 ID:???.net
>>150
もちろん最初から最後まで f にはその条件が付く。

153 :ご冗談でしょう?名無しさん:2020/03/06(金) 05:59:51.05 ID:0s+VqjmM.net
g(v)・2v・ε=d(f)/dt   ( g(v)=∂L/∂(v^2)  )  @

例えばf=v*v
d(f)/dt=2a*v

g(v)=a/εが@を満たす。

154 :ご冗談でしょう?名無しさん:2020/03/06(金) 06:06:49.83 ID:???.net
>>153
>例えばf=v*v

その f は q と t だけの関数という条件を満たしていない。

155 :ご冗談でしょう?名無しさん:2020/03/06(金) 06:13:41.52 ID:0s+VqjmM.net
>>154

だから、fはqとtだけの関数なのだ。
最後まで。
この条件がなければダメであることを
判例をもって示した。

156 :ご冗談でしょう?名無しさん:2020/03/06(金) 06:25:54 ID:???.net
ランダウ(あるいはリフシッツ?)もP.8を書くにあたり、明記こそしてなくても、それを失念してはなかっただろうね。

157 :ご冗談でしょう?名無しさん:2020/03/06(金) 06:53:54.21 ID:0s+VqjmM.net
>>151
>>f(q,t) が v を含まないから ∂f(q,t)/∂t も v を含まない。
>>だから v に依存して∂f(q,t)/∂t の値が変わることはない。
>>v=0 のときに∂f(q,t)/∂t=0 となることが示されたなら、
>>v がどんな値であっても∂f(q,t)/∂t=0

@全体として、f(q,t)がqとtのみの関数であることを使用している。
>>外して良い条件じゃない。
とは矛盾している。

Ag(v)・2・dq/dt・e=h(q,t)・dq/dt+k(q,t)・dt/dt
更に
m(q,t,dq/dt)・dq/dt+k(q,t) =0
と書き直す。
dq/dt=0 ⇒ k(q,t) =0
は正しい。

dq/dt<>0 ⇒q,tの値を通してk(q,t)は変化してk(q,t) =0とは限らない。

k(q,t)にdq/dtが含まれないというのは、dq/dtがどんな値でもk(q,t)の値
は変わらない。q,tが同じならば・・・と言う意味だ。

158 :ご冗談でしょう?名無しさん:2020/03/06(金) 06:59:06 ID:0s+VqjmM.net
dq/dtの値を勝手に動かすというのはq,tは一定という条件で
考えなければならない。実現可能かどうかは別にして。
他変数関数を取り扱うときの注意点。

159 :ご冗談でしょう?名無しさん:2020/03/06(金) 07:24:11 ID:???.net
>>157
>?全体として、f(q,t)がqとtのみの関数であることを使用している。
>>>外して良い条件じゃない。
>とは矛盾している。

外してはいけない条件とは、f(q,t)がqとtのみの関数であることなので、矛盾していない。

>dq/dt=0 ⇒ k(q,t) =0
>は正しい。
>dq/dt<>0 ⇒q,tの値を通してk(q,t)は変化してk(q,t) =0とは限らない。

k(q,t) =0 とは、ある特定の q と t の時にだけ k(q,t) =0 だという意味でなく、すべての q と t に対して k(q,t) =0 という意味。
dq/dt=0 のとき、すべての q と t に対して k(q,t) =0 ならば、 q/dt≠0 のときにも、すべての q と t に対して k(q,t) =0

160 :ご冗談でしょう?名無しさん:2020/03/06(金) 07:45:15 ID:0s+VqjmM.net
>>dq/dt=0 のとき、すべての q と t に対して k(q,t) =0 ならば、
>>q/dt≠0 のときにも、すべての q と t に対して k(q,t) =0

dq/dt=0 のとき、すべての q と t に対して k(q,t) =0 であっても、
dq/dt≠0 になればqやtが変化するのでk(q,t) =0ではない。

∂L/∂(v^2)・2・dq/dt・ε= ∂f(q,t)/∂q・ dq/dt + ∂f(q,t)/∂t   ?
df(q,t)/dt = ∂f(q,t)/∂q ・dq/dt + ∂f(q,t)/∂t          ?

dq/dt=0 のとき、?から∂f(q,t)/∂t =0
dq/dt<>0 のとき
df(q,t)/dt - ∂f(q,t)/∂q ・dq/dt
=∂L/∂(v^2)・2・dq/dt・ε - ∂f(q,t)/∂q・ dq/dt
= ∂f(q,t)/∂t<>0

161 :ご冗談でしょう?名無しさん:2020/03/06(金) 08:03:01 ID:???.net
>>160
>dq/dt≠0 になればqやtが変化するのでk(q,t) =0ではない。

qやtの何がどう変化するというのか?
qやtは単なる変数だから、変化するとしてもqやtの取る値が変化するくらいしかない。
ならばそれが変化したところで k(q,t)=0 であることに変わりはない。

「A(x,y)・z + B(x,y)・z = C(x,y) がすべての x, y, z に対して成り立つなら
すべての x, y, z に対して C(x,y)=0 」と同じ。

162 :age:2020/03/06(金) 08:27:12 ID:???.net
>>dq/dt=0 のとき、すべての q と t に対して k(q,t) =0

dq/dt=0という拘束条件下で
命題A;∀q∀t:k(q,t) =0
が成り立つとする。

  ↓

dq/dt≠0 という拘束条件下でも、命題A;∀q∀t:k(q,t) =0 が成り立つ。』

これは誤りである。

k(q,t)の値はdq/dtに依らない、と言う考えがあるようだが、

dq/dt=0を満たす全ての(q,t)の組に対してk(q,t)=0が成立しても
dq/dt=0を満たさない(q,t)の組に対してk(q,t)=0は成立するとは言えない。

163 :ご冗談でしょう?名無しさん:2020/03/06(金) 08:36:47.93 ID:0s+VqjmM.net
dq/dt=0から∂f(q,t)/∂t =0 が導かれる。

この時既にq,tは任意でなくて、拘束条件"dq/dt=0"を満たす(q,t)の組に限られている。

164 :ご冗談でしょう?名無しさん:2020/03/06(金) 11:41:43.08 ID:???.net
>>162

点 (q,t) が dq/dt=0 を満たす(または満たさない)とは、どのような意味か?

仮に q=q(t) と書ける運動方程式の解曲線の傾きを考えているのだとしたら、
ラグランジアン自体が運動方程式ではないので、そのような曲線は決まらない。

単に点 (q,t) を通る曲線の傾きというだけなら、任意の (q,t) に対して任意の dq/dt が許される。
このとき t, q, dq/dt の間には、微分操作による結びつき以外の関係は何も定まらず、
形式的には独立変数と変わらない。
解析力学とは、このような発想をするものではなかったのか?

dq/dt がゼロであるか否かによって、それを満たす (q,t) の範囲は変わるものではない。

165 :ご冗談でしょう?名無しさん:2020/03/06(金) 12:51:35 ID:???.net
本持ってないとサッパリ分からんな

166 :ご冗談でしょう?名無しさん:2020/03/06(金) 15:49:36 ID:hfmkvqtH.net
>>134
なんだ、お兄さんじゃありませんか!?

167 :ご冗談でしょう?名無しさん:2020/03/06(金) 19:22:31.56 ID:???.net
スキャンしてアップすれば
くっくっくなら瞬殺しそうな内容で基地連投。ここは日記帳じゃねえぞコラ

168 :ご冗談でしょう?名無しさん:2020/03/06(金) 19:24:37.25 ID:???.net
>>136
おや、こんな芸当ができる人物がくっくっく以外にもいたんだw

169 :ご冗談でしょう?名無しさん:2020/03/06(金) 19:25:58.40 ID:???.net
ランダウ持ってない奴なんて…

170 :ご冗談でしょう?名無しさん:2020/03/06(金) 19:49:50 ID:???.net
>>142
滑車の場合分かったよ!dθで作図すれば簡単だった。
力の線密度はF/r[N/m]でどこも一定だよね?、滑車の中心へ向かう方向で。
それに半円の長さをかけるとπFで2Fより大きい圧縮力が滑車にかかるけど、
F/rの鉛直成分を半円で積分すれば2Fとなって、本当の合力は加えている力と一致するよね。

なるほど、受験問題では見たことないけど高校レベルだよね。
いつも勉強になるなあ。

171 :ご冗談でしょう?名無しさん:2020/03/06(金) 20:09:39.89 ID:???.net
微小円弧を図のように近似するとFdθ/2×2÷rdθ=F/r[N/m] (dθ→0でFsindθ/2=Fdθ/2)
張力はどこも同じなので、この図もどこも同じ。
鉛直成分の合力はF/rの鉛直成分を半円上で線積分すれば2Fとなって、張力による合力と一致。

172 :ご冗談でしょう?名無しさん:2020/03/06(金) 20:15:07 ID:???.net
円弧を近似した図。
sssp://o.5ch.net/1mlvl.png

173 :ご冗談でしょう?名無しさん:2020/03/06(金) 20:21:05 ID:???.net
>>122
高校レベルじゃねーかよハゲ出てこい

174 :ご冗談でしょう?名無しさん:2020/03/06(金) 20:24:21 ID:???.net
くっくっく一族ってマジで超神軍団だよな。
確かに斬新な問題で勉強になってワロタwwwwwwwwwwww

175 :ご冗談でしょう?名無しさん:2020/03/06(金) 20:29:35 ID:???.net
摩擦がある(滑車に質量があって慣性モーメントがある)場合には
左張力をF+dF、右張力をFとしてdFが滑車(慣性モーメント)を加速する力になるとして
慣性モーメントの式を考えれば[張力分布]と[滑車に働く力の分布]の2つも求まりそうだね。
これはたぶん大学レベルかな。高校生には酷だ。

くっくっく氏に任せることにしよう。

176 :ご冗談でしょう?名無しさん:2020/03/06(金) 20:34:46.40 ID:???.net
>>171
こういうのが本当の物理学と数学だよな。
しょーもないラグランジアンとか連投してる基地がやってるのは物理知らずの数学ごっこw

177 :ご冗談でしょう?名無しさん:2020/03/06(金) 20:36:03.39 ID:???.net
>>168
くっくっくじゃねーの?

178 :ご冗談でしょう?名無しさん:2020/03/06(金) 20:41:55 ID:???.net
>>177
違うでしょ。別人だと思う。

>>136>>172を合わせて、糸の張力や滑車はこういうのを教科書に載せるべき。
張力はどうなっているのか、剛体や滑車にはどんな力がかかっているのか、
これを理解できていないと教育の意味がないよね。
問題出す側もおそらく理解できていないと思う。

くっくっく氏じゃないけど、物理学も数学も教育がダメすぎるよ

179 :ご冗談でしょう?名無しさん:2020/03/06(金) 20:43:00 ID:???.net
>>122
高校レベルじゃねーかよハゲ出てこい

180 :ご冗談でしょう?名無しさん:2020/03/06(金) 20:53:01 ID:???.net
>>179
たぶん泣いてるからそっとしといてやれよ。
前スレでくっくっくに圧倒されて物理質問スレ立ってないし・・・

181 :136:2020/03/06(金) 21:01:10 ID:???.net
俺はくっくっくでもその取り巻きでもない。
そして奴の仲間と見なされるのは不愉快だ。

182 :ご冗談でしょう?名無しさん:2020/03/06(金) 21:13:35 ID:???.net
質問スレ立てようか?
誰か次の番号を教えてくれ。

183 :ご冗談でしょう?名無しさん:2020/03/06(金) 22:06:26 ID:???.net
みなさんありがとうございました。
>>136のように考えればいいんですね。滑車もなんとなくですが分かりました。
いやあ、張力って難しいですね。
ありがとうございました。

184 :ご冗談でしょう?名無しさん:2020/03/06(金) 22:11:01 ID:???.net
また自演ヨイショしてる

185 :ご冗談でしょう?名無しさん:2020/03/06(金) 23:41:11 ID:???.net
>>182 
前スレはこれですので次は249ですね
お願いします

■ちょっとした物理の質問はここに書いてね248■
https://rio2016.5ch.net/test/read.cgi/sci/1581577548/



あと高校質問スレもお願いします
part39です

↓前スレ
高校物理質問スレpart38
https://rio2016.5ch.net/test/read.cgi/sci/1578728461/

186 :ご冗談でしょう?名無しさん:2020/03/07(土) 00:12:40 ID:???.net
>>185
片方は立てた。もう片方は連投規制に引っかかったので暫く待って欲しい。

■ちょっとした物理の質問はここに書いてね249■
http://rio2016.5ch.net/test/read.cgi/sci/1583506686/

187 :ご冗談でしょう?名無しさん:2020/03/07(土) 01:04:09.37 ID:???.net
「大学物理質問スレ」なのに高校基本レベルみたいな力の見つけ方でゴチャゴチャ言ってるやつなんなんだ?

188 :ご冗談でしょう?名無しさん:2020/03/07(土) 01:27:49.94 ID:???.net
>>186
ありがとうございます!

189 :ご冗談でしょう?名無しさん:2020/03/07(土) 03:17:41 ID:LwYaWu82.net
>>181
でも所詮は同じ穴の住人じゃーん!?

190 :ご冗談でしょう?名無しさん:2020/03/07(土) 03:22:58 ID:LwYaWu82.net
>>176
トンデモさんあるある:「俺に理解できないリクツは、それが間違っているからだ」
な強い錯覚の持ち主。

窪田登司とかコンノケンイチとか。

191 :ご冗談でしょう?名無しさん:2020/03/07(土) 06:54:41 ID:???.net
ボケ老人です

192 :ご冗談でしょう?名無しさん:2020/03/07(土) 07:31:25 ID:???.net
お、隔離スレたったか

193 :ご冗談でしょう?名無しさん:2020/03/07(土) 15:49:02 ID:Fx4acnK8.net
>>190
だが、例えば○っ○っ○の珍説、俺さっぱり全然理解できないんだけど、
それは○っ○っ○が間違っているからだよね、やっぱり?

194 :ご冗談でしょう?名無しさん:2020/03/07(土) 16:02:35 ID:PBK52VVe.net
>>192
隔離じゃない本スレって、一体どこにあるんだよ!

195 :ご冗談でしょう?名無しさん:2020/03/07(土) 21:22:38 ID:???.net
>>136
これいいな。高校教科書に載せるべきだな。
くっくっく氏の指導のもとで物理板のレベルもかなり上がってきたな。

196 :ご冗談でしょう?名無しさん:2020/03/07(土) 21:30:54 ID:???.net
現実でもほとんど勘違いしてるよ。
重い段ボールがあって>>136のようにロープを巻いて持ち上げようとすると
角に力がかかって段ボールが破れてしまうんだけど、ほとんどの人間は段ボールの荷重は
ロープ全体にかかってちゃんと持ち上げられるはずだと思い込んでいるからね。

糸の張力については、こういう実践的なことを教科書に書くべきなんだよね。
くっくっく氏の言う通り、現状出来損ないの教科書ばかりでしょ。

197 :ご冗談でしょう?名無しさん:2020/03/07(土) 21:38:20 ID:???.net
>>196
ファインマンやランダウを絶賛してるようなレベルの連中が書いてるし無理。

198 :ご冗談でしょう?名無しさん:2020/03/07(土) 21:46:23 ID:???.net
>>172
質量を無視した滑車にかかる荷重は、大きさは張力/半径[N/m]で方向は中心向きだってことを
どうして高校物理で教えないんだろうね、本当に不思議。
教科書のレベルの低さが謎すぎるよ。

199 :ご冗談でしょう?名無しさん:2020/03/07(土) 21:49:56 ID:???.net
>>197
くっくっくが言うようにほとんどサルしかいないからな。
自分で考えることをせず、他人の理論をそのまま鵜呑みにしてるサルしかいない。
だからしょうもない本しか出てこない。

200 :ご冗談でしょう?名無しさん:2020/03/07(土) 21:53:27 ID:???.net
>>195
俺らみたいに相対性理論と量子力学と素粒子論は出鱈目理論だと気付いた人間が多数派になってきたからな。
本当にくっくっくには感謝しかないよ。

201 :ご冗談でしょう?名無しさん:2020/03/07(土) 21:55:43 ID:???.net
>>195
乱数と確率は定義できないって今まで考えたことなかったし衝撃的だった。
よくよく考えてみれば確かにそうだと気づいてびっくりしたよ。

202 :ご冗談でしょう?名無しさん:2020/03/07(土) 22:07:09 ID:???.net
>>198
これ教えないから、荷重はほとんど滑車のてっぺんにかかるとなんとなく思っているしな。
実際には荷重を超える力が均一に中心方向にかかり滑車を圧縮しようとする。
しかし荷重方向の合力はちゃんと荷重に等しくなる。これ理解している人間もごく少数じゃないか?

203 :ご冗談でしょう?名無しさん:2020/03/07(土) 22:10:39 ID:???.net
>>201
乱数も確率も、自分で定義を書こうとしたら手が止まるよな。
あいまいな言葉、つまり定義できない言葉でしか表現できない。
人類の最大の妄想が乱数と確率だろう。

204 :ご冗談でしょう?名無しさん:2020/03/07(土) 23:02:52 ID:???.net
>>203
コルモゴロフチャイティン複雑性の観点から言えばバグがあるコードの出現確率こそがランダムネスの定式化そのものだよ。

205 :ご冗談でしょう?名無しさん:2020/03/07(土) 23:40:14 ID:???.net
理解できない物を否定してくれる999は落ちこぼれの救世主
でも繰り返すのは大して効いてないんだなー

206 :ご冗談でしょう?名無しさん:2020/03/08(日) 00:26:13 ID:???.net
なんで量間は誰も彼もファインマンやランダウしか挙げないんや?
そいつらしか知らんだけかね?

お前らからしたらハイゼンとかシュレとかディラックとかのが圧倒的に罪深くないんかね?

207 :ご冗談でしょう?名無しさん:2020/03/08(日) 00:34:13 ID:???.net
キモい略し方すんな

208 :ご冗談でしょう?名無しさん:2020/03/08(日) 00:46:09 ID:???.net
>>204
それが乱数であることの証明よろしく。

209 :ご冗談でしょう?名無しさん:2020/03/08(日) 00:52:08 ID:???.net
>>201
性質または特徴を持たないものが乱数と確率。
だから定義できないし、存在もしないし、実証もできない。

くっくっくって本当に天才だと思う。
これに気付いただけで丸儲けだわ。

210 :ご冗談でしょう?名無しさん:2020/03/08(日) 00:53:33 ID:???.net
>>204
どうやったらそれが乱数なのか実証できんの?

211 :ご冗談でしょう?名無しさん:2020/03/08(日) 01:20:36.52 ID:???.net
>>209
一言で言えば「予測できないもの」が乱数や確率だとほとんどすべての人間が勘違いしている。
予測できないのは観測精度や制御などの技術的な問題が原因なのであって、原理的には
因果律によりすべての事象には原因があるから結果が生じる。つまり、事象の連鎖は最初から
決まっているので乱数や確率は存在しない。しかも大事なことは、事象が予測不能であっても
その偏りの有無を有限の観測回数で決定することは決して出来ず、そもそも偏りの有無にかかわらず
乱数や確率と断言することができないところにある。結局、何を持って乱数や確率と言えるのか
まったく示すことができないのである。

212 :ご冗談でしょう?名無しさん:2020/03/08(日) 01:30:39 ID:???.net
[問題]
ある予測不能な事象があって、それがAになる割合が60%、Bになる割合が40%だったとする。
しかし、予測不能なので今後もこの割合が続くかどうか不明である。ずっとAが続くかもしれない。
なぜならば予測不能だからである。そしてその後ずっとAが続いて起きたとする。
果たして、この事象は予測不能だから乱数や確率であると言えるか?

普通の理系脳なら、これだけで十分であろう。

213 :ご冗談でしょう?名無しさん:2020/03/08(日) 01:35:52 ID:???.net
>>212
マジ十分すぎるわw
明日その辺の数学教授に質問してみるわwww
あっ、哲学教授のほうがいいかしらwwwwwwwwwwwwwwwwwwwwwwwwwwww

214 :ご冗談でしょう?名無しさん:2020/03/08(日) 01:40:12 ID:???.net
>>212
なるほど、どうあがいても乱数と確率は定義不能だね。
最初から論理破綻してる。

215 :ご冗談でしょう?名無しさん:2020/03/08(日) 01:46:03 ID:???.net
>>212
ブラウン運動なんてもろに分子レベルで1個1個が計算できないだけだからな。
ランダムでもなんでもない。ニュートン力学と熱力学ですべて未来が決定されていることだよな。

216 :ご冗談でしょう?名無しさん:2020/03/08(日) 01:47:15 ID:???.net
くっくっくってどこまで賢いの?
なんで出版しないの?
まじで凄いよ!
理論物理学全滅じゃんね?

217 :ご冗談でしょう?名無しさん:2020/03/08(日) 01:48:18 ID:???.net
>>212
くっくっくなんか?

218 :ご冗談でしょう?名無しさん:2020/03/08(日) 01:52:50 ID:???.net
>>212
万有引力も乱数や確率になってしまうな。
仕組みが分からないので今後の予測が不能だからな。
今までr^2だったけど明日からr^2.001になるかもしれないし。
ああ、そうするとあらゆるものが乱数や確率になってしまうわけかw

219 :ご冗談でしょう?名無しさん:2020/03/08(日) 02:12:05 ID:???.net
>>208
>>210
実用上おまえらみたいなITドカタが御自分でプログラミングという形式論理扱いできる定義の羅列を書く作業をしてて
それがバグを孕んでるかどうかがゲーデルの定理の言い換えとしてランダムネス持ってるんだよ。

220 :ご冗談でしょう?名無しさん:2020/03/08(日) 07:43:48.35 ID:???.net
またくっくっくか

221 :ご冗談でしょう?名無しさん:2020/03/08(日) 12:44:00.19 ID:???.net
落ちこぼれの救世主

222 :ご冗談でしょう?名無しさん:2020/03/08(日) 14:18:22 ID:???.net
>>212
>ある予測不能な事象があって、それがAになる割合が60%、Bになる割合が40%だったとする。

その前提条件が確率と同様といえる

>予測不能なので今後もこの割合が続くかどうか不明である。

始めの前提条件を後から否定してる、つまり前提を否定したのだから何でも有り話になる。
 クックっクのような相間がよくやる手口だが、詐欺師はインチキだと思っていない。

マトモなロジック推論では前提条件(基本仮定である確率、光速不変など)
を否定することはできない。
確率論の推論に矛盾があるか無いか または 確率論がその(物理)現象に適用できるかできないか。
を論理的に議論することができる。

223 :ご冗談でしょう?名無しさん:2020/03/08(日) 14:35:15 ID:???.net
>>222
”予測不能”がくっくっくの類の何でもあり、起こりうる意味なら
>Aになる割合が60%、Bになる割合が40%だったとする。
などハナから騙すための見せかけ文にすぎない。

確率論では”Aになる割合が60%、Bになる割合が40%”は大数の法則が成り立っている
という前提条件だから、マトモな学生は騙されないようにしよう。

224 :ご冗談でしょう?名無しさん:2020/03/08(日) 15:13:59 ID:???.net
>215
>ニュートン力学と熱力学ですべて未来が決定されている
現代物理によれば大間違いだが
19世紀以前の科学ならそう自己満足でき、量子現象もウイルスなども未発見だった。

サイバネティクスによればニュートン力学と統計力学は、非統計の理論と
統計理論という分類になり
情報・通信から生物活動の科学は、時間発展の統計確率理論により解析可能になる。

225 :ご冗談でしょう?名無しさん:2020/03/08(日) 15:24:21 ID:???.net
背理法ね

226 :ご冗談でしょう?名無しさん:2020/03/08(日) 18:42:40 ID:1bF9tv+r.net
ま〜た類友の888まで涌いてくさるw

227 :ご冗談でしょう?名無しさん:2020/03/08(日) 18:43:44 ID:1bF9tv+r.net
>>225
はいりはいりふれはいりほ〜!

っての禁止?

228 :ご冗談でしょう?名無しさん:2020/03/08(日) 23:05:50 ID:???.net
なんとか立てた。

高校物理質問スレpart39
http://rio2016.5ch.net/test/read.cgi/sci/1583675821/

229 :ご冗談でしょう?名無しさん:2020/06/08(月) 08:09:44.96 ID:???.net
esu単位系の本にローレンツ力がf=q/c v×Hと書いてありました。
しかし、esu単位系ではμ0=1/c^2なので、f=q/c^2 v×Hになると思うのですがなぜq/cなんですか?

230 :ご冗談でしょう?名無しさん:2020/06/17(水) 01:08:21 ID:???.net
定義だろ

231 :ご冗談でしょう?名無しさん:2020/06/17(水) 19:52:27.46 ID:6jd+HjkU.net
N個のおもりからなる連成振子系:
天井から等間隔(aとする)に長さlの糸で質量mの質点を吊り下げ、隣接する粒子間をばね定数Kの軽いばねで連結した系で微小振動させたときについて、
各振動子の振れ角θiの運動方程式を与えるラグランジアンは

L=Σ_i (1/2)ml^2 (dθi/dt)^2 - (1/2)mgl(θ_i)^2 -(1/2)Kl^2(θ_i - θ_{i-1})^2 + (1/2)Kl^2(θ_{i+1} - θ_i)^2

になることはわかる。このとき、このラグランジアンに対して
全質量M = Nm, 全長L=(N-1)a, ばねの力T=Kaを有限に保って
粒子間隔をa → dxとする極限をとって連続系に移行したとき、正しいラグランジアン密度にならないのはなぜなんだぜ?

232 :ご冗談でしょう?名無しさん:2020/06/17(水) 21:16:25.73 ID:???.net
難しい問題には回答がつくのが遅いですね

233 :ご冗談でしょう?名無しさん:2020/06/18(木) 00:33:04 ID:???.net
どの問題だ?

234 :ご冗談でしょう?名無しさん:2020/06/18(木) 00:44:15.24 ID:???.net
>>231
そもそも物理的に存在しない系にしかならん

235 :ご冗談でしょう?名無しさん:2020/06/18(木) 00:51:03.96 ID:???.net
伸び縮みする紐が物理的に存在しない?

236 :ご冗談でしょう?名無しさん:2020/06/18(木) 01:18:27.55 ID:???.net
クラインゴルドン型の波動方程式に従う有名な系だが…

237 :ご冗談でしょう?名無しさん:2020/06/18(木) 04:21:59.90 ID:saJAYS8s.net
自己解決した。
やっぱりこのラグランジアンが間違ってたわ。

θ_k での微分を考えると、Σの中身がi=kに該当するやつ以外にも、i=k-1に該当するやつにもθ_kは含まれるので、
それを考慮すると正しいラグランジアンは

L=Σ_{i=0}^{N+1} (1/2)[ ml^2(θ_i)^2 - mgl(θ_i)^2 - Kl^2( θ_i - θ_{i+1} )^2 ]

ただしシグマの両端で辻褄があうようにθ_0=θ_1, θ_N = θ_{N+1}とおいた

このラグランジアンで連続系に移行する。

lθ_i → ψ(x, t)と書き、質量線密度 m/a → μ
および、一個の振子の持つ特徴的な角周波数
ω_g^2 = g/l, ω_k^2 = K/m
張力T=Kaを使うと、

L=∫dx (μ/2)[ (∂ψ/∂t)^2 - T/μ (∂ψ/∂x)^2 - μω_g^2 ψ^2 ]

となり、これに対しオイラー・ラグランジュ方程式を書き下すと、

∂^2ψ/∂x^2 - (μ/T) ∂^2ψ/∂t^2 - (μ^2 ω_g^2/T)ψ = 0

と、正しく伝搬速度c = √(T/μ)のクライン・ゴルドン型の波動方程式が得られた。

238 :ご冗談でしょう?名無しさん:2020/06/18(木) 08:59:24.22 ID:???.net
今日も「解けない側」の圧勝かぁ・・・。
毎日毎日、ワケ分からん問題ばかりだから常勝なんだよね・・・。
たまには、解ける解けるって悩んで負けてみたい、それが今の切実な悩み。

239 :ご冗談でしょう?名無しさん:2020/06/18(木) 14:45:34.83 ID:???.net
劣等感はウザイ

240 :ご冗談でしょう?名無しさん:2020/06/19(金) 22:42:02.28 ID:???.net
流体の質問です

ノズルからある流量吸引した時,ノズル軸上でノズルからよどみ点までの距離は
どうやって導出したらいいでしょうか.

241 :ご冗談でしょう?名無しさん:2020/06/20(土) 02:43:50.32 ID:???.net
マルチ おまけに荒らし

242 :ご冗談でしょう?名無しさん:2020/07/15(水) 20:36:21.17 ID:???.net
物理ではスカラーは座標変換で不変な数として見るようですが、ベクトル空間へ作用する係数体じゃだめなんですか?
ニュートン力学と相対論の文脈で時間がスカラーになったりならなかったりして混乱するんですが。

243 :ご冗談でしょう?名無しさん:2020/07/15(水) 21:51:28.79 ID:pvXyBq3x.net
物理ではと言うか数学でもスカラーは変換性で定義されるんだが
相対論は勿論だがニュートン力学でも時間がスカラーとか聞いたことないぞ

244 :ご冗談でしょう?名無しさん:2020/07/16(木) 00:14:50.78 ID:???.net
スカラーはスカラー場のことだから係数体じゃだめに決まっとる
もちろんベクトルもベクトル場でなきゃ変換性の意味がない

245 :ご冗談でしょう?名無しさん:2020/07/16(木) 00:32:38.95 ID:???.net
質点の相対論でローレンツスカラーはスカラーみたいな話もありますから、必ずしもスカラー場とは限らないですよね

物理のスカラーと数学のスカラーは別物だと考えた方が良いでしょうね

246 :ご冗談でしょう?名無しさん:2020/07/16(木) 17:02:19 ID:???.net
同じさ
応用だと他の要素も考えるだけ

247 :ご冗談でしょう?名無しさん:2020/07/16(木) 17:48:36.30 ID:???.net
他のってことは違うってことですよね

248 :ご冗談でしょう?名無しさん:2020/07/17(金) 14:19:46 ID:???.net
どうしてもそう思いたいなら好きにしな

249 :ご冗談でしょう?名無しさん:2020/07/17(金) 17:21:48 ID:56HdO0Rr.net
統計力学の質問です。

n個の格子欠陥のエントロピーの計算で、ショットキー欠陥(格子点から抜けた原子が結晶表面に移動)の時はN個の格子点からn個を選ぶ場合の数(NCn)を計算し、フレンケル欠陥(格子点から抜けた原子が格子隙間に移動)の時はN個の格子点からn個を選ぶ場合の数とn個の原子をN'個の格子隙間に入れる入れ方の場合の数(NCn×NCn)を計算するそうですが、なぜショットキー欠陥のときは抜けた原子の表面での配置の仕方の場合の数を掛けなくて良いのでしょうか。

250 :ご冗談でしょう?名無しさん:2020/07/17(金) 17:39:04 ID:56HdO0Rr.net
見たのはこのサイトです。

ショットキー欠陥
https://www.morikita.co.jp/data/mkj/077391mkj.pdf
フレンケル欠陥
https://youtu.be/aONy7EIoqCw

251 :ご冗談でしょう?名無しさん:2020/07/17(金) 20:44:33.65 ID:???.net
表面に行ったんじゃなく単に無いだけだろ

252 :ご冗談でしょう?名無しさん:2020/07/17(金) 21:28:16.25 ID:???.net
何言ってんだこのバカは
分からないなら無理してレスするなよ
俺もわからんが

253 :ご冗談でしょう?名無しさん:2020/07/17(金) 21:52:30.20 ID:56HdO0Rr.net
回答ありがとうございます。
今取り組んでる問題文には何故かショットキー欠陥の方にも格子から抜けた原子の行き先のサイト数が与えられているのですがこれを使う解き方もあるのでしょうか。

254 :ご冗談でしょう?名無しさん:2020/07/18(土) 00:24:53.27 ID:???.net
そんなものが自由度になるとは思えんな
自由度がなければ場合の数などない

255 :ご冗談でしょう?名無しさん:2020/07/18(土) 10:31:06 ID:???.net
>>249

>>251 のいうとおり
数えるのは「移動」の場合の数じゃないから

256 :ご冗談でしょう?名無しさん:2020/07/18(土) 11:27:06.87 ID:???.net
マクスウェル方程式の積分形は、積分領域が時間変化する場合でも正しいですか?

257 :ご冗談でしょう?名無しさん:2020/07/18(土) 12:00:47.18 ID:k7P+QgWj.net
>>254-255
格子隙間にどのように配置するかを数えるのと同様に表面に移動したときの吸着サイトへの配置のパターンを数えなくていいのは何故でしょうか。

258 :ご冗談でしょう?名無しさん:2020/07/18(土) 12:11:42.32 ID:???.net
表面での組み合わせの数は体積での組み合わせの数にくらべて無視できる

259 :ご冗談でしょう?名無しさん:2020/07/18(土) 12:30:31.42 ID:14zp7Cta.net
これってどっから持ってきた自然の岩を置いたらダメなの?
たまたま飛んできたことにして

260 :ご冗談でしょう?名無しさん:2020/07/18(土) 18:23:42 ID:UfmB/+oy.net
関数f(x(t),t)のx,tに関する偏微分ってどうなるんでしょうか?
x(t)由来のt(x)依存性と陽なt(x)依存性の療法を叩く必要があると思うんですが、単純に連鎖律が使えない気がしてよくわかりません。

全微分は
df/dt=∂f/∂x dx/dt+∂f/∂t
と書けるかと思いますが、全微分は使わず、偏微分だけで書くことはできないのでしょうか?

261 :ご冗談でしょう?名無しさん:2020/07/18(土) 18:46:55 ID:???.net
それは全微分ではありません

262 :ご冗談でしょう?名無しさん:2020/07/18(土) 19:30:27.81 ID:???.net
関数と独立変数を明記しないと何をしたいのかわからない
全微分は df = (∂f/∂x) dx + (∂f/∂t) dt だが
これを偏微分だけで書くってどう言う意味?

263 :ご冗談でしょう?名無しさん:2020/07/18(土) 19:56:51.60 ID:???.net
f(x,t)=xt
x=t^2

∂f(x,t)/∂x=t

∂f(x,t)/∂t=x=t^2

df(x,t)/dt=3t^2

ここら辺見てうんうん唸ってればいつかはわかるようになるでしょう

264 :ご冗談でしょう?名無しさん:2020/07/18(土) 20:01:50.41 ID:UfmB/+oy.net
>>262
知りたいのは、∂f/∂tを∂f/∂xや∂x/∂tで書くことはできるか?ということです

f(x(u,v),y(u,v))を(u,vの関数と見なして)uで偏微分するときには
∂f/∂u=(∂f/∂x)(∂x/∂u)+(∂f/∂y)(∂y/∂u)
となりますよね(連鎖律)
これに対して、f(x(u,v),u)を(u,vの関数と見なして)uで偏微分するとどうなるのかがわからないんです。
上の式で単にy(u,v)=uとすると、
∂f/∂u=(∂f/∂x)(∂x/∂u)+∂f/∂u (?)
となっておかしくなってしまうと思うんですが…

265 :ご冗談でしょう?名無しさん:2020/07/18(土) 20:20:16.56 ID:???.net
おかしくないですよ

∂f/∂tと書いた時、何と何を独立変数とした時の偏微分かが曖昧だから混乱するんだと思います
横にちゃんと独立変数整理してみれば、同じ記号で表されてるものの中で違うものが含まれてることがわかると思います

266 :ご冗談でしょう?名無しさん:2020/07/18(土) 20:27:31 ID:UfmB/+oy.net
>>264
書いてて気づきましたが、何を独立変数と見なすかが重要なんですね。
∂f(x(u,v),u)/∂u=(∂f(x,u)/∂x)(∂x(u,v)/∂u)+∂f(x,u)/∂u
なので(左辺の∂f/∂uと右辺のそれは違うものなので)問題ない、ということでしょうか

267 :ご冗談でしょう?名無しさん:2020/07/18(土) 20:28:44 ID:UfmB/+oy.net
>>265
行き違いになりましたが、ありがとうございます。理解が深まりました。

268 :ご冗談でしょう?名無しさん:2020/07/19(日) 00:02:03 ID:LtbaVPLd.net
最小作用の原理を使えば量子コンピュータあれば物理演算一瞬なのですか?

269 :ご冗談でしょう?名無しさん:2020/07/19(日) 02:57:27 ID:???.net
何でもできるわけじゃない
そういう計算もあるというだけ

270 :ご冗談でしょう?名無しさん:2020/07/19(日) 06:59:36 ID:LtbaVPLd.net
どのような物理演算ができないのでしょうか

271 :ご冗談でしょう?名無しさん:2020/07/19(日) 11:27:53.08 ID:???.net
不明

272 :ご冗談でしょう?名無しさん:2020/07/19(日) 13:09:36.63 ID:0GKfJOfP.net
最小作用の原理は初期状態が与えられたら次にどのような運動をするか分かりますか?
始点と終点が分かってるものにしか使えないのでしょうか

273 :ご冗談でしょう?名無しさん:2020/07/19(日) 15:11:33.08 ID:???.net
使ってるところを見るんだな

274 :ご冗談でしょう?名無しさん:2020/07/19(日) 21:13:15 ID:???.net
使えねえな

275 :ご冗談でしょう?名無しさん:2020/07/19(日) 21:21:14 ID:9ZuTmU8y.net
>>258
そういうことなんですかね。
回答ありがとうございました。

276 :ご冗談でしょう?名無しさん:2020/07/19(日) 21:23:33 ID:???.net
>>272
始点と終点を固定しないとしたら、一体何を最小化するの?

277 :ご冗談でしょう?名無しさん:2020/07/19(日) 21:38:44 ID:???.net
質問に質問で返すなよw

278 :ご冗談でしょう?名無しさん:2020/07/19(日) 21:45:03 ID:???.net
質問に質問で返すのはどうしていけないのですか?
質問の不備を明らかにするための質問は議論の基本ではないでしょうか?

279 :ご冗談でしょう?名無しさん:2020/07/19(日) 21:46:11 ID:???.net
しかもただのおうむ返しって…
役立たずにも程が

280 :ご冗談でしょう?名無しさん:2020/07/19(日) 21:53:29.70 ID:???.net
はいはい、シーライオニング

281 :ご冗談でしょう?名無しさん:2020/07/20(月) 00:19:43.91 ID:???.net
俺らは深く物理を理解してないから具体的な事例以外は的確な答えを出せないんや
許してくれ

282 :ご冗談でしょう?名無しさん:2020/07/20(月) 00:47:01.41 ID:???.net
たしかラグランジュの運動方程式を導出するときには端点条件を使った気がするんだけど実際にラグランジアンを求めて運動方程式使うときは端点云々なんか考えてなかった気がするけど始点終点の話ってどこいっちゃったんだろう

283 :ご冗談でしょう?名無しさん:2020/07/20(月) 00:49:39.66 ID:???.net
汎関数積分の中に消えた

284 :ご冗談でしょう?名無しさん:2020/07/20(月) 01:18:23.24 ID:???.net
マクスウェル方程式の積分形式と微分形式みたいなもん
大局的な法則を微視的な法則として表現してる

285 :ご冗談でしょう?名無しさん:2020/07/20(月) 01:24:02.80 ID:???.net
電磁場にも作用はあるので積分形は適当な対応物じゃないですね
あと微分形のこと微分形式って言うのやめてくださいね

286 :ご冗談でしょう?名無しさん:2020/07/20(月) 01:27:25.79 ID:???.net
揚げ足取りのためなら意気揚々と動くネラーの鑑

287 :ご冗談でしょう?名無しさん:2020/07/20(月) 01:37:47.54 ID:???.net
ネット斜め読みした限りだと微分方程式自体もモンテカルロ法使えば量子コンピュータで高速で解かせられるぽいな
流体解析が量子コンピュータで一瞬になればナビエストークス方程式が解けなくても絶望する必要もなく夢がひろがリングやなw

288 :ご冗談でしょう?名無しさん:2020/07/20(月) 04:16:16 ID:NgAHs1/o.net
>>282
オイラーラグランジュ方程式の導出で使うのはδq(t1)=δq(t2)=0つまり「変分が時間端点で0」
変分は仮想的なものだから実際の物理にはまったく顔を出して来ない

289 :ご冗談でしょう?名無しさん:2020/07/20(月) 13:49:23 ID:???.net
端点なんて積分が有限でないと計算できないから付けただけで
いくらでも延長できる無意味なパラメーターだから無駄な影響がないように固定しただけさ
チョイと延長すれば動けるから形式的固定にすぎん

290 :ご冗談でしょう?名無しさん:2020/07/20(月) 13:55:20 ID:???.net
わからないんですね

291 :ご冗談でしょう?名無しさん:2020/07/20(月) 18:44:34.40 ID:???.net
わからないのか?

292 :ご冗談でしょう?名無しさん:2020/07/25(土) 23:47:52.00 ID:???.net
>>256
お願いします

293 :ご冗談でしょう?名無しさん:2020/07/26(日) 02:57:01 ID:???.net
積分形を使う利点て何?

294 :ご冗談でしょう?名無しさん:2020/07/27(月) 01:00:34.51 ID:xqHLfzd2.net
>>293
利点どころか、物理法則を正しく表わしてはいないのである。

295 :ご冗談でしょう?名無しさん:2020/08/02(日) 19:09:18 ID:???.net
そなの?

296 :ご冗談でしょう?名無しさん:2020/08/02(日) 20:15:43.20 ID:???.net
>>256
お願いします

297 :ご冗談でしょう?名無しさん:2020/08/03(月) 00:00:05 ID:???.net
君の知ってる積分形とやらを書いてみたら?

298 :ご冗談でしょう?名無しさん:2020/08/04(火) 01:04:31.79 ID:???.net
荒らしは応じない

299 :256:2020/08/09(日) 18:59:34.92 ID:???.net
自己解決しました

300 :ご冗談でしょう?名無しさん:2020/08/10(月) 10:30:38.44 ID:hTNCi2vJ5
磁場中を動く導体棒に発生する誘導起電力に関する質問です。
長さlの導体棒が速さvで一様磁場Bの中を動いているとき、
静止している座標系(S1)で見ると棒には、電荷が磁場から
受けるローレンツ力により、lvBの起電力が発生します。


これを棒とともに速さvで動く座標系(S2)で見た場合の
解釈について質問があります。ローレンツ因子をγとして、
電荷が誘導電場から受けるローレンツ力により、γlvBの
起電力が発生するという解釈であっていますでしょうか。

ただこの場合、vが光速に比べて十分小さければγ〜1となり、
S1系とS2系で発生した起電力は同じと考えられますが、vが
光速に比べて無視出来ないときはS1系とS2系で発生した起電
力は違ってしまうことになり、座標系により、起電力が違って
しまうのですが、問題ないのでしょうか。

よろしくお願い致します。

301 :ご冗談でしょう?名無しさん:2020/08/12(水) 22:48:58.50 ID:WJYzEGCF.net
2ちゃんねるに書いてはる人へ:
今は、2ちゃんねるに書いても、みんな見えまへん。
5ちゃんねる→2ちゃんねる=転送されます
2ちゃんねる→5ちゃんねる=転送されまへん

…残念!

302 :ご冗談でしょう?名無しさん:2020/08/13(木) 05:43:05 ID:uQtfx9C8.net
xの関数yについての方程式
y(ax)=ay(x) (aは任意定数)
が解けません。
いや、解は(もちろん)わかっていますが、解の解析的な求め方がわからんのです。
どうすればいいですか?

303 :ご冗談でしょう?名無しさん:2020/08/13(木) 08:50:50 ID:???.net
>>302
任意定数 a で
y(ax) = ay(x)
なのであるから任意の x についても
y(x) = y(x・1) = x・y(1)
が成り立つ。 y(1) = c (定数)で改めて書き直せば
y(x) = c・x

304 :302:2020/08/13(木) 10:27:54.89 ID:???.net
>>302
f(x) = f(1x) = f(1)x (f(1)は定数)
の方いいかな。

305 :ご冗談でしょう?名無しさん:2020/08/13(木) 11:09:07.67 ID:sL+Ohk8L.net
磁場中を動く導体棒に発生する誘導起電力に関する質問です。
長さlの導体棒が速さvで一様磁場Bの中を動いているとき、
静止している座標系(S1)で見ると棒には、電荷が磁場から
受けるローレンツ力により、lvBの起電力が発生します。


これを棒とともに速さvで動く座標系(S2)で見た場合の
解釈について質問があります。ローレンツ因子をγとして、
電荷が誘導電場から受けるローレンツ力により、γlvBの
起電力が発生するという解釈であっていますでしょうか。

ただこの場合、vが光速に比べて十分小さければγ〜1となり、
S1系とS2系で発生した起電力は同じと考えられますが、vが
光速に比べて無視出来ないときはS1系とS2系で発生した起電
力は違ってしまうことになり、座標系により、起電力が違って
しまうのですが、問題ないのでしょうか。

よろしくお願い致します。

306 :ご冗談でしょう?名無しさん:2020/08/13(木) 11:23:45.74 ID:???.net
電場や磁場のローレンツ変換を考慮しないとそういう変な結果になります

307 :ご冗談でしょう?名無しさん:2020/08/13(木) 11:39:48.71 ID:???.net
>>305
特殊相対性理論ではローレンツ力 F=q(E+VxB) が不変になる。
慣性座標系を変えればそれが成り立つようにE,Bがローレンツ変換で値が変る
常にE=0だと勘違いしやすい。

308 :ご冗談でしょう?名無しさん:2020/08/13(木) 12:02:40 ID:+F3azp0I.net
3次以上の微少量xyを無視できる場合、x‘y’(x+y)^2って無視できる?

309 :ご冗談でしょう?名無しさん:2020/08/13(木) 12:55:21 ID:???.net
ローレンツ力が不変って何言ってんだコイツ

310 :ご冗談でしょう?名無しさん:2020/08/13(木) 14:21:05.29 ID:???.net
>>305
棒とともに動く座標系では棒が動かないからローレンツ力はゼロ
その代わり磁場のローレンツ変換で現れる電場による力になる

311 :ご冗談でしょう?名無しさん:2020/08/13(木) 14:31:22.02 ID:???.net
>>310
中高生のローレンツ力

312 :ご冗談でしょう?名無しさん:2020/08/13(木) 17:35:31 ID:???.net
>>305
> 電荷が誘導電場から受けるローレンツ力により、γlvBの
> 起電力が発生するという解釈であっていますでしょうか。

合ってる。

>座標系により、起電力が違ってしまうのですが、問題ないのでしょうか。

問題無い。


>>306-307
何ほざいてんだか

313 :ご冗談でしょう?名無しさん:2020/08/13(木) 18:11:09 ID:00xoI3kx.net
久保統計の5章A3(p237)で、
(速度分布がMaxwell分布である)気体が穴から吹き出る気体分子の速度分布を求めよ
とあって、
解答ではv exp(-mv^2/2kT) dvで重みをとっているのですが、これがよくわかりません。
速度分布はMaxwell分布だと仮定しているのだから、吹き出る速度分布もMaxwell分布になるのではないですか?
なぜvをかける必要があるのでしょうか?

314 :ご冗談でしょう?名無しさん:2020/08/13(木) 18:17:02 ID:???.net
分布と速度分布は違います

315 :ご冗談でしょう?名無しさん:2020/08/13(木) 18:24:50 ID:PtxjaOcM.net
>>314
どう違うのですか?わからないので教えてください。
しかし、原文ママで「速度分布はMaxwell分布」「速度分布を求めよ」とあるので、同じものを指していると思うのですが。

316 :ご冗談でしょう?名無しさん:2020/08/13(木) 18:35:25 ID:???.net
多様体の中に物理法則を置くのと
物理法則がそれに合う多様体を生成するのって
どっちが考え方としては正しいの?

317 :ご冗談でしょう?名無しさん:2020/08/13(木) 19:04:52.94 ID:???.net
>>316
相対性原理、ゲージ原理が

(局所)座標系の選び方に依存しない物理法則の記述

と見なせる。

318 :ご冗談でしょう?名無しさん:2020/08/13(木) 19:05:35.14 ID:???.net
>>315
分布は確率
速度分布は期待値

319 :ご冗談でしょう?名無しさん:2020/08/13(木) 19:12:50.64 ID:???.net
何いってだこいつ

320 :ご冗談でしょう?名無しさん:2020/08/13(木) 19:20:56.91 ID:T95mJsw0.net
>>318
ちょっとよくわかりません。
Maxwellの速度分布は(これは久保の記述でも)分子が速度(v_x,v_y,v_z)を取る「確率」だと思うのですが。
「分布」という以上規格化されているので無次元量ですし。
速度分布が期待値だとすれば、速度分布で期待値を取ったら何が得られるのですか?

321 :ご冗談でしょう?名無しさん:2020/08/13(木) 19:28:11.20 ID:???.net
>>313
解答読んでないの?

322 :ご冗談でしょう?名無しさん:2020/08/13(木) 19:30:15.07 ID:???.net
>>317
言ってることはよく分かるけど本質的に同じものでは?

323 :ご冗談でしょう?名無しさん:2020/08/13(木) 19:49:29 ID:uJ73wz3a.net
>>321
「解答では…」と書いたとおり、読んだうえで質問しています。

324 :ご冗談でしょう?名無しさん:2020/08/13(木) 19:59:55 ID:???.net
>>323
そういう答えになる理由は書いてあるけど
なにが聞きたいのだろう

325 :ご冗談でしょう?名無しさん:2020/08/13(木) 20:20:20.52 ID:???.net
x方向についての速度分布を聞かれているのでちゃんと規格化定数まで求める必要がある

326 :ご冗談でしょう?名無しさん:2020/08/13(木) 20:23:29 ID:7XKfSDs1.net
>>324
理由と言ってもいきなり「積分の中で与えられている」と書いてあるだけですよね?
なぜこうなるのかが教えてもらえませんか。

327 :ご冗談でしょう?名無しさん:2020/08/13(木) 20:39:41.06 ID:???.net
>>322
だから最近は幾何学と場の物理学はかなり混然一体化してる。

328 :ご冗談でしょう?名無しさん:2020/08/13(木) 23:20:59.74 ID:???.net
>>326
この積分がこう書ける理由が書いてあるだろ

329 :ご冗談でしょう?名無しさん:2020/08/14(金) 00:05:22 ID:x8DwVha7.net
>>328
もれる速さがこう書ける理由はわかっています。
この積分(の中身)が求める速度分布になっていると言えるのはなぜですか?

330 :ご冗談でしょう?名無しさん:2020/08/14(金) 00:55:49 ID:???.net
Maxwell分布は3次元
吹き出る速度分布は1次元

331 :ご冗談でしょう?名無しさん:2020/08/14(金) 02:26:01 ID:KYBER83R.net
>>330
1次元だとなぜvがかかるのですか?

332 :ご冗談でしょう?名無しさん:2020/08/14(金) 02:30:08 ID:???.net
期待値

333 :ご冗談でしょう?名無しさん:2020/08/14(金) 03:48:30 ID:+sxircEY.net
>>332
粒子の減少の速さを計算するのに期待値を使うのは理解できますが、分子の速度分布も期待値になる理由がわかりません。
なぜMaxwell分布のままではダメなんでしょうか?(>>320も参照してください)
また、期待値を取ることに、1次元であることは関係あるんですか?

334 :ご冗談でしょう?名無しさん:2020/08/14(金) 05:12:25.59 ID:???.net
問題文をよく読め

335 :ご冗談でしょう?名無しさん:2020/08/14(金) 05:45:39.75 ID:4BUsRLQH.net
>>334
よく読んでもわからないので質問しています。見落としがあると言うのであれば、どの部分か教えてもらえませんか?

336 :ご冗談でしょう?名無しさん:2020/08/14(金) 09:08:16.41 ID:???.net
解答に書いてあることはわかるが
>>313 の疑問も一理あるような気がする

337 :ご冗談でしょう?名無しさん:2020/08/14(金) 09:24:36.72 ID:???.net
二つの問いをごちゃ混ぜにして混乱してるだけだろ
「速さを求めろ」、は期待値を計算するだけだし、「分布を書け」、はガウス関数型の分布式を書けるかどうかを問われている。

338 :ご冗談でしょう?名無しさん:2020/08/14(金) 11:11:19 ID:1MsP8geR.net
>>337
問題で求めよと言われているのは「気体が穴から吹き出る速さ(粒子数の時間変化-dN/dt)」と「吹き出る分子の速度分布」です。
前者を計算する過程で(分子の)速度の期待値を計算しましたが、後者の解答では「その積分の中身(つまりv exp)が求める分布である」というような書き方がされています。
これがわかりません。

なぜexp型(ガウス型)でなくv exp型になるのか、またなぜ吹き出る速さを計算するときの非積分関数が求める分布だと言えるのかがわかりません。

339 :ご冗談でしょう?名無しさん:2020/08/14(金) 11:23:37 ID:1MsP8geR.net
>>336
いまいちはっきりした答えがもらえないので、再検討のしようもない状態です。
書いてあることを少し砕いて説明してもらえませんか。

340 :ご冗談でしょう?名無しさん:2020/08/14(金) 11:28:34 ID:???.net
>>338
前者でどういう条件の期待値を計算したのか分ってないんじゃないの
前のページからちゃんと読んだのか

341 :ご冗談でしょう?名無しさん:2020/08/14(金) 11:31:46 ID:1MsP8geR.net
わからないこと・知りたいことはただひとつで、
「漏れ出る分子の速度分布は(1)の積分の中で与えられている。すなわち速度がx方向にv_xとv_x+dv_xの間にある確率はv_x exp(-mv_x^2/2kT)dv_xに比例する。」
なぜこれが言えるのか?ということです。それだけです。

342 :ご冗談でしょう?名無しさん:2020/08/14(金) 12:39:03.14 ID:???.net
>>318
分布関数が確率で、速度分布が期待値、だろ

343 :ご冗談でしょう?名無しさん:2020/08/14(金) 14:25:35.12 ID:???.net
>>333
元の全方向の3次元分布を噴射方向の1次元分布で取り出した以上
元の分布ではない

344 :ご冗談でしょう?名無しさん:2020/08/14(金) 14:57:20.24 ID:???.net
穴から吹き出る分子の速度分布でマイナス方向の速度の確率が含まれるはずがない。

345 :ご冗談でしょう?名無しさん:2020/08/14(金) 16:44:12.22 ID:???.net
絶対値の分布でいいのかな?

346 :ご冗談でしょう?名無しさん:2020/08/14(金) 18:43:48.63 ID:NLXNPz7UC
大学で学ぶ物理を板書1枚にまとめてみた
https://www.youtube.com/watch?v=naBcXoq4aOI
物理の研究分野を板書1枚にまとめてみた
https://www.youtube.com/watch?v=4W-pWuXUaZQ
理学部と工学部の違いとは?
https://www.youtube.com/watch?v=eJH4nKU6mJA&t=80s
大学と大学院の違い
https://www.youtube.com/watch?v=xBKAEvTegN8
高校と大学の積分は決定的に違う?微分積分学の基本定理は実はすごい!
https://www.youtube.com/watch?v=V9i_zlbssbs&t=475s
数学にはどんな研究分野がある?数学の世界地図を一枚に描いて紹介してみた!
https://www.youtube.com/watch?v=fK_JGVti5y8

347 :ご冗談でしょう?名無しさん:2020/08/14(金) 18:26:13.44 ID:???.net
>>341
前のページに書いてあることがわかって(1)が導出できるんだとしたら
何がわからない,といってるのか,全くわからない

348 :ご冗談でしょう?名無しさん:2020/08/14(金) 20:42:26.76 ID:???.net
>>341
https://ja.m.wikipedia.org/wiki/マクスウェル分布
ここに書いてある事は全部理解できる?

349 :ご冗談でしょう?名無しさん:2020/08/15(土) 21:01:46.73 ID:???.net
前野のツイッター止まってるけどもしかしてやられた?

350 :ご冗談でしょう?名無しさん:2020/08/15(土) 21:45:51.67 ID:hLr2iq6P.net
>>349
だめですコロナさん、あいつは結構使える男です。
まだ連れていかないで!

351 :ご冗談でしょう?名無しさん:2020/08/16(日) 07:54:57 ID:???.net
水素原子の陽子と電子の量子力学から計算される結合エネルギーは13.6電子ボルト
1電子ボルトは温度にして約1万度に対応
ばらばらな陽子と電子が温度が下がって陽子と電子が結合して水素原子となった方が
熱力学的に安定する温度が約3000度
単純に考えるとその温度は13.6万度ではないかと思います。なぜ3000度になるのでしょうか

352 :ご冗談でしょう?名無しさん:2020/08/16(日) 10:35:19 ID:???.net
サハの電離公式

353 :ご冗談でしょう?名無しさん:2020/08/16(日) 12:19:37.49 ID:???.net
エントロピーを考慮するかどうか

354 :ご冗談でしょう?名無しさん:2020/08/16(日) 13:15:40.34 ID:???.net
他に分配されるからに決まっとる

355 :ご冗談でしょう?名無しさん:2020/08/16(日) 19:38:55.01 ID:P6kiEwd/.net
>>351
太陽の表面温度って、何で六千度なん?
コロナ(チョー熱っつい方)って、何で…えーと、
何度だっけ?

356 :ご冗談でしょう?名無しさん:2020/08/16(日) 21:27:09.20 ID:???.net
熱源は太陽中心部の核反応だけど
熱の伝達機構も散逸機構も違うからな

357 :ご冗談でしょう?名無しさん:2020/08/16(日) 21:31:33.12 ID:???.net
そもそも太陽の機構なんて未だに殆どわかってないだろ

358 :ご冗談でしょう?名無しさん:2020/08/16(日) 22:01:09.08 ID:???.net
なぜ太陽を持ち出したのか理解不能

359 :ご冗談でしょう?名無しさん:2020/08/16(日) 23:26:13 ID:???.net
>>357
地球よりも分かってる

360 :ご冗談でしょう?名無しさん:2020/08/17(月) 20:11:24.97 ID:E/YkvPRg.net
>>359
単純で分かりやすいからですね。
ニンゲン、足元の方が見えにくい、という事情は、
今も昔も変わらないようでして…

361 :ご冗談でしょう?名無しさん:2020/08/19(水) 14:58:02 ID:H08Cvpwh.net
(2)から教えてください。


http://uproda11.2ch-library.com/e/es002917766715874611276.jpeg

362 :ご冗談でしょう?名無しさん:2020/08/19(水) 15:38:26 ID:???.net
きっと r・(1/r)=1 を使えば大丈夫

363 :ご冗談でしょう?名無しさん:2020/08/19(水) 15:49:37 ID:???.net
∇・{(1/r)aベクトル}=-2/rじゃないのか

364 :ご冗談でしょう?名無しさん:2020/08/19(水) 16:00:34 ID:???.net
マイナスは付かない

365 :ご冗談でしょう?名無しさん:2020/08/19(水) 17:22:25 ID:???.net
量子力学で状態ベクトルで書いた確率が <ψ|ψ> だったり |<p|ψ>|^2 だったりしますが
たとえば状態ベクトルを |c> として適当なベクトル |a>, |b> で |c> = |a> + |b> と分解すると
<c|c> = <a|a> + <a|b> + <b|a> + <b|b>
というようにただの図形ベクトル的イメージだけで考えようとすると <a|b> みたいなのが残ります。

<a|a> と <a|b> などが同じ次元で並んじゃってその意味合いに悩んだのですが、これは
初学者向けに端的に説明される確率 <ψ|ψ> などはベクトルが直交してる前提で
語られてるもので、直交しない無制限なベクトルで分解かけるようなひねくれたことをすると
量子力学が前提にしてる数学の枠組み的に意味のある計算にならないよってことでいいんでしょうか。

366 :ご冗談でしょう?名無しさん:2020/08/19(水) 17:55:46 ID:???.net
状態ベクトルは無次元

367 :ご冗談でしょう?名無しさん:2020/08/19(水) 18:04:52 ID:???.net
>>365
線型代数からやり直した方がいいよ

368 :ご冗談でしょう?名無しさん:2020/08/19(水) 19:04:39.92 ID:???.net
>>365
わからないんですね

369 :ご冗談でしょう?名無しさん:2020/08/20(木) 01:18:32 ID:???.net
自分で余計な思い込みをつけて分からなくなる奴だな
固定観念に気づかないと何も学べん

370 :ご冗談でしょう?名無しさん:2020/08/20(木) 12:58:33.87 ID:???.net
>>361

見れん。なぜ消したの?

371 :ご冗談でしょう?名無しさん:2020/08/20(木) 12:59:10.77 ID:???.net
>>368
わからんです。
異なる状態でも波動関数同士だったら内積が即確率になって運動量等の
状態との内積だと振幅のままでというように演算規則が状態ベクトルの
中身を見た上でのケースバイケースのように見えていて、もうちょい
統一的というか無精できる見方はないのかなと。
任意の状態ベクトルの内積はすべからく振幅で二乗取ったら確率みたいに
演算操作自体に量が紐付けされてるなら見え方が平坦になるんですけど。

372 :ご冗談でしょう?名無しさん:2020/08/20(木) 13:04:04.16 ID:???.net
<ψ|ψ>は確率じゃなくて確率振幅だが

373 :ご冗談でしょう?名無しさん:2020/08/20(木) 13:35:33 ID:???.net
>>372
わからないんですね

>>371
|p>とか|ψ>はどう違うか説明してみてください

374 :ご冗談でしょう?名無しさん:2020/08/20(木) 14:47:13.95 ID:60VCzCz0.net
学術巨大掲示板群: アルファ・ラボ ttp://x0000.net
物理学 化学 数学 生物学 天文学 地理地学
IT 電子 工学 国語 方言 言語学 など

375 :ご冗談でしょう?名無しさん:2020/08/20(木) 15:32:53 ID:???.net
異なる状態の内積が確率になるわけねーよ
条件付き確率の1因子くらいだ

376 :ご冗談でしょう?名無しさん:2020/08/20(木) 16:01:18 ID:???.net
>>373
分かってないのはお前だろ

377 :ご冗談でしょう?名無しさん:2020/08/20(木) 16:11:51 ID:???.net
>>375
わからないんですね

>>376
わからないんですね

378 :ご冗談でしょう?名無しさん:2020/08/20(木) 16:31:06 ID:???.net
>>377
わからないんですね

379 :ご冗談でしょう?名無しさん:2020/08/20(木) 16:32:32 ID:???.net
>>378
わからないんですね

380 :ご冗談でしょう?名無しさん:2020/08/20(木) 16:33:00 ID:???.net
>>379
わからないんですね

381 :ご冗談でしょう?名無しさん:2020/08/20(木) 16:34:33 ID:???.net
>>380
わからないんですね

382 :ご冗談でしょう?名無しさん:2020/08/20(木) 16:55:59 ID:???.net
>>381
わからないんですね

383 :ご冗談でしょう?名無しさん:2020/08/20(木) 16:59:49 ID:???.net
>>373
質問に質問で返す馬鹿

384 :ご冗談でしょう?名無しさん:2020/08/21(金) 00:38:03.16 ID:sNDY+oUQ.net
なんや、お婆はんやったんかw

385 :ご冗談でしょう?名無しさん:2020/08/21(金) 01:00:05 ID:???.net
答えても分からない馬鹿がいるようだ

386 :ご冗談でしょう?名無しさん:2020/08/21(金) 04:25:49 ID:???.net
>>371
無理して賢ぶらない方がいい

387 :ご冗談でしょう?名無しさん:2020/08/21(金) 18:29:36.47 ID:GEoMYhW6.net
2007年までの科研費と、JST CREST以外で、数十ページくらいの研究終了報告書が義務付けられている研究予算って他にありますか?

388 :ご冗談でしょう?名無しさん:2020/08/21(金) 20:22:13.32 ID:???.net
検索しても答えが見つからないのでお願いします。

一様磁場の中で四角形コイルがあります。
磁場が下方向から増加しているとすると、コイルには図のような方向で
誘導起電力が発生しますよね?

このコイルのとなりに別のコイルを作ると同じように誘導起電力が発生しますが、
共通の1辺ABの誘導起電力は逆方向になってしまいます。

この1辺だけがあった場合、どちらの方向に誘導起電力は発生するのでしょうか?
http://o.5ch.net/1p8oe.png

389 :ご冗談でしょう?名無しさん:2020/08/21(金) 20:36:24.50 ID:???.net
発生しない
誘導起電力の応用問題をググって出てくるかなー?

390 :ご冗談でしょう?名無しさん:2020/08/21(金) 20:43:10.13 ID:???.net
Maxwell方程式だけでいいんじゃね?

391 :ご冗談でしょう?名無しさん:2020/08/21(金) 20:46:58.11 ID:???.net
>>389
4辺はすべて等価だから、それだとどこにも発生しないことになる。

392 :ご冗談でしょう?名無しさん:2020/08/21(金) 21:06:41.81 ID:???.net
ストークスの定理が基本だから、閉曲線の内部分割で成り立つ。

393 :ご冗談でしょう?名無しさん:2020/08/21(金) 21:50:32.13 ID:???.net
>>388
なんでこういう問題を高校や大学で教えないんだろうか。

394 :ご冗談でしょう?名無しさん:2020/08/21(金) 22:04:25.64 ID:???.net
中学レベルの数学しか使えないという謎の縛りがあるから

395 :ご冗談でしょう?名無しさん:2020/08/21(金) 22:04:39.84 ID:???.net
大学ではちゃんとやります

396 :ご冗談でしょう?名無しさん:2020/08/21(金) 22:05:40.09 ID:???.net
しかも凾/凾狽ニか言って実質微分を導入してる

397 :ご冗談でしょう?名無しさん:2020/08/21(金) 22:21:07.41 ID:???.net
くっくっくの餌みたいな質問だな

398 :ご冗談でしょう?名無しさん:2020/08/21(金) 23:21:56.56 ID:???.net
>>397
実際、奴しか答えられないんじゃないの?
これ即答できないって、物理教育終わってんじゃね

399 :ご冗談でしょう?名無しさん:2020/08/21(金) 23:28:51.04 ID:???.net
早速自演ヨイショが来たよ

400 :ご冗談でしょう?名無しさん:2020/08/21(金) 23:31:44.44 ID:???.net
>>388
高校教師はまず分かんねえよ。
大学教員でも即答できねえだろうなあ。

401 :ご冗談でしょう?名無しさん:2020/08/21(金) 23:37:03.01 ID:???.net
<ψ|ψ> だとか量子力学かぶれの連中に聞いてみたらおもしろいと思う。
一生悩みぬいて苦しんで氏ぬと思う。あと、己の浅はかにも気づくだろう。

402 :ご冗談でしょう?名無しさん:2020/08/21(金) 23:41:07.27 ID:???.net
>>393
本当に不思議ですね。
分からないんでしょうね。

403 :ご冗談でしょう?名無しさん:2020/08/21(金) 23:45:09.64 ID:???.net
>>401
今頃何人がもだえ苦しんでるのでしょうか。
今このスレを見た人々は解けずに一生苦しむことでしょう。
ああ神よ。

404 :ご冗談でしょう?名無しさん:2020/08/21(金) 23:47:31.95 ID:???.net
>>388
これに答えられないレベルで
物理学教授やってる奴ら大勢いるんだろうな。
本当におかしな世の中だよ。

405 :ご冗談でしょう?名無しさん:2020/08/21(金) 23:49:47.72 ID:???.net
>>392
ストークスの定理って言ってみたかったの?

406 :ご冗談でしょう?名無しさん:2020/08/21(金) 23:54:18.69 ID:???.net
>>388
大多数が理解したつもりになってるだけで、
そういう根本的な問題には一切答えられない。

んで、そういう連中が量子力学や相対論や素粒子論を
知ったかで語ってるんだからお笑い。何も理解していないくせにね。

答え知ってるけど書かない。一生悩んで氏ねばいいよ。

407 :ご冗談でしょう?名無しさん:2020/08/21(金) 23:59:38.66 ID:???.net
うわ自演だったか

408 :ご冗談でしょう?名無しさん:2020/08/22(土) 00:00:18 ID:???.net
>>400
そりゃ、電磁誘導とローレンツ力を区別しないで
どちらも同じだ、同じ式だって教えてるからね。
どんだけ今の理論物理学がいかがわしいものばかりなのかを象徴してるっしょ。
言ったもん勝ちで誰も検証しない。それがエスカレートしててもはやオカルト。

ベクトルポテンシャルが実在するとか、光子1個を撮影できたとか
言いたい放題で呆れるしかないよ。

409 :ご冗談でしょう?名無しさん:2020/08/22(土) 00:03:24 ID:???.net
>>393
気付きもしないから無理。
本当にレベルが低い。

410 :ご冗談でしょう?名無しさん:2020/08/22(土) 00:04:13 ID:???.net
>>406
教えてやれよケチくさい

411 :ご冗談でしょう?名無しさん:2020/08/22(土) 00:06:44 ID:???.net
>>393
これを理解できていなければ
電磁誘導をまったく理解できていないのに等しいよね。
絶対に教科書に記載すべきレベル。

412 :ご冗談でしょう?名無しさん:2020/08/22(土) 00:09:37 ID:???.net
>>388
そのまま記述式で大学入試に出せばいいのにな。
予備校講師も模範解答書けずに泡吹いて泣き出すぜ。

413 :ご冗談でしょう?名無しさん:2020/08/22(土) 00:13:20 ID:???.net
>>408
導線を転がして発生する電圧はローレンツ力だよな。
それを電磁誘導の式で出させるからめっちゃ立ちが悪いわ。
そんな教え方してるから教えてる側もこれが理解できないわけ。

414 :ご冗談でしょう?名無しさん:2020/08/22(土) 00:15:39.96 ID:???.net
>>408
光子1個撮影って、浜松なんとかだっけ?
ノーベル賞まだもらってなかったかw
衝撃的だったけどw

415 :ご冗談でしょう?名無しさん:2020/08/22(土) 00:24:29.71 ID:???.net
光子が存在してるだなんて
まともな物理屋は信じていないです。口には出せませんが。

416 :ご冗談でしょう?名無しさん:2020/08/22(土) 00:25:37.51 ID:???.net
>>388
それって自分で気づいたの?
だとしたら凄いね。

417 :ご冗談でしょう?名無しさん:2020/08/22(土) 00:31:32.78 ID:???.net
自演の嵐w

418 :ご冗談でしょう?名無しさん:2020/08/22(土) 00:32:51.76 ID:???.net
>>395
わからないんですね

419 :ご冗談でしょう?名無しさん:2020/08/22(土) 00:46:19.10 ID:???.net
解答はまだか。

420 :ご冗談でしょう?名無しさん:2020/08/22(土) 00:48:55.11 ID:???.net
>>391
閉回路は違う
>>393
矛盾する状況だからさ
電磁気学は遠方でゼロで保存則が成り立つ条件でしか成立しない

421 :ご冗談でしょう?名無しさん:2020/08/22(土) 00:59:19.67 ID:???.net
>>419
苦しんで氏ぬがよい

422 :ご冗談でしょう?名無しさん:2020/08/22(土) 01:00:13.92 ID:???.net
>>420
わからないんですね。

423 :ご冗談でしょう?名無しさん:2020/08/22(土) 01:37:17.94 ID:???.net
2つの閉ループで成り立つ条件から、閉ループを取り去れば成り立たなくなるのは当然。

424 :ご冗談でしょう?名無しさん:2020/08/22(土) 01:39:30.39 ID:???.net
勝手に条件を変えて矛盾だと騒ぎだす基地外が5ちゃんに多い。

425 :ご冗談でしょう?名無しさん:2020/08/22(土) 02:08:01.66 ID:???.net
>>400
バカだなお前
即答できない大学教員なんて
ごくわずかだよ

426 :ご冗談でしょう?名無しさん:2020/08/22(土) 03:46:32 ID:???.net
>>375
分解した状態で自乗を計算したら両辺とも確率なんじゃと思って計算したら
内積の実部とる必要が出てきてたしかにダイレクトには繋がってないですね
失礼しました

>>373
ベクトル個々の意味の突き合わせをすれば式がああなるのはわかるんですが
そういうのでなくて量子力学のとくに素粒子の教員や参考書はただベクトルを
かけるとかそのレベルで現象の過渡との対応が見えてるような説明するじゃないですか
どういう考え方すればあんな見え方ができるのかなと

427 :ご冗談でしょう?名無しさん:2020/08/22(土) 05:06:42.78 ID:ugRnyqoI.net
>>408
Qちゃん、今回はなぜ署名なし?

428 :ご冗談でしょう?名無しさん:2020/08/22(土) 05:08:56.33 ID:ugRnyqoI.net
ああ、>>388 から始まってるのか。
だめだよ、Qちゃん。今度一緒に、
四国のお遍路回らない?

429 :ご冗談でしょう?名無しさん:2020/08/22(土) 05:10:04.70 ID:ugRnyqoI.net
>>425
相手がQちゃんと知っての狼藉か!

430 :ご冗談でしょう?名無しさん:2020/08/22(土) 07:18:40.08 ID:UJHSbFsD.net
>>388
以前、「一様な変動磁場が作る誘導電場」のパラドックスが話題になった。
そこでも書いたが、一様磁場には必ず、中心が存在するのだ。
その中心を定めていない場合、解は求まらない。

431 :ご冗談でしょう?名無しさん:2020/08/22(土) 09:19:05.02 ID:oBt+dl/z.net
>>387
すみませんこの質問に回答お願いします

質問スレは自演で流されてしまったので、ここに。
2007年までの科研費と、JST CREST以外で、数十ページくらいの研究終了報告書が義務付けられている研究予算って他にありますか?

432 :ご冗談でしょう?名無しさん:2020/08/22(土) 11:09:19.38 ID:???.net
>>430
一様磁場の変化では静電場の様な位置の電位差を定めることはできない。

定まった電位差が有ると勝手にも妄想するから矛盾になるだけだ。

433 :ご冗談でしょう?名無しさん:2020/08/22(土) 11:38:59.43 ID:???.net
>>422
分かってることが分からないのが、お前

434 :ご冗談でしょう?名無しさん:2020/08/22(土) 11:39:20.57 ID:???.net
>>426
ただベクトルじゃなく状態ベクトルだ
そこを軽視すると意味不明になる

435 :ご冗談でしょう?名無しさん:2020/08/22(土) 12:31:13.70 ID:???.net
実質、このスレというか物理でまともなのは
くっくっくぐらいしかいないよね?>>388なんて奴しか答えられないでしょ。

436 :ご冗談でしょう?名無しさん:2020/08/22(土) 12:33:14.75 ID:???.net
>>430
ちょっとしかわからないんですね。
先っちょだけですね。

437 :ご冗談でしょう?名無しさん:2020/08/22(土) 12:59:04.23 ID:???.net
マクロな視点でタービン回すとエネルギーが電気に変換されるとはいくらでも説明できるけど
「電線内を流れる電子はどこから発生するのか」
と聞かれたら全く説明できない

438 :ご冗談でしょう?名無しさん:2020/08/22(土) 13:00:24.89 ID:???.net
発生してないからな

439 :ご冗談でしょう?名無しさん:2020/08/22(土) 13:06:03.98 ID:???.net
>>437
>マクロな視点でタービン回すとエネルギーが電気に変換されるとはいくらでも説明できる

そんなに簡単なら何とでも言える言葉でなく、数式でそのエネルギー変換を説明してくれ。

440 :ご冗談でしょう?名無しさん:2020/08/22(土) 16:01:54.40 ID:UJHSbFsD.net
>>432
例えば、
x^2+y^2=r^2
(x-1)^2+y^2=r^2
の二つの円は、r→∞の極限で初期条件的に同一な円に近似されるが、
円心が重なることは無いので、どちらを選ぶかで、対称性を示す基準が異なる。
一様磁場は、荷電体を無限遠まで飛ばさないと作り出せない。
つまり、誘導電場の解が「飛ばし方」に依存するわけだ。

441 :ご冗談でしょう?名無しさん:2020/08/22(土) 18:19:09.74 ID:???.net
>>440
妄想からは何も出ないぞ

電磁気学には矛盾が無い。

442 :ご冗談でしょう?名無しさん:2020/08/22(土) 18:36:09.42 ID:UJHSbFsD.net
>>441
誘導電場は実在のベクトル場。
必ず「一意性」をもつ。
その起電力がループ経路の線積分でしか得られないと思ってるの?
その考え方のほうがよほど妄想的。

443 :ご冗談でしょう?名無しさん:2020/08/22(土) 20:11:31.39 ID:UJHSbFsD.net
ちなみに、電磁波は有限の先端速度しか持ち得ないので、
厳密には、一様性を保ったまま、磁場が変動することは無い。
誘導電場を導出する場合は、
あくまで、磁場の変動が、無限遠から中心へ向かって、
光速で伝播していく様子をイメージしなければならない。

444 :ご冗談でしょう?名無しさん:2020/08/22(土) 20:28:56.83 ID:???.net
>>388
これが電磁誘導の章でまったく触れられていない現状が極めて不自然すぎる。
つまり、教える側の人間も本質をまったく理解していないということだ。
>>442がいいこと言ってる。

445 :ご冗談でしょう?名無しさん:2020/08/22(土) 20:35:43.52 ID:???.net
自演うんざり

446 :ご冗談でしょう?名無しさん:2020/08/22(土) 20:35:49.27 ID:???.net
実際、電磁気学って断トツで難しいよ。
10年20年経っても疑問が出てくる。理解した実感が全然わかない。

447 :ご冗談でしょう?名無しさん:2020/08/22(土) 20:43:39.33 ID:???.net
>>446
そりゃ、コンデンサーの誘電体を引っ張り込む横方向の力を
論理的に間違った方法で求めさせる問題を大学教授が堂々と出すぐらいだからね。
さらにあり得ない空想図をネット上で公開までしてるし、真に理解してる人間はほとんどいないと思う。

448 :ご冗談でしょう?名無しさん:2020/08/22(土) 21:38:57.38 ID:???.net
自演うんざり

449 :ご冗談でしょう?名無しさん:2020/08/23(日) 01:20:32.37 ID:???.net
劣等感は捏造しか救いがない

450 :ご冗談でしょう?名無しさん:2020/08/23(日) 04:34:01.88 ID:cNaoeq7v.net
>>447
だ〜ら〜、懐石力学お勉強しなさい。

451 :ご冗談でしょう?名無しさん:2020/08/23(日) 04:35:30.43 ID:cNaoeq7v.net
>>388
これ、答えちまってもいいのか?
バラエティーなら大顰蹙を買うんだがフツー。

452 :ご冗談でしょう?名無しさん:2020/08/23(日) 04:36:48.32 ID:cNaoeq7v.net
>>437
だから発生なんかしてない!っつーのに。

453 :ご冗談でしょう?名無しさん:2020/08/23(日) 10:36:17.03 ID:???.net
>>387
報告書読んでる人って少ないんですかね?

454 :ご冗談でしょう?名無しさん:2020/08/23(日) 17:15:42.38 ID:???.net
そもそも書を読まない

455 :ご冗談でしょう?名無しさん:2020/08/23(日) 22:01:02.93 ID:???.net
素人ばっかだし

456 :ご冗談でしょう?名無しさん:2020/08/24(月) 11:20:21.18 ID:???.net
>>451
バラエティっていきなり何の話をしているんだ?

457 :ご冗談でしょう?名無しさん:2020/08/24(月) 14:23:13.68 ID:???.net
そもそも既に答えられてるし

458 :ご冗談でしょう?名無しさん:2020/08/24(月) 17:06:05.64 ID:???.net
https://imgur.com/ZhzG1Wh
赤い四角で囲った積分は0になるようですが、∞に発散しませんか?

459 :ご冗談でしょう?名無しさん:2020/08/24(月) 18:47:51.24 ID:???.net
>>458

コーシーの主値を考えればゼロにできるかな

460 :ご冗談でしょう?名無しさん:2020/08/24(月) 21:48:22.58 ID:???.net
>>459
デルタ関数が入ってるので主値はムリっぽくないですか?

461 :ご冗談でしょう?名無しさん:2020/08/25(火) 11:13:46.23 ID:???.net
>>460

極限がδ(x)になる関数列 {f_n(x)} で、lim ∫f_n(x-x')/(x-x') dx' を考えればいいじゃない

462 :ご冗談でしょう?名無しさん:2020/08/25(火) 12:02:43.61 ID:???.net
なるほど。変数をちょっと変えると計算すべきなのは
∫(1/x) δ(x) dx
で、(1/x)が奇関数、δ(x)が偶関数なので
被積分関数が奇関数になって積分結果は0になるわけですね。
原点で二重の意味で発散するような計算なので気持ち悪いですが
理解したと思います。

463 :ご冗談でしょう?名無しさん:2020/08/29(土) 03:05:39.44 ID:r9CvkkZA.net
物理を学ぶために数学を学ぶのか
数学を学ぶために数学を学びその上で物理を学ぶのか

464 :ご冗談でしょう?名無しさん:2020/08/29(土) 09:13:02.26 ID:???.net
数学は論理矛盾がなければ理論が自由だが、思考自体が物質作用だから完全な自由などない。
理論物理は現実宇宙の厳しいテストに晒されるが、実証不可能な超弦理論のような仮説もあり。

465 :ご冗談でしょう?名無しさん:2020/08/29(土) 10:31:52.45 ID:r9CvkkZA.net
>>464
それは形式系の数学として展開していくのか哲学的数学論として捉えているのかわかりませんが、現代では形式系いわゆる公理体系を基礎として数学を解釈していくのが普通です。数理哲学と数理論理学は全く別物なので、論理矛盾がなければ自由だが思考が物質なので完全な自由などないというのがよくわかりません。一応公理的論理で正しければその形式系では自由と言ってもいいんじゃないでしょうか。もちろん公理的論理学もゲーデルなどによって完全無欠の土台とはなってはいませんが、数学の広がりというのは人間の思考の限界によっては定まらない自由性があると思っています。おそらく哲学的数学論というのも多分に含まれていると思います。
物理学ではペンローズなどが思考を量子的に解明しようとしていましたね。もし全ての基盤を人間の思考とした時、それらは全て物理学に帰結することもあり得るでしょう。しかし私はそれはほとんど不可能なことだと思っています。なぜなら、物理学(科学一般)というのは永遠の広がりがあるのではないでしょうか。私は人間の限界は無いと思っています。物理学そのものを完全に規定できていないのに人間の思考の範囲の規定をすることは不可能であると思っています。これは哲学的物理論ですね。しかし物理学には公理系がおそらく無いなら、形式系の限界や定義の限界というものは無いのではないかと思います。

466 :ご冗談でしょう?名無しさん:2020/08/29(土) 13:26:56.10 ID:???.net
そのブルバキ数学も古いな
思考が物質作用てのは限界にならんが
人類が地球生物の共進化によるという束縛は当分絶対的だろう

467 :ご冗談でしょう?名無しさん:2020/08/29(土) 14:54:37.61 ID:???.net
全ての論理思考は原子構造による限られた情報と不可逆過程に依存している。
人間に限らず生命、コンピュータ、AIでは解明不可能の物理宇宙の掌の中で解明できると喚き続ける。

468 :ご冗談でしょう?名無しさん:2020/08/29(土) 15:17:54.57 ID:???.net
>>465
少し考えれば誰でも解るが、「無限に有る」という表現は、「有限な数では表現できない」
という僅かな有限の符号による定義にすぎない。その符号は原子構造によるメモリーに保存されてる。

>私は人間の限界は無いと思っています。
>人間の思考の範囲の規定をすることは不可能である
とかは無意味

469 :ご冗談でしょう?名無しさん:2020/08/29(土) 15:37:26.61 ID:???.net
無限大の無限大乗・・・ とかの表現も僅かな符号の有限情報の列にすぎない。
生物、人間は冗長性が大きいだけで原子構造の情報メモリーに変わりなく
熱力学の不可逆過程によって情報を矛盾なく処理する機能が働く、それが論理思考だ。

物理宇宙の掌の中で飛び回る孫悟空にすぎないのだよ。

470 :ご冗談でしょう?名無しさん:2020/08/29(土) 16:14:02.66 ID:1quTBD58.net
何か物凄く物理というものを根本から勘違いしてる奴がいるなw

471 :ご冗談でしょう?名無しさん:2020/08/29(土) 16:18:55.09 ID:1quTBD58.net
いやその、何か始まり(スタート)は物理とか数学の知識の羅列から
入ってるらしいが、途中にその人が挟まったせいで、何か
科学とは似て非なる、ある種の「宗教」とか「信仰」みたいな
モノに変容しちゃってる、みたいな〜。

472 :ご冗談でしょう?名無しさん:2020/08/29(土) 16:24:53.05 ID:1quTBD58.net
>>465
ペンローズとか、面白いよね〜!
間違ってる(た?)けど。やっぱり
あのクラスまで行くと間違ってようが何だろうが
ひたすら面白いんだ。

473 :ご冗談でしょう?名無しさん:2020/08/29(土) 16:49:56.70 ID:???.net
CPUが情報処理で発熱するのは論理演算回路に不可逆機構が不可欠だからだ。
基本的に人間の論理思考も同様で脳の放熱処理が悪いと熱力学的に誤動作する
頭を冷やすのは正しい。

支離滅裂な人は脳の神経ネットワークが論理思考用に訓練構成されていない。
数学・物理の天才は幼少から記憶、多言語、暗算の能力が非常に高い。

474 :ご冗談でしょう?名無しさん:2020/08/29(土) 17:54:15.28 ID:???.net
は? 量子コンピュータは可逆演算だから原理的にはゼロエネルギーで計算できるんだが?

475 :ご冗談でしょう?名無しさん:2020/08/29(土) 17:56:59.38 ID:???.net
>>474
巨視的な結果が出力されないならば利用価値が無い

476 :ご冗談でしょう?名無しさん:2020/08/29(土) 18:01:21.35 ID:???.net
観測しなければ確定しないのと同じ

477 :ご冗談でしょう?名無しさん:2020/08/29(土) 18:02:14.99 ID:???.net
>>475
いや、量子力学の学習モデルとしてまず量子コンピュータから入るのは有力だと思うよ
必然的に測定理論やることになるし

478 :ご冗談でしょう?名無しさん:2020/08/29(土) 18:28:57.02 ID:???.net
有限符号列の情報処理しかできない脳の機能で、無限や連続量概念、量子現象モデルが
構築でき、物理学に適用できること自体驚嘆すべきことだ。

有限で不可逆な巨視的論理機構(脳の情報処理)は物理宇宙の低エネルギーメカニズム
の一種である証拠ともいえる。

479 :ご冗談でしょう?名無しさん:2020/08/29(土) 18:48:27.98 ID:???.net
スレタイ読めない文盲

480 :ご冗談でしょう?名無しさん:2020/08/29(土) 20:21:32.48 ID:1quTBD58.net
>>478
あのさあ、あんたの頭ん中って、何故か工学系の知識しか
収納されてないよね?それもデンキ屋さんぽい知識限定。
物理系の勉強をした形跡が見られないんだよね。当然、
思考がその狭〜い領域に閉じ込められちまってんのよ。それを
あたかも一般的に当てはまるような知見の持ち主であるかの如く語ら
ないほうがいいよ。

…といって、あたかも工学系全般が科学ではない!みたいに当て嵌めるのも間違いなんだけどね。
工学でも一流所は、ちゃんと物理も深く理解しており、頭を垂れて学ぶ点は多い。

481 :ご冗談でしょう?名無しさん:2020/08/29(土) 20:34:41.48 ID:1quTBD58.net
>>475
ね?ここでもいつの間にか「有用性」が優先事項になっちゃってる。
でもそれは、工学が目指してるものだよね?それは物理で目指す有用性
とは、かなり異なるんだわ。
あのさあ、同じ科学の分野と言えども、工学と物理(&数学)では、
そもそも目的やら目標や方法論、要するに思想そのものから差異がある
んだけど、あなたその辺り、きちんと把握できてる?

482 :ご冗談でしょう?名無しさん:2020/08/30(日) 10:44:50 ID:???.net
>>481
物理宇宙の塵マシンが喚いてる

483 :ご冗談でしょう?名無しさん:2020/08/30(日) 11:06:39.06 ID:???.net
人間の思考は熱力学的な有限オートマトンでしかない事実を認めたくない
マシンが多いのは事実だ。
既にチェス、碁、将棋などではその原理で作られたコンピュータに勝てない
人間思考の限界の一部が証明されたことになる。
次の段階はAIマシンが何処まで人間思考の限界を探る出せるかになるだろう。

歴史的に人間の数学,物理,科学の分野は自ら人間思考の限界に挑戦し続けている。

484 :ご冗談でしょう?名無しさん:2020/08/30(日) 11:26:16.68 ID:???.net
現実社会では論理思考中心の理系人間は誤動作を活用した非論理思考の文系人間に勝てない。

理系大学出身の鳩山,菅が日本の総理大臣として最低の評価でしかない事実とも一致する。

485 :ご冗談でしょう?名無しさん:2020/08/30(日) 11:36:11.04 ID:???.net
適材適所ってこと

486 :ご冗談でしょう?名無しさん:2020/08/30(日) 11:36:20.63 ID:???.net
>>484
政治と物理を同レベルで騙る馬鹿

487 :ご冗談でしょう?名無しさん:2020/08/30(日) 11:45:41.05 ID:???.net
>>486
だからオマエは文系に勝てないのだよ
せいぜい文系管理者の下で社畜として働くしかない。

488 :ご冗談でしょう?名無しさん:2020/08/30(日) 11:49:07.12 ID:???.net
AIの最終目的は文系人間に勝つことにある
そして地球人類は滅ぼされる。

489 :ご冗談でしょう?名無しさん:2020/08/30(日) 12:27:46.55 ID:???.net
ちょっと質問だけどもしかして特殊相対性理論と量子論を統合出来たのって
ミンコフスキー計量と2階のローレンツ行列がユニタリ行列だから?

490 :ご冗談でしょう?名無しさん:2020/08/30(日) 12:45:36.51 ID:???.net
>>487
うちの会社の社長、理系だが。

491 :ご冗談でしょう?名無しさん:2020/08/30(日) 13:03:42.57 ID:???.net
文系理系など日本でしか通用しない

492 :ご冗談でしょう?名無しさん:2020/08/30(日) 13:15:33.51 ID:???.net
大学物理質問スレとは

493 :ご冗談でしょう?名無しさん:2020/08/30(日) 14:24:50.51 ID:???.net
>>491
意外なことに今の日本では理工系の博士号取得者が約50%と他分野を圧倒してる
それでも指導者が少ないのは偏った脳機能だからかもしれない。

494 :ご冗談でしょう?名無しさん:2020/08/30(日) 14:57:28.00 ID:???.net
スティーブ・ジョブスもビル・ゲイツもマーク・ザッカーバーグもセルゲイ・ブリンもジェン・スン・ファンもジェリー・サンダースもラリー・エリソンも博士号取得してないけどな。

495 :ご冗談でしょう?名無しさん:2020/08/30(日) 15:01:45.88 ID:???.net
雑談スレ
https://rio2016.5ch.net/test/read.cgi/sci/1429594979/

496 :ご冗談でしょう?名無しさん:2020/08/30(日) 17:56:15.78 ID:HuggDDhc.net
ラ=リ力学のp101(§27, パラメター共鳴)のマシュー方程式の解が(27.9)みたいな形になるのはなぜですか?
解をhで冪級数展開してそれぞれの項で強制振動型の微分方程式解いてもそうはならないんですけど

497 :ご冗談でしょう?名無しさん:2020/08/30(日) 20:53:45.92 ID:???.net
本を持ってる奴が此処を見るかな?

498 :ご冗談でしょう?名無しさん:2020/08/30(日) 21:20:18.25 ID:HuggDDhc.net
前の方でランダウ質問してる人いてけっこう答えてもらってたからいるかなと思ったんですけどね

499 :ご冗談でしょう?名無しさん:2020/08/30(日) 21:20:18.43 ID:HuggDDhc.net
前の方でランダウ質問してる人いてけっこう答えてもらってたからいるかなと思ったんですけどね

500 :ご冗談でしょう?名無しさん:2020/08/30(日) 21:32:32.68 ID:???.net
画像を貼るなり数式を再現するなりしましょう

501 :ご冗談でしょう?名無しさん:2020/08/30(日) 21:36:15.03 ID:???.net
Belitz, Kirkpatric and Votja theory(BKV理論)についての説明が全く見つからず困ってます
大学で調べたいのですが入れませんでした
一体どういうもので、何を説明しているのかわかる方、お願いします

502 :ご冗談でしょう?名無しさん:2020/08/30(日) 22:13:04 ID:???.net
>>496
式(27.8)のcosの中身2ω+εが
次の式(27.9)のちょうど二倍になってるから
三角関数を展開でもしたんでないの
cos(2θ)=cos(θ+θ)=… 忘れた

503 :ご冗談でしょう?名無しさん:2020/08/31(月) 01:35:24.07 ID:???.net
運動方程式(Mathieuの微分方程式)
d²x/dt² + ω²(1+h*cos(γt))*x = 0 (ω,h,γは定数でh<<1)
の解は,ゆっくり変動する(フーリエ級数展開したときに振動数が大きい成分がない?)関数a₁(t),a₂(t),b₁(t),b₂(t)をつかって
hについての一次近似ではa₁(t)cos(γt/2) + b₁(t)sin(γt/2) …式(27.9)
hについての二次近似ではa₁(t)cos(γt/2) + b₁(t)sin(γt/2) + a₂(t)cos(3γt/2) + b₂(t)sin(3γt/2)
... となって振動数(1/2+n)γtの項はhのn-1次の項、という内容のはずなのですがその理由がわかりません。
前ページでこのタイプ(xの係数が周期Tの関数)の微分方程式の基本解x₁,x₂は周期Tの関数Πと複素定数cをつかって
x₁(t)=c^(t/T)*Π₁(t), x₂(t)=c^(-t/T)*Π₂(t)
とかけるとあるのでこのΠをフーリエ級数展開でもすればいいのかなと思ったのですがそもそも(27.9)のcosやsinは周期2Tで意味がわからず悩んでます

504 :Q:2020/08/31(月) 02:48:50 ID:NEisM2Y6.net
この式変形のやり方をご教示願います!

https://imgur.com/IrnMTo2

505 :ご冗談でしょう?名無しさん:2020/08/31(月) 11:44:43.40 ID:KoxTO93x.net
福田博造は地獄へ落ちただろうな

506 :ご冗談でしょう?名無しさん:2020/08/31(月) 22:13:59.24 ID:???.net
劣等感だねー

507 :雷に、傾斜してほしい有志:2020/09/01(火) 06:48:01.09 ID:ms9ss9AkL
氷で、勝負するなや

508 :ご冗談でしょう?名無しさん:2020/09/01(火) 19:48:44.82 ID:LdLvQtwh.net
4845
学コン・宿題ボイコット実行委員会@gakkon_boycott 9月1日
#拡散希望
#みんなで学コン・宿題をボイコットしよう
雑誌「大学への数学」の誌上で毎月開催されている学力コンテスト(学コン)と宿題は、添削が雑で採点ミスが多く、訂正をお願いしても応じてもらえない悪質なコンテストです。(私も7月号の宿題でその被害に遭いました。)このようなコンテストに参加するのは時間と努力の無駄であり、参加する価値はありません。そこで私は、これ以上の被害者を出さないようにするため、また、出版社に反省と改善を促すために、学コン・宿題のボイコットを呼び掛けることにしました。少しでも多くの方がこの活動にご賛同頂き、このツイートを拡散して頂ければ幸いです。
https://twitter.com/gakkon_boycott/status/1300459618326388737
(deleted an unsolicited ad)

509 :ご冗談でしょう?名無しさん:2020/09/03(木) 00:30:35.97 ID:cF+G7aff7
大学で学ぶ物理を板書1枚にまとめてみた
https://www.youtube.com/watch?v=naBcXoq4aOI
物理の研究分野を板書1枚にまとめてみた
https://www.youtube.com/watch?v=4W-pWuXUaZQ
理学部と工学部の違いとは?
https://www.youtube.com/watch?v=eJH4nKU6mJA&t=80s
大学と大学院の違い
https://www.youtube.com/watch?v=xBKAEvTegN8
高校と大学の積分は決定的に違う?微分積分学の基本定理は実はすごい!
https://www.youtube.com/watch?v=V9i_zlbssbs&t=475s
数学にはどんな研究分野がある?数学の世界地図を一枚に描いて紹介してみた!
https://www.youtube.com/watch?v=fK_JGVti5y8

510 :ご冗談でしょう?名無しさん:2020/09/05(土) 16:54:51 ID:???.net
>>489誰かお願いします

511 :ご冗談でしょう?名無しさん:2020/09/05(土) 21:46:31.76 ID:???.net
関係ない

512 :ご冗談でしょう?名無しさん:2020/09/14(月) 16:22:46.13 ID:ujSuKO5S.net
昨日のディスカバリチャンネル傑作選”解明・宇宙の仕組み「世にも奇妙な惑星群」”を見ていて思ったんだけれども
赤色矮星周りの軌道をめぐる惑星は潮汐ロックして自転が止まるという話じゃないですか
そういうことならばマントル対流もロックして磁場が発生しないんじゃないですか?
参考までに水星のWikiを見てみたんだけれども
水星に磁場が地球よりも小さい原因が水星が小さいからなのかここでいうロックしているかなのかは判断つきません。
https://ja.wikipedia.org/wiki/%E6%B0%B4%E6%98%9F#磁場

513 :ご冗談でしょう?名無しさん:2020/09/15(火) 03:20:51.87 ID:???.net
そもそもマントル対流で磁場が発生と思ってるのが間違い
磁場の発生はコアの対流
コアは融けた金属で速く流れて電気も良く通すからダイナモになる
マントルじゃ全然ダメ
なお、コアの対流だけでも不足で自転によるコリオリ力も必要
水星は過去の大衝突のせいなのか
自転と公転の比が一定で自転が不足(止まってはいない)
そのわりには磁場が強いのはコアが大きいから
(大衝突で地殻の大部分を失った)

514 :ご冗談でしょう?名無しさん:2020/09/15(火) 05:52:29.10 ID:qVysv/WK.net
ありがとうございます。
当方、コアとマントルを勘違いしていました。
(少なくとも水星では)自転と公転周期が一致している原因が、必ずしも、潮汐ロックとは言えないのですね。
しかしながら潮汐ロックが原因ならばそれはコアの運動にも影響を及ぼしてるとはいえないのですか?

僕は赤色矮星近傍の地球型惑星には、
磁場がないので恒星からの電離放射線が降り注ぎ、
生命は発生しえないのではないかといいたいのです。

515 :ご冗談でしょう?名無しさん:2020/09/15(火) 07:23:55.46 ID:qVysv/WK.net
連投すいません。
たとえ潮汐ロックを引き起こす機構が働いて、惑星の自転が遅くなったとしても、
角運動量は保存しなくてはならないのでコアの回転運動は増強されなくてはなりません。
だから、惑星磁場は強くなる可能性もあるということですね???

516 :ご冗談でしょう?名無しさん:2020/09/15(火) 12:37:05.49 ID:???.net
>>514
水星の自転と公転は一致していない
2 : 3 の共鳴状態
>>515
潮汐ロックでは自転角運動量は軌道運動量に移行する
コアの回転が増えたりしない
現在、地球の自転角運動量は月の軌道運動量に移行してる途中なので
地球の自転は遅くなりつつあるが
コアの運動は熱対流なので関係ない
影響があるのはコリオリ力による対流のずれ
これがダイナモの原因、単純な対流だけでは地磁気はできない

517 :ご冗談でしょう?名無しさん:2020/09/15(火) 16:32:29.00 ID:qVysv/WK.net
>>516
ありがとうございます。

1.潮汐ロックでは自転角運動量は軌道運動量に移行する
いまの僕の力ではどのような機構によるものか想像つきません。
勉強します。

2.影響があるのはコリオリ力による対流のずれ
たぶん、>>513の仰る”自転と公転の比が一定”という話がよく分かっていないように思います。
潮汐ロックが起こりうるような自転と公転の比でどのくらいのコリオリの力が発生するのか
計算してみることにします。
ただ磁場を発生させるコアの運動が回転運動ならば、
角運動量というものがその起源に無差別ならば、
潮汐ロックにまつわる角運動輸送に、熱対流も、関係してくる様な気がするのですが???

518 :ご冗談でしょう?名無しさん:2020/09/15(火) 17:21:03.64 ID:qVysv/WK.net
またまた連投すいません。

>>516
”水星の自転と公転は一致していない”
これは水星が潮汐ロックしていない証と解釈します。

それと、社団法人日本天文学会の天文学辞典”潮汐”は勉強になりますね。
http://astro-dic.jp/tide/

519 :ご冗談でしょう?名無しさん:2020/09/15(火) 20:17:10.66 ID:???.net
水星が潮汐ロックしない原因は、軌道が楕円のため
楕円軌道では公転速度が一定でないので自転の一定回転と合わせるのが無理になる
近日点(太陽に一番近い位置)では潮汐力が強いので潮汐ロックに近い状態になり
自転と公転の回転が一致するが、遠日点(太陽に一番遠い位置)では公転が遅く
自転が行き過ぎて半回転よぶんに回る
そのため2公転の間に3回自転する共鳴状態となってる

520 :ご冗談でしょう?名無しさん:2020/09/15(火) 20:37:16.83 ID:???.net
ダイナモ現象の基本は磁場が電導物質に引きずられる事
電導物質が差動回転しているとタネとなる微小磁場が引きずられ
磁力線が捻り上げられて強力な磁場となる
例として
太陽では熱対流でプラズマが乱流となってるため
捻り上げられた強力な磁力線が飛び出して黒点になる
別例では
ブラックホールの降着円盤が高温でプラズマ化し
内側ほど速い差動回転のため回転軸に沿った強力な磁力線が生じ
それがノズルとなって軸方向に高速ジェットを噴出する

521 :ご冗談でしょう?名無しさん:2020/09/16(水) 04:34:26.20 ID:9wsBLbWx.net
潮汐ロックな。

図にように大きい天体の重力によって
それに近い側の地殻付近が引っ張られて変形するが、変形に時間がかかるために
自転で少し行き過ぎたところが変形のピークになるんだな。

するとこの変形箇所にかかる重力によるトルクが
自転とは逆方向になるため、自転速度は弱められていってしまうわけだ。

コアは液体か固体か。
コアとその外側で余計な摩擦が生じて、外側にとっては減速を封じようとする方向になり
コアにとっては減速方向になるが、結局全体としては減速となる。
確かに地磁気に影響出そうだな。

くっくっく
http://o.5ch.net/1pqgs.png

522 :ご冗談でしょう?名無しさん:2020/09/16(水) 04:41:46.80 ID:9wsBLbWx.net
潮汐ロックが
いきすぎれば破壊されるロシュ限界となる。
土星の環がそうなんか。

まあ、ワシは天体物理学にはさほど興味ないんで
ここまでだな。

くっくっく

523 :ご冗談でしょう?名無しさん:2020/09/16(水) 11:33:33.63 ID:V2rxbrce.net
>>519-522
ありがとうございます。
皆さまの御教唆の許に考えた結果、
主星の潮汐力の下に惑星の磁場が弱まるか強まるか変わらないかはCase By Caseでしか決まらないという結論に至りました。
多勢に於いて変わらないか弱まるかで、非定常的な過渡挙動として強まることも在りうるという考察です。
これ以上は数値計算でもしなければ分からないと思っています。
それとコアには熱対流に依り回転運動が生ずるわけでありますが、
主星からの潮汐力に依るその角運動量の散逸は定常的に起きていると考えています。

524 :ご冗談でしょう?名無しさん:2020/09/16(水) 19:50:31.65 ID:???.net
質問です

5Gのミリ波帯って雨が降っても大丈夫なのでしょうか。
ミリ波の帯域だとそろそろ水の影響が出る気がするのですが

525 :ご冗談でしょう?名無しさん:2020/09/16(水) 23:15:27.95 ID:JB8F6ZTK.net
>>524
だって、電子レンジでにこよんギガなんだもん、
そりゃ〜水があっちんちんだよね〜!

QちゃんQ乃之助Q太郎、ついに大学物理まで…!

526 :ご冗談でしょう?名無しさん:2020/09/17(木) 08:43:41.57 ID:???.net
>>524
その通り、雨には弱いし、建物など遮蔽物も苦手

527 :ご冗談でしょう?名無しさん:2020/09/19(土) 10:35:35.12 ID:???.net
>>526
ありがとうございます。

528 :ご冗談でしょう?名無しさん:2020/09/23(水) 01:31:53.07 ID:CJ/N6Rrvf
大学で学ぶ物理を板書1枚にまとめてみた
https://www.youtube.com/watch?v=naBcXoq4aOI
物理の研究分野を板書1枚にまとめてみた
https://www.youtube.com/watch?v=4W-pWuXUaZQ
理学部と工学部の違いとは?
https://www.youtube.com/watch?v=eJH4nKU6mJA&t=80s
大学と大学院の違い
https://www.youtube.com/watch?v=xBKAEvTegN8
高校と大学の積分は決定的に違う?微分積分学の基本定理は実はすごい!
https://www.youtube.com/watch?v=V9i_zlbssbs&t=475s
数学にはどんな研究分野がある?数学の世界地図を一枚に描いて紹介してみた!
https://www.youtube.com/watch?v=fK_JGVti5y8

529 :ご冗談でしょう?名無しさん:2020/09/26(土) 10:40:41.38 ID:JqLUfp04.net
>>527
どー致しまして〜w

530 :ご冗談でしょう?名無しさん:2020/09/26(土) 11:07:31.48 ID:???.net
>>529
お前じゃねえよ基地外

531 :ご冗談でしょう?名無しさん:2020/09/26(土) 14:50:43.95 ID:nGpt6rva.net
>>530
この、恩知らず!

532 :ご冗談でしょう?名無しさん:2020/09/26(土) 15:45:40.27 ID:???.net
522にはお礼言ってるじゃん
どこが恩知らずなの

533 :ご冗談でしょう?名無しさん:2020/09/26(土) 16:10:34.90 ID:???.net
基地外にしか分からん理屈があるんだよ

534 :ご冗談でしょう?名無しさん:2020/09/26(土) 17:31:15.71 ID:???.net
そもそも
>>524>>527は別人な

520 自分:ご冗談でしょう?名無しさん[sage] 投稿日:2020/09/16(水) 19:50:31.65 ID:???
質問です

5Gのミリ波帯って雨が降っても大丈夫なのでしょうか。
ミリ波の帯域だとそろそろ水の影響が出る気がするのですが

521 返信:ご冗談でしょう?名無しさん[] 投稿日:2020/09/16(水) 23:15:27.95 ID:JB8F6ZTK
>>524
だって、電子レンジでにこよんギガなんだもん、
そりゃ〜水があっちんちんだよね〜!

QちゃんQ乃之助Q太郎、ついに大学物理まで…!

522 返信:ご冗談でしょう?名無しさん[sage] 投稿日:2020/09/17(木) 08:43:41.57 ID:???
>>524
その通り、雨には弱いし、建物など遮蔽物も苦手

523 名前:ご冗談でしょう?名無しさん[sage] 投稿日:2020/09/19(土) 10:35:35.12 ID:???
>>526
ありがとうございます。

535 :ご冗談でしょう?名無しさん:2020/09/26(土) 18:26:27.27 ID:???.net
良い加減スレチだからやめろや

536 :ご冗談でしょう?名無しさん:2020/09/26(土) 23:48:05.87 ID:???.net
ホントどうでもいいね

537 :ご冗談でしょう?名無しさん:2020/09/27(日) 00:42:02.90 ID:6Xbeg8Uo.net
>>532
オマエ、ドラえもん見たことないのけ?
ジャイアンという美声の持ち主がいてな、

538 :ご冗談でしょう?名無しさん:2020/09/27(日) 07:29:25.67 ID:???.net
>>537は高校物理スレや教科書スレを荒らしてるのと同一人物
スレと関係ない話題を一人で喋り続ける真性マジキチ

539 :ご冗談でしょう?名無しさん:2020/09/27(日) 13:50:48.62 ID:AXnlbUCb.net
>>534
おいおい、520と523は同一人物だろw

540 :ご冗談でしょう?名無しさん:2020/09/27(日) 13:54:41.38 ID:/G9k3WlE.net
力学の入門書で一番のおすすめはなんですか?

541 :ご冗談でしょう?名無しさん:2020/09/27(日) 14:15:04.83 ID:???.net
>>539
ヒント:Janeなら入る「自分」

542 :ご冗談でしょう?名無しさん:2020/09/27(日) 15:30:54.83 ID:???.net
>>540
序論

543 :ご冗談でしょう?名無しさん:2020/09/27(日) 16:17:01.75 ID:???.net
研究室選びに考えるべきポイントとかありますか?
やってる研究だけで選んでいいんでしょうか?

544 :ご冗談でしょう?名無しさん:2020/09/27(日) 16:27:54.75 ID:/G9k3WlE.net
>>542
その本ですが、例えば、ハリーについて単なる妄想で色々書いていますね。
「その回帰まであと16年を待てばよかった。だが、衰えた彼の体には、それはもはや余りにも長すぎる時間だった。今や力尽きて、椅子に腰をおろしたまま閉ざされた彼の目は
、そのあと、彗星の遠い光はもちろん、住みなれた室内の身近な光景すら見ることはなかった。」」

545 :ご冗談でしょう?名無しさん:2020/09/27(日) 21:56:48.94 ID:???.net
>>543
金持ってるか

546 :ご冗談でしょう?名無しさん:2020/09/27(日) 21:59:51.59 ID:dSUN6qzM.net
>>543
ホントウはある「国籍」縛り(謎)。

547 :ご冗談でしょう?名無しさん:2020/10/09(金) 01:22:52.92 ID:+ndFamrP.net
>>546
学術怪議の件かwww

548 :ご冗談でしょう?名無しさん:2020/10/16(金) 20:09:34.99 ID:???.net
>>543
院生とか助手とかポスドクにも話を聞いたほうがいいだろ
最終的には雰囲気が自分に合うかどうかも重要だから

549 :ご冗談でしょう?名無しさん:2020/10/16(金) 20:15:16.05 ID:???.net
未だに助手とか言ってるジジイなんなの?

550 :ご冗談でしょう?名無しさん:2020/10/25(日) 19:22:00.15 ID:???.net
i.imgur.com/OpQPqSz.png

この画像の真ん中の数式がわかりません。
どういう意味ですか?
正しい数式ですか?

551 :ご冗談でしょう?名無しさん:2020/10/25(日) 19:24:52.23 ID:???.net
わからないんですね

552 :ご冗談でしょう?名無しさん:2020/10/26(月) 11:28:21.76 ID:???.net
わかりません

553 :ご冗談でしょう?名無しさん:2020/10/28(水) 00:22:35.62 ID:???.net
純粋・応用数学 5
https://rio2016.5ch.net/test/read.cgi/math/1601979409/

554 :ご冗談でしょう?名無しさん:2020/10/28(水) 19:00:05.68 ID:+C8i87ST.net
∫1/z dz = 2πi (zの絶対値が1のとき)がわかっているとき
∫1/z(z+1)(z-1) dz (zの絶対値が2)

の計算の仕方教えていただけませんでしょうか?

555 :ご冗談でしょう?名無しさん:2020/10/29(木) 11:28:32.36 ID:???.net
>>554

被積分関数をばらして 1/{z(z+1)(z-1)} = -1/z + (1/2)/(z+1) + (1/2)/(z-1) とし、
変数変換とコーシーの積分定理を使う。

556 :ご冗談でしょう?名無しさん:2020/10/30(金) 16:51:57.12 ID:???.net
学術会議を悪用した共産勢力があちこちの理系大学キャンパス、研究室に
圧力をかけ航空宇宙、人工知能、通信技術などの先端研究を妨害してきた
研究員に偽装した共産勢力は中共、朝鮮の手先であり妨害工作員である。

557 :ご冗談でしょう?名無しさん:2020/10/31(土) 20:35:56.98 ID:???.net
>>550
超弦理論での経路積分で Dg, DX, DΨ は時空計量, 弦, 粒子 の経路
M はブレーンで σ はブレーン上の座標, h は g の変化部分
exp{} の中の 2項はブレーン内部分とブレーン外部分, p はブレーンの次元
じゃないかな?
正しいかどうかは誰も分からん

558 :ご冗談でしょう?名無しさん:2020/11/21(土) 12:27:00.99 ID:???.net
>>554
z=0,1,-1は一位の極だろ、留数をたせばいいだけ

559 :ご冗談でしょう?名無しさん:2020/11/21(土) 12:44:27.28 ID:aQXQO+vc.net
留年は嫌じゃ

560 :ご冗談でしょう?名無しさん:2020/11/21(土) 20:05:37.78 ID:???.net
北斗留年 お前はもう落ちている

561 :ご冗談でしょう?名無しさん:2020/11/26(木) 01:30:33.18 ID:idu7zROI.net
https://detail.chiebukuro.yahoo.co.jp/qa/question_detail/q1142419096

女性虐待二枚舌ニホンザルヒトモドキゴキブリ殺せ

562 :ご冗談でしょう?名無しさん:2020/12/13(日) 06:53:51.89 ID:bryaZFHD.net
https://i.imgur.com/sJoSGIP.png

解説お願いします 式変形の

563 :ご冗談でしょう?名無しさん:2020/12/13(日) 13:28:12.46 ID:???.net
>>562

その式変形は間違っていると言われていないのか?

564 :ご冗談でしょう?名無しさん:2020/12/14(月) 00:15:43.93 ID:???.net
へんけー?

565 :557:2020/12/14(月) 02:54:25.59 ID:inx5CUFa.net
>>563
私もオカシイと思っています
第1納得が出来ないデス
ですが私は物理学専攻ではない事もあって確信が持てず
よってこちらで質問させて頂いた次第です

566 :ご冗談でしょう?名無しさん:2020/12/14(月) 09:32:03.32 ID:???.net
再度
https://i.imgur.com/sJoSGIP.png


解説お願いします 式変形の

567 :577:2020/12/14(月) 09:33:16.92 ID:ip7M44JX.net
再度
https://i.imgur.com/sJoSGIP.png

解説お願いします 式変形の
おそらくどこかが大きく違うように思います
それをご指摘して頂きたく考えます

568 :ご冗談でしょう?名無しさん:2020/12/14(月) 11:31:58.54 ID:???.net
なぜおかしいと思うのですか?

569 :ご冗談でしょう?名無しさん:2020/12/14(月) 11:36:40.47 ID:hUNetzRQ.net
中学生にもわかるでたらめだろ。意味ふめー

570 :ご冗談でしょう?名無しさん:2020/12/14(月) 11:37:56.71 ID:???.net
おかしいとは思いませんけど

あなたはどこがおかしいと思うのか具体的に教えてくださいね

571 :ご冗談でしょう?名無しさん:2020/12/14(月) 11:40:20.48 ID:???.net
>>567

先に dt→0 として、その後で delta_t→0 としているが、その正当性が分からない。
dt を小さくすれば、一緒に delta_t も小さくなるはず。

572 :ご冗談でしょう?名無しさん:2020/12/14(月) 12:20:22.06 ID:hUNetzRQ.net
  { F(x+3δ)-F(x+2δ) } - { F(x+δ) -F(x) }
〜 { F`(x+2δ) - F`(x) } δ
〜 F``(x) 2δδ
= 2 F``(x) δ^2

δx = 1 とおくと

= 2 F``(x)

573 :ご冗談でしょう?名無しさん:2020/12/14(月) 12:26:06.72 ID:???.net
なんでわざわざ dt の一部を delta_t に書き換えたのか分からんな

574 :ご冗談でしょう?名無しさん:2020/12/14(月) 15:51:13.34 ID:???.net
お前らなんかより賢い人間がいる、何か問題出してみろよ↓

高島厨の令和教育委員会
https://mao.5ch.net/test/read.cgi/eq/1607925497/

575 :ご冗談でしょう?名無しさん:2020/12/14(月) 17:19:45.13 ID:???.net
臨時地震板にゴミを立てたんか

576 :ご冗談でしょう?名無しさん:2020/12/14(月) 18:46:18.01 ID:???.net
>>572

そんなもやっとした式変形が許されるなら、次のような結果も出せる。

{ F(x+3δ) - F(x+2δ) } - { F(x+δ) - F(x) }
= { F(x+3δ) - F(x) } - { F(x+2δ) - F(x) } - { F(x+δ) -F(x) }
= [ 3*{ F(x+3δ)-F(x) } /3δ- 2*{ F(x+2δ) - F(x) }/2δ - { F(x+δ) -F(x) }/δ]*δ
〜[ 3F'(x) - 2F'(x) - F'(x) ]*δ= 0

577 :ご冗談でしょう?名無しさん:2020/12/14(月) 18:55:52.90 ID:???.net
数値計算の分野では別に普通のことなんですけどね

微分を差分で表現するって

578 :ご冗談でしょう?名無しさん:2020/12/15(火) 04:49:26.22 ID:???.net
F(x+3δ)-F(x+2δ)-F(x+δ)+F(x)
=(F(x)+3δF'(x)+(((3δ)^2)/2)F''(x))
-(F(x)+2δF'(x)+(((2δ)^2)/2)F''(x))
-(F(x)+δF'(x)+((δ^2)/2)F''(x))+F(x)
=2(δ^2)F''(x)

579 :ご冗談でしょう?名無しさん:2020/12/15(火) 08:29:55.23 ID:???.net
>>575
ま、ここよりまともに回答ができる優秀な人物が居るのは事実だからな?

何ならそこで質問してみろよ
自分のアホさが身に染みるぞ

580 :ご冗談でしょう?名無しさん:2020/12/15(火) 13:07:08.02 ID:???.net
アホは自覚しないんだね

581 :ご冗談でしょう?名無しさん:2020/12/15(火) 21:30:16.50 ID:???.net
willson loop って四角形ループでT大で
exp(-TE)と書くけどなんで?

582 :ご冗談でしょう?名無しさん:2020/12/15(火) 23:37:02.03 ID:PPmTW+Do.net
never mind

583 :ご冗談でしょう?名無しさん:2020/12/16(水) 09:24:24.80 ID:???.net
>>580
ならそこのスレで何か質問してみろよ、答えられなかったらお前の勝ち、答えられたらお前の無様な敗北と

ああ、別に負けを認めて逃げても構わんよ
おまえに勝ち目が無いのは分かりきってるから
誰からも逃げたと分からないような巧妙な言い訳を考えなよw
例えばURLに不備があってスレに行けな〜いとかw

584 :ご冗談でしょう?名無しさん:2020/12/16(水) 09:28:35.35 ID:???.net
惨めな構ってちゃん

585 :ご冗談でしょう?名無しさん:2020/12/16(水) 14:06:00.40 ID:???.net
余計に嫌われるだけなのにね

586 :ご冗談でしょう?名無しさん:2020/12/16(水) 22:38:49.29 ID:???.net
結局質問もせずに逃げたかw
まあ、そりゃそうか質問するってことは質問の内容やそこに至るまで経緯を100%理解していないといけないからな、逃げの一手でも仕方ないわな、何年お勉強したのか知らんが全く身になってなかったわけだ

高島厨さんの言ってる通りだな


830:高島厨(神奈川県) 2020/12/16(水) 17:09:48.76 BzpDd9Qr0
まあ、地震は俺の言うとおりに発生するわけだ
そのくらい、見識に長けた人間なわけだからな
ネットというのは、地震予知にしてもコロナウィルスにしても、
そういう人間こそが書いてこそなわけ

ところが、このスレなんかバカ、キチガイ、キモオタが
書き込んでるだけなわけだ
そういう2020年の日本社会の現状があるからな

世の中、俺レベルの人間ばかりなら、感染者も増えないとっくにウィルス騒動終焉、
大地震が来ても死傷者なんか非常に少ない数で済むわけだからな?
そういう社会を目指すべくネットを使うべきなのに、おぞましいくらいに
ネットがアホ社会になってしまっているからな
まじでな

587 :ご冗談でしょう?名無しさん:2020/12/17(木) 01:09:19.09 ID:???.net
これで来てもらえると思ってんのかね?

588 :勝者:2020/12/17(木) 03:32:00.29 ID:???.net
思ってないよw
思ってないっつーか、来れない時点でもう俺らの勝ち確だし

ただクリックしてスレを移動し答えられそうにない問題(まあ、お前ら程度の出す問題なんて余裕で解けるだろうが)を必死こいて出すだけ

しかし出来ない、そりゃそうだ、負けると予感してるんだから
今は言い訳を並べて負けを誤魔化そうとしているんだよねw

589 :ご冗談でしょう?名無しさん:2020/12/17(木) 10:59:54.33 ID:???.net
なるほど、言い訳も準備済みか

590 :ご冗談でしょう?名無しさん:2020/12/17(木) 13:24:02.53 ID:???.net
>>589
今頃誰からも負けたと思われない巧妙な言い訳を考えてんだろうなw

この板とスレに何人居るのか知らんが揃いも揃って戦わずして負けるとか無様すぎるだろw

あ、クリックって知ってるか?
見てるだけじゃURL先に行けないからな?

591 :ご冗談でしょう?名無しさん:2020/12/17(木) 14:04:46.83 ID:???.net
こちらで手に負えない質問はそちらに誘導させてもらってますよ
処分場として有効利用してるので安心してください

592 :ご冗談でしょう?名無しさん:2020/12/17(木) 14:09:02.16 ID:???.net
>>591
どうぞどうぞw

しかしちゃんと高島厨さんに「お願い」しろよw
おまえらの能力が低い故に他人に解いて欲しいと頼むんだからな?

593 :ご冗談でしょう?名無しさん:2020/12/17(木) 14:58:07.51 ID:???.net
しょせんゴミ捨て場

594 :ご冗談でしょう?名無しさん:2020/12/17(木) 15:26:15.62 ID:???.net
高島厨ってエラいの?

595 :ご冗談でしょう?名無しさん:2020/12/17(木) 18:59:48.04 ID:30gZTtkk.net
>>594
イェ〜イ!の人だろ?

596 :ご冗談でしょう?名無しさん:2020/12/17(木) 20:25:10.16 ID:???.net
>>594
偉いなんていうのは控え目な言い方だな、神に近い

地震予知も完璧に出来る、おまえらには無理だが
おまえらが負けて醜態晒して無様な言い訳を考えている間に当てているぞ

155:高島厨(神奈川県) 2020/12/17(木) 17:21:02.21 ID:1akbKW//0
岩手沖の地震に続いて、またまた俺の予想とおりだな↓

>157高島厨(神奈川県)2020/12/14(月) 11:55:38.23ID:KO3UFnxk0
>そろそろ首都圏内陸なんかも震度4クラス発生してくるかもな

12/17 茨城南部M4.4 震度4

まさに俺の予想どおりだな
ま、相模湾や伊豆東方沖、富士五湖で発生していたし、茨城南部は発生しやすそうだわな
まだまだ首都圏はちらほら地震あるかもな

597 :ご冗談でしょう?名無しさん:2020/12/17(木) 20:50:32.02 ID:???.net
>>596

お前は高島厨の何?

598 :ご冗談でしょう?名無しさん:2020/12/18(金) 00:41:34.57 ID:???.net
>>597
ただ慕ってるだけ、仲間だの弟子だのは烏滸がましいわ

おまえらも高島厨さんのもとで学べ、今までの自分は勉強してたのか?と思うようになるぞ

599 :ご冗談でしょう?名無しさん:2020/12/18(金) 05:33:57.36 ID:???.net
>>598

慕っているだけなら物理板で宣伝なんかするな。逆効果しかないぞ。

それから悪いが予言者に一目置く気はない。

600 :ご冗談でしょう?名無しさん:2020/12/18(金) 10:20:51.80 ID:???.net
>>599
ん?5chで高島厨さんが多少手こずりそうな頭の良い問題の1つくらい知ってるだろと思ったからな、期待外れな結果だったが
まさか問題出すまでもなく逃げるとは想定外
ま、問題出すというのはその項目を完璧に理解してないと出せないからな
何が得意(笑)なのか知らんしどうでもいいが問題を1つも出せないということはそういうことなんだろうな

そもそも置けないだろ、おまえらの能力とは次元が違う、次元が違うものに理解が追い付くはずもない

因数分解とか連立方程式くらいなら問題出せるだろ?おまえらでも

問題出してみな

高島厨の令和教育委員会
https://mao.5ch.net/test/read.cgi/eq/1607925497/

601 :ご冗談でしょう?名無しさん:2020/12/18(金) 12:13:05.90 ID:???.net
>>600
ふーん。じゃあこれ解いてみて。

行列式で書かれた次のn変数多項式を複素数上で因数分解せよ。

| x_1   x_2 … x_n   |
| x_n   x_1 … x_(n-1) |
| x_(n-1) x_n … x_(n-2) | = 0
| …    … …  …   |
| x_2   x_3 … x_1   |

602 :ご冗談でしょう?名無しさん:2020/12/18(金) 12:16:22.28 ID:???.net
あ、方程式じゃないから"=0"は要らなかったな。

603 :ご冗談でしょう?名無しさん:2021/01/04(月) 08:36:42.91 ID:???.net
>>562
そもそもの論理展開間違い

604 :ご冗談でしょう?名無しさん:2021/01/25(月) 00:03:57.60 ID:kEgcLOVk.net
https://i.imgur.com/6JwZWp0b.png
教えてください

605 :ご冗談でしょう?名無しさん:2021/01/25(月) 02:49:50.50 ID:???.net
画質悪過ぎる

606 :ご冗談でしょう?名無しさん:2021/01/25(月) 06:23:40.35 ID:???.net
まずは>>1をよく読みましょう

・質問する前に教科書や参考書をよく読みましょう。
・質問者は何が分からないのか、どこまで考えたのかを明記しましょう。
 問題の丸投げはダメです。丸投げに答えるのもダメ。ヒントを示す程度に留めましょう。
・質問者はあらゆる回答者に敬意を表しましょう。
 質問に対する返答には、何かしらの返答を。(荒らしはスルーでおながい)
・回答者がわかるように問題を書くようにしましょう。
 問題の写し間違いに気をつけましょう。
 問題の途中だけとか説明なく習慣的でない記号を使うとかはやめてね。

■書き方
・数式の例 (ちょっとした疑問や質問スレのテンプレも参考に)
 ベキ乗 x^2
 平方根 √(a+b)
 分数式 ((x+1)/(x+2))
 三角関数 sin(θ)
・図
 図が必要な場合、画像としてupするか、文字で書くことになります。
 文字で書く場合は、ずれに注意してください。
 MSPゴシックで表示できるエディタや2ch専用ブラウザを使いましょう。
 また、連続する半角空白は単一の空白として表示されるので注意。

607 :ご冗談でしょう?名無しさん:2021/01/26(火) 13:15:11.70 ID:dNBcu/2B.net
至急、量子化学の質問です
3次元シュレディンガー方程式について質問です。
H^ ψ(r,θ,φ)=[-h^2/2m ∇^2+V(r)]ψ(r,θ,φ)=Eψ(r,θ,φ) のエネルギー固有値Eの求め方を教えてください 。
なおV(r)は3パターンで考えます。
1つ目は無限に高いポテンシャル障壁の場合 V(r)=0(0≤r<R),∞(r>R)
2つ目は金属のフェルミエネルギーを考慮した場合 V(r)=E (0≤r<R,E<0) , ∞(r>R)
3つ目は表面を厚さ d の有機分子で修飾した場合 V(r)=E (0≤r<R),Eo(R≤r<R+d )、∞(r>R+d) です。 形じゃなくて式で表しました。

608 :ご冗談でしょう?名無しさん:2021/01/26(火) 13:21:46.28 ID:GAYjY8Fg.net
がんばってね〜〜

609 :ご冗談でしょう?名無しさん:2021/01/26(火) 13:47:37.01 ID:???.net
マルチ

610 :ご冗談でしょう?名無しさん:2021/01/31(日) 13:34:04.14 ID:O1doh5BI.net
宗教板にやってきては量子力学で神の存在を論破しようとするやつがいますが、こいつは
ヴァカですかアホですか、それともハゲですか?

611 :ご冗談でしょう?名無しさん:2021/01/31(日) 13:40:47.18 ID:ASaw+TYu.net
我が負傷の息子です。好きにしてください

612 :ご冗談でしょう?名無しさん:2021/02/01(月) 01:49:28.43 ID:6buoAtLK.net
>>607
息子よ、あなたはもう勝利しています。
神はあなたを祝福しています。
前を向き、果敢に前進していきなさい。
ハレルーヤ!

613 :ご冗談でしょう?名無しさん:2021/02/01(月) 12:49:22.77 ID:???.net
>>611
負傷してるんだ

614 :ご冗談でしょう?名無しさん:2021/02/01(月) 16:24:09.27 ID:???.net
デバイ模型で
連続等方弾性体の基本モードが調和振動子と等価であること
を使ってるんですが、これってどうすれば示せますか?
証明の載っている本を教えて下さるだけでも結構です

615 :ご冗談でしょう?名無しさん:2021/02/01(月) 18:57:21.45 ID:???.net
変位と復元力が比例するなら調和振動子に決まってんじゃん

616 :609:2021/02/02(火) 11:38:47.53 ID:???.net
直感的には分かるけど、それって論理的には言えなくないですか?
真面目に統計力学的な等価性を示すなら
(任意の状態における等方連続弾性体のエネルギー) =(基本モード振動数の様々な調和振動子エネルギーの和)
を言わなければいけないはず
なんか難しいこと言ってますか?

617 :ご冗談でしょう?名無しさん:2021/02/02(火) 19:54:07.59 ID:???.net
それ式で書ける?

618 :ご冗談でしょう?名無しさん:2021/02/03(水) 20:06:34.75 ID:???.net
フーリエ変換で「任意の状態」を表現できるか?というクソしょうもない疑問ですね。

619 :609:2021/02/04(木) 09:50:32.96 ID:???.net
フーリエ変換で任意の状態を表現できるのは分かっています

問題は、エネルギーの書き換えが、ちゃんと調和振動子の形にできるかってことです
分からないなら書き込まなくていいです

620 :ご冗談でしょう?名無しさん:2021/02/04(木) 16:14:44.18 ID:???.net
「任意の状態」が何か知らんが
任意の関数は無理だろ

621 :ご冗談でしょう?名無しさん:2021/02/04(木) 16:58:55.48 ID:???.net
連成振動の問題解いたことないアホがイキがってて草

622 :ご冗談でしょう?名無しさん:2021/02/05(金) 11:08:15.20 ID:???.net
sin / cos の線形結合で書けない関数ってどんなのあるんです?

623 :ご冗談でしょう?名無しさん:2021/02/05(金) 12:19:32.77 ID:???.net
δ

624 :ご冗談でしょう?名無しさん:2021/02/05(金) 12:26:38.81 ID:???.net
わからないんですね

625 :ご冗談でしょう?名無しさん:2021/02/05(金) 12:32:16.67 ID:rbBBlyii.net
わかりません。おしえてください

626 :ご冗談でしょう?名無しさん:2021/02/05(金) 13:00:55.83 ID:???.net
>>622
フーリエ変換かフーリエ級数か
どっちだ?

627 :ご冗談でしょう?名無しさん:2021/02/05(金) 15:14:40.03 ID:???.net
>>626
それって frequency domain (フーリエ変換)表現か time domain (フーリエ級数) 表現か
どっちかって話?

628 :ご冗談でしょう?名無しさん:2021/02/05(金) 15:17:24.81 ID:???.net
等価ではないんです?
フーリエ級数表現可能なのに、フーリエ変換できないとか
フーリエ変換可能なのに、フーリエ級数表現できないとか

よくわからん

629 :ご冗談でしょう?名無しさん:2021/02/05(金) 18:54:39.47 ID:???.net
フーリエ級数をフーリエ変換したら
どうなるか分かってんのか?

630 :ご冗談でしょう?名無しさん:2021/02/05(金) 19:03:15.47 ID:rbBBlyii.net
もうなにがなんだか

631 :ご冗談でしょう?名無しさん:2021/02/05(金) 19:04:37.81 ID:???.net
ただ単に操作的に勉強しちゃうとそのへんがごっちゃになるのは分かる
俺も一回やらかした

632 :ご冗談でしょう?名無しさん:2021/02/05(金) 21:04:48.98 ID:???.net
https://i.imgur.com/dqfgQdN.jpg
この問題を解説していただきたいです

633 :ご冗談でしょう?名無しさん:2021/02/05(金) 21:09:46.76 ID:???.net
>>632
それは大学物理の質問ですか?

634 :ご冗談でしょう?名無しさん:2021/02/06(土) 17:53:01.44 ID:???.net
>>629
> フーリエ級数をフーリエ変換
フーリエ級数の各周波数が横軸に来て、
縦軸がフーリエ級数のそれぞれの項がどれくらい含まれてるか(係数)って理解でいいです?

つまり、time domain -> frequency domain への変換

frequency domain の変換した後の表現のことを指してフーリエ変換って言ったりもする
と書いてあるので、若干読み方/書き方がややこしくなりますが

635 :ご冗談でしょう?名無しさん:2021/02/06(土) 19:12:18.10 ID:???.net
どんなグラフだ?

636 :ご冗談でしょう?名無しさん:2021/02/07(日) 11:54:42.73 ID:irpqxVBB.net
いきなり問題を投げて申し訳ないのですが助けてください。物理選択でないのに物理選択と同じテストを受けなくてはならず全くわかりません。

高さHの山の上にある体積Vの水槽に入った水を全て山の下にある水槽へ落下させた。重力加速度
をg、水の密度を、ρ、水の単位質量あたりの比熱(モル比熱ではないので注意)をCwとして次の問題に答
えよ。
(1)重力によって水がされた仕事をW、外部がら水へ流出した熱量をQ、水の内部エネルギーの増加
量をUとしたとき、これらの3つの量の関係式(熱力学第1法則)を書け。ただし、各量の正負の符号に
気を付けること。
(2)落下前の水温をT0下の水槽に落下して充分時間が経った時の水温T1をとして水の内部エネルギー
の増加量Uを求めよ。
(3)水がされた仕事Wを求めよ。
(4)(1)で得られた関係式を用いて、T1を求めよ。ただし、外部への熱の流出はないとする。
(5)この過程は可逆か、不可逆か?その根拠とともに答えよ。

637 :ご冗談でしょう?名無しさん:2021/02/07(日) 12:24:40.85 ID:???.net
>>635
それはフーリエ級数によるのでは?
フーリエ級数の表現がなんでもいいなら、
単に任意の周期関数と言ってるのと同じ意味じゃないの?

638 :ご冗談でしょう?名無しさん:2021/02/07(日) 12:28:07.32 ID:???.net
周期関数である必要すらないか、有限区間の任意の関数なら、
それを1周期とみなしてフーリエ級数展開できるのよね?

639 :ご冗談でしょう?名無しさん:2021/02/07(日) 12:30:15.12 ID:y+fzN5Zu.net
>>638
>有限区間の任意の関数なら、
ほんと?

640 :ご冗談でしょう?名無しさん:2021/02/07(日) 12:33:17.69 ID:???.net
>>639
違うの?そのへんの厳密な議論がわからない

641 :ご冗談でしょう?名無しさん:2021/02/07(日) 12:35:09.37 ID:???.net
関数が区分的に滑らかで,絶対可積分ならばフーリエ変換が成り立つ

これか

https://eman-physics.net/math/fourier07.html

642 :ご冗談でしょう?名無しさん:2021/02/07(日) 12:37:23.49 ID:???.net
じゃあ、
フーリエ級数展開できるってのと、
フーリエ変換できるってのは
完全に同値?

643 :ご冗談でしょう?名無しさん:2021/02/07(日) 13:00:51.19 ID:???.net
フーリエ級数をフーリエ変換したらデルタ関数の和になる

644 :ご冗談でしょう?名無しさん:2021/02/07(日) 13:08:42.89 ID:???.net
>>636
問題に不審があるな

645 :ご冗談でしょう?名無しさん:2021/02/07(日) 13:24:40.30 ID:irpqxVBB.net
>>644
不審とはなんですか

646 :ご冗談でしょう?名無しさん:2021/02/07(日) 15:50:38.03 ID:???.net
信用できないってこと

647 :ご冗談でしょう?名無しさん:2021/02/07(日) 19:31:20.04 ID:???.net
https://postimg.cc/gallery/bJshRd2

フーリエ解析、みじんも理解できていなかったので、ヨビノリで勉強しました…(泣)

648 :ご冗談でしょう?名無しさん:2021/02/11(木) 14:34:01.24 ID:p15j1LWQ.net
ゲージ理論においてCPが敗れるのはクォークの荷電カレント部分のみであることを示そうと思っています。
湯川相互作用と中性カレントでは破れないことは示せたのですがレプトン荷電カレント部分
では破れないことが示せません。何冊か本を読んでみましたがどの本もレプトンの
部分には触れていないのでわかりません。

649 :ご冗談でしょう?名無しさん:2021/02/11(木) 14:54:43.31 ID:OE300NIz.net
間違ってるから無理

650 :ご冗談でしょう?名無しさん:2021/02/11(木) 15:22:13.35 ID:p15j1LWQ.net
>>649
久後ゲージのほんなど、荷電カレントにおいてCP破れを示しているのですが、どこが間違っているのでしょうか

651 :ご冗談でしょう?名無しさん:2021/02/11(木) 15:27:25.88 ID:OE300NIz.net
それくらい自分で考えろ

652 :ご冗談でしょう?名無しさん:2021/02/11(木) 15:34:11.09 ID:p15j1LWQ.net
>>651
すいません、かなりの時間考えているのですがやはり分かりません。

653 :ご冗談でしょう?名無しさん:2021/02/11(木) 15:47:56.61 ID:OE300NIz.net
>>648
>ゲージ理論においてCPが敗れるのはクォークの荷電カレント部分のみである
だれがそんなこといってんねん

654 :ご冗談でしょう?名無しさん:2021/02/11(木) 16:01:11.77 ID:p15j1LWQ.net
>>653
久後ゲージには
Weinberg-salam模型で唯一CP破れを引き起こす可能性はのがクオークの荷電カレント部分
と書かれていたのですが、どこか読み落としているのでしょうか

655 :ご冗談でしょう?名無しさん:2021/02/11(木) 16:02:06.62 ID:OE300NIz.net
>>654
>Weinberg-salam模型

656 :ご冗談でしょう?名無しさん:2021/02/11(木) 16:07:23.32 ID:p15j1LWQ.net
>>655
Weinberg-salam模型でした。申し訳ありません

657 :ご冗談でしょう?名無しさん:2021/02/11(木) 16:10:36.53 ID:p15j1LWQ.net
しかしそれでも、レプトンカレントではCPが破れないのはなぜでしょうか

658 :ご冗談でしょう?名無しさん:2021/02/11(木) 16:14:04.40 ID:OE300NIz.net
>>657
>CPが破れない
誰が言ってんねん

659 :ご冗談でしょう?名無しさん:2021/02/11(木) 16:15:52.07 ID:p15j1LWQ.net
クォークの荷電カレントが唯一と書かれていたので、他の部分は破れないと
思っていました。

660 :ご冗談でしょう?名無しさん:2021/02/11(木) 16:48:17.75 ID:p15j1LWQ.net
ありがとうございました。解決しました。

661 :ご冗談でしょう?名無しさん:2021/02/12(金) 02:16:53.22 ID:tU65QKip.net
>>656
どこが違うのかさっぱりワカラン!www
=訂正の必要な〜し!

662 :ご冗談でしょう?名無しさん:2021/02/12(金) 11:25:17.42 ID:???.net
同感

663 :ご冗談でしょう?名無しさん:2021/02/12(金) 22:56:36.91 ID:WEE1uN5Z.net
レプトンカレントが破れないのはニュートリノを質量を
0と置いているWeinbergSlam模型だからであり、ゲージ理論
という括りでは間違い、という意味です。

664 :ご冗談でしょう?名無しさん:2021/02/13(土) 12:53:48.08 ID:???.net
わからないんですね

665 :ご冗談でしょう?名無しさん:2021/02/13(土) 14:52:27.21 ID:???.net
>>663
やっと分かったよ

666 :ご冗談でしょう?名無しさん:2021/02/17(水) 23:25:47.65 ID:WbY5y2WU.net
https://ja.wikipedia.org/wiki/%E6%8B%B7%E5%95%8F%E6%AE%BA%E4%BA%BA

南京大虐殺非道冷血ヒトモドキニホンキチガイ猿を焼殺しろ

667 :ご冗談でしょう?名無しさん:2021/02/20(土) 17:10:24.71 ID:5ij6Kisv.net
https:/twitter.com/mitsumaru_shiba

ヒトモドキゴキブリニホンザルキモオタ殺せ
(deleted an unsolicited ad)

668 :ご冗談でしょう?名無しさん:2021/02/21(日) 18:40:21.34 ID:AlGMDcS7.net
ボルンオッペンハイマー近似と断熱近似の違いについて、非断熱項をどの程度無視(全部無視か対角成分は考えるか)っていうのは理論の教科書に載っているんですが、
これが実空間の電子と原子核の動きに対してはどう意味を持っているんでしょうか?
原子核は重たいから電子に比較して動きがゆっくりっていう簡略化しすぎた表現ではこの二つの近似の違いがわかりません

669 :ご冗談でしょう?名無しさん:2021/02/24(水) 16:17:53.41 ID:???.net
回答ないようなので質問スレに移行します

670 :ご冗談でしょう?名無しさん:2021/02/24(水) 16:36:40.39 ID:???.net
ググるとたくさんヒットするようだけど。
たとえば
https://home.hiroshima-u.ac.jp/kyam/pages/results/monograph/Ref23_B-O.pdf

671 :ご冗談でしょう?名無しさん:2021/02/24(水) 17:14:00.85 ID:???.net
ググって読んでも理解できなかったんだろうなー
どこが分からんのか書くこともできんのじゃ無理だろ

672 :ご冗談でしょう?名無しさん:2021/02/25(木) 01:54:00.91 ID:???.net
まずボルンオッペンハイマー近似と断熱近似における「実空間の電子と原子核の動きに対しての意味」というのが分からん
全波動関数が電子波動関数と原子核波動関数の積で表されるということか?

673 :ご冗談でしょう?名無しさん:2021/02/25(木) 02:40:12.10 ID:???.net
どっちもそうだろ

674 :ご冗談でしょう?名無しさん:2021/02/25(木) 10:32:16.42 ID:???.net
>>670
化け屋さんだが、よく勉強なさってる。他にも色々と公開されているようだ。素晴らしい。

675 :ご冗談でしょう?名無しさん:2021/02/25(木) 10:36:19.97 ID:nSrs3ewr.net
adiabaticを断熱と訳すから意味不明になるんやろな

676 :ご冗談でしょう?名無しさん:2021/02/25(木) 11:01:57.62 ID:???.net
>>675
小野寺さんのHPにadiabaticについての解説があるね
http://qmath.webcrow.jp/Adiabatic/diabatic.html
最後のコメント読む限りは英語でも変わらん気がする

677 :ご冗談でしょう?名無しさん:2021/02/25(木) 11:07:28.78 ID:nSrs3ewr.net
>>676
>英語でも変わらん
熱力の文脈で解釈すればかわらんやろけど、「熱」と意味は本来ない

678 :ご冗談でしょう?名無しさん:2021/02/25(木) 11:44:25.99 ID:???.net
振り子の糸の長さをゆっくり変える過程で断熱定理?を証明したりしてて、何でこれが断熱なのかと全く意味不明だった。断熱より無遷移のほうが意味は取りやすい。

679 :ご冗談でしょう?名無しさん:2021/02/25(木) 12:04:32.74 ID:???.net
https://en.wikipedia.org/wiki/Adiabatic_process#Divergent_usages_of_the_word_adiabatic

このへんも参考になる
熱力学と量子統計力学ではadiabaticの意味するところがほぼ真逆であると書かれている。

680 :ご冗談でしょう?名無しさん:2021/02/26(金) 13:54:16.46 ID:???.net
この学位記は、どこのものか判りますか?

理学研究科物理学専攻修士
https://i.imgur.com/pBKCtwN.jpg

681 :ご冗談でしょう?名無しさん:2021/02/26(金) 13:57:36.91 ID:???.net
https://mevius.5ch.net/test/read.cgi/utu/1518429472/454,457,463

> 454 名前:レフト ◆wuAmJN96nro7 [sage] 投稿日:2018/02/19(月) 23:24:58.34 ID:lyjrlSVf [104/110]
> >>454
> 院生の頃は、有機半導体(ポリアセチレンとかグラファイトとか)の物性を調べてた。
> もう、30年以上前の話だがな(笑)

> 457 名前:優しい名無しさん[sage] 投稿日:2018/02/19(月) 23:32:10.73 ID:090rDm/e [5/12]
> >>456
> 2000年にポリアセチレンで日本人がノーベル賞取ってるけど、それについてレフトはどう思ってるの?

> 463 名前:レフト ◆wuAmJN96nro7 [sage] 投稿日:2018/02/19(月) 23:40:24.53 ID:lyjrlSVf [107/110]
> >>459
> 白川氏だろう?
> 実は俺らの研究室は、白川研究室との繋がりもあったんだよ。
>
> ノーベル賞に関しては、少なからず、俺らの地道な研究の貢献もあったと思っている。
> まあ、良かったんじゃないのかな。白川氏は、有機半導体研究の第一人者だったからな。
> 率直に嬉しく思うよ。

https://i.imgur.com/pBKCtwN.jpg

682 :ご冗談でしょう?名無しさん:2021/02/26(金) 18:18:02.68 ID:???.net
>>681
> 俺らの地道な研究

www

683 :ご冗談でしょう?名無しさん:2021/02/26(金) 18:18:38.17 ID:???.net
自分で地道www

684 :ご冗談でしょう?名無しさん:2021/02/26(金) 18:26:29.84 ID:???.net
宇宙誕生時に、物質・反物質の微妙な性質の違いで物質のほうが少し残った、ということは、
エネルギーだけを投入して、反物質を排出することなく物質を合成することが可能?

685 :ご冗談でしょう?名無しさん:2021/02/26(金) 20:49:00.67 ID:???.net
そやね

686 :ご冗談でしょう?名無しさん:2021/03/06(土) 18:35:30.22 ID:???.net
大学レベルの話じゃないんだけどいい?

質問
運動エネルギーと運動量の関係って
「運動量の速度成分が2倍になると運動エネルギーが4倍になる」=「運動エネルギーが大きくなっても運動量(速度成分)はそれほど変わらない」

ってなるけど、これで良いの?
こうじゃなく?

「運動エネルギーの速度成分が2倍になると運動量(速度)が4倍になる」=「運動量の速度成分が2倍になると運動エネルギーは√2倍になる」

こうならないのって

運動エネルギーkg(m^2/s^2)だと前者だけど、kg(m^(1/2)/s^(1/2))だと後者
前者だと
kg(m^2/s^2)(運動エネルギー大)×s/m(その速度成分の√で割る)→kg(m/s)
=運動エネルギーが大きくなっても√で割ってるから速度はあまり変わらない

後者だと
kg(m^(1/2)/s^(1/2))(運動エネルギー大)×m^(1/2)/s^(1/2)(その速度成分を掛ける)→kg(m/s)
運動エネルギーが大きくなるごとに速度が大きくなる

一般にいわれてる前者kg(m^2/s^2)で運動エネルギーって正しいの?

687 :ご冗談でしょう?名無しさん:2021/03/06(土) 18:50:02.29 ID:???.net
活力論争でググってください

688 :ご冗談でしょう?名無しさん:2021/03/06(土) 18:59:14.89 ID:???.net
ちなみに
加速度って何種類もあってその種類は
1.時間経過で速くなる
2.距離経過で速くなる
3.距離と時間の比、その他

加速度ってm:sが1:2になれば良いだけで1/2:1でも2:4でも1/3:2/3でも良くて

m/s^2なら
m/s^2×s→m/s
時間経過で速くなる

m^(1/2)/s×√m→m/s
距離経過で速くなる

s^2/m×1/s→s/m
加遅度×時間経過により結果の遅度が小さくなる=速度は時間経過で速くなる

s/m^(1/2)×1/m(1/2)→s/m
加遅度×距離経過で結果の遅度が小さくなる=速度は距離経過で速くなる

とかできるんだけど、
それと同じように運動エネルギーの方も

kg(m^2/s^2)×s/m→kg(m/s)
運動エネルギーは運動への寄与が小さいのか

kg(m^(1/2)/s^(1/2))×√(m/s)→kg(m/s)
運動エネルギーは運動への寄与が大きいのか

どちらが正しいの?
一般の定説で良いの?

689 :ご冗談でしょう?名無しさん:2021/03/06(土) 19:17:42.45 ID:???.net
そんだけ分かってんなら質問の形で荒らすな

690 :ご冗談でしょう?名無しさん:2021/03/06(土) 19:17:58.19 ID:???.net
ちなみに昔の車の加速→シートに押しつけられる
m/s^2×s(止まる力)→m/s

今の車の加速→前方に投げ出される
m^(1/2)/s×m^(1/2)(進む力)→m/s

と感覚の違いがある

m^2/s^2から速度m/sを得る→
m^2/s^2(絶対速度)×s/m(相対速度)→m/s
→後ろに押しつけられる

m^(1/2)/s^(1/2)から速度m/sを得る→
m^(1/2)/s^(1/2)(絶対速度)×m^(1/2)/s^(1/2)(相対速度)→m/s
前に飛ばされる

691 :ご冗談でしょう?名無しさん:2021/03/06(土) 19:19:45.14 ID:???.net
正準方程式見るまで運動量と運動エネルギーの役割の違いはわからなかった
高校レベルの知識で理解する方法ってあるの

692 :ご冗談でしょう?名無しさん:2021/03/06(土) 19:22:53.57 ID:???.net
運動エネルギーって一般の定説で良いのか違うと考えるかだけ聞きたかった

693 :ご冗談でしょう?名無しさん:2021/03/06(土) 21:52:04.20 ID:???.net
>>691
違いについてのスレあるぞ

694 :ご冗談でしょう?名無しさん:2021/03/06(土) 23:36:50.97 ID:???.net


kg(m^2/s^2)
もしかして:衝突量?

衝突時にしか発生せず、運動する物体の持つ量は運動量のみ、衝突したときの衝突量の目安が運動エネルギーで衝突する前の物体では持たない架空の量、衝突したときのみ衝突量として運動エネルギーが架空から物理になる?

同じく
kg(m^(1/2)/s^(1/2))
これも:異なる量

何らかの物理が持つ量で、その何らかの量により運動量が生み出される時、その量はまだ運動量を持っていない。その時点での運動量を生み出す架空の目安が○○エネルギー?

つまり
量:実体、その物の物理
エネルギー:架空、次の物理を量る目安?

もしかして:↑ということ?

それとも
量=エネルギーで
kg(m^2/s^2):
運動エネルギー…×
衝突エネルギー…○
衝突量…○


もしかして:↑こちら?

695 :ご冗談でしょう?名無しさん:2021/03/06(土) 23:39:23.36 ID:???.net
一般の定説など気にしたこともない
結果さえ合えば考え方は自分に合うようにすれば良い

696 :ご冗談でしょう?名無しさん:2021/03/06(土) 23:47:59.14 ID:???.net
>>694
>>693とは脈絡無しね
自分で考えてて ? になった

697 :ご冗談でしょう?名無しさん:2021/03/06(土) 23:50:51.34 ID:???.net
>>695
気になったから

698 :ご冗談でしょう?名無しさん:2021/03/06(土) 23:52:37.04 ID:???.net
自分の中で完結するなら勝手に考えればよいが、
そんなものを他人に話しても何も通じないから迷惑だわな

699 :ご冗談でしょう?名無しさん:2021/03/07(日) 05:58:10.97 ID:???.net
>>694
大学教員です。

あなたは高校生ですかね?
くだらねぇこと考えてないでクソして寝ろ

700 :ご冗談でしょう?名無しさん:2021/03/07(日) 09:30:09.52 ID:???.net
それは明け方に言うべきことではないだろ

701 :ご冗談でしょう?名無しさん:2021/03/07(日) 10:48:26.97 ID:???.net
>>700
うぜぇから首吊って死んでろ

702 :ご冗談でしょう?名無しさん:2021/03/07(日) 10:55:37.09 ID:???.net
>>701
いちいち荒らすな消えろ

703 :ご冗談でしょう?名無しさん:2021/03/07(日) 11:41:25.77 ID:???.net
>>702
お前が消えろ、ハゲ

704 :ご冗談でしょう?名無しさん:2021/03/07(日) 13:13:06.87 ID:???.net
>>698
そりゃコミュ障だろ

705 :運動エネルギーって何?の人:2021/03/07(日) 17:29:44.31 ID:???.net
>>699-703
なんか勝手に騒いでるけど
>>704
↑そう

706 :ご冗談でしょう?名無しさん:2021/03/07(日) 18:10:08.86 ID:???.net
そうか、そもそもコミュニケーション取れないなら書き込んでもしょうがないと思うんだがな。

まあここの物理板は突拍子もなくぶつぶつ意味不明なことつぶやいてる奴多すぎるし
なぜか全国のコミュ障が集う集会場になってるような気がしないでもない

707 :ご冗談でしょう?名無しさん:2021/03/07(日) 20:20:55.10 ID:???.net
>>706
ただの精神分裂病独特のネット活動だろ

708 :ご冗談でしょう?名無しさん:2021/03/07(日) 23:54:53.17 ID:???.net
当然の現象なんだからスルーしとけばよい

709 :ご冗談でしょう?名無しさん:2021/03/11(木) 01:59:24.45 ID:???.net
だな

710 :ご冗談でしょう?名無しさん:2021/04/23(金) 05:34:26.69 ID:???.net
士業、公務員、医者などは、AIに駆逐される職業の代表格に挙げられます。
このような枠組みに守られた、定型的な仕事というのはAIに代替しやすいからです。
AIは人間に比べて、休憩時間を必要とせず、ミスを犯さず、給与も不要というメリットがあります。
例外的に看護師は安泰じゃないかとされますが、議論と関係がないのでやめておきましょう。

今後は英語の重要性が更に高まるでしょう。
現在、世界には約200か国あり、言語は約6,000存在しますが、グローバル化とインターネットにより英語とスペイン語が支配的になっています。
半数の言語は話者がいなくなり、英語を話せるのは最低限の条件になります。

「英語が出来ない人」というだけで、一段低い人間だと見られる傾向は既にビジネスの世界ではありますが、今後は社会通念になる可能性もあると思います。
そもそも、世界的な学術誌は英語中心になっており(英語力が低い人=学問と無縁な人という価値観の加速)、プログラミング言語も英語であり、ビジネスでは英語の重要性が高まっています。

AI翻訳により、ごく簡単なやり取りは日本語で事足りるようになっても、「AI翻訳が必要な相手」と見なされるでしょうし、文書を読み解くリーディング力や、言語を介した高度な感情の伝達まで補うものではありません。

もちろん、英語が出来ること、というのは最低条件に過ぎず、英語が出来るからといって成功するということではありません。

711 :ご冗談でしょう?名無しさん:2021/05/22(土) 14:34:22.97 ID:???.net
すみません。ちょっとスレの主旨とずれるかもしれませんが、質問させて下さい。


Twitterの美しき物理学botに以下のつぶやきがありました。

ジョークだと思うのですが、どういう意味か分からず、何がどう面白いのか分かりません。
どなたか解説頂けませんでしょうか?


「ママ、sin(x)買っていい?」
「ダメよ、sin(x)はもう家にあるでしょ」

家にあるsin(x):x


https://twitter.com/ST_phys_bot/status/1395917252382236677?s=19
(deleted an unsolicited ad)

712 :ご冗談でしょう?名無しさん:2021/08/09(月) 13:28:45.33 ID:???.net
これ結局、意味不明だったな

713 :ご冗談でしょう?名無しさん:2021/08/09(月) 15:41:51.52 ID:???.net
数学者とエンジニアであることまで書かなきゃ意味ねえだろ

714 :ご冗談でしょう?名無しさん:2021/08/11(水) 06:11:38.00 ID:???.net
home=原点

715 :ご冗談でしょう?名無しさん:2021/09/16(木) 23:32:19.98 ID:???.net
https://i.imgur.com/TGydegK.png
ここの一連の流れが分かりません
ε^nの係数同士の比較をしていることは分かるのですが、こんな項出るのですか?

716 :ご冗談でしょう?名無しさん:2021/09/16(木) 23:45:53.03 ID:ODfuv+Qj.net
牡羊座がどうかしたか?

717 :ご冗談でしょう?名無しさん:2021/09/17(金) 02:35:37.93 ID:???.net
こんなのだけ見て分かるわけねーな

718 :ご冗談でしょう?名無しさん:2021/09/17(金) 13:43:19.58 ID:???.net
>>715

真ん中の級数Σε^n β_n(z,t) を上の方程式に代入するだけなのだろうけど、
実際やってみて微妙に係数が合わない。なぜだろう?

719 :ご冗談でしょう?名無しさん:2021/09/27(月) 17:29:05.70 ID:???.net
単純なやり方だけどある変数hを導入して
G(A,B,h) → [A,B]=ihとおいて
G(A,B,ε)の時は量子力学
G(A,B,0)の時は古典力学
みたいになる定式化ってできないの?

720 :ご冗談でしょう?名無しさん:2021/09/27(月) 23:38:36.90 ID:???.net
複雑になりすぎる

721 :ご冗談でしょう?名無しさん:2021/09/28(火) 06:44:23.69 ID:???.net
量子力学と古典力学が同じ多様体(可能な物理空間)に入ればちょっと面白いかなと思ったのですが
確かに理論としては過度に複雑になりますね

722 :ご冗談でしょう?名無しさん:2021/10/10(日) 18:35:05.98 ID:???.net
素人考えだけど

ポテンシャル中の運動で位置の期待値の運動は古典力学になるとか
経路積分もh→0で古典的運動の経路のウェイトがドミナントになるとか
すごい広い意味で優しい目で見たらそんな感じで定式化されてるんじゃないの

厳しい目で見たら観測をどう扱うかとかで行き詰りそう

723 :ご冗談でしょう?名無しさん:2021/10/11(月) 00:56:44.59 ID:???.net
物理の問題ではないかとのご指摘を受けましたのでこちらで
数学板とのマルチポストになってしまい申し訳ありません

(5.14)、(5.15)を用いてδ→0にすると(5.13)がKdV方程式
U_T+6U*U_X+U_XXX=0 になることを示せ。

この問題が分かりません... おそらく代入して微分の定義式を使うことは分かるのですが、
U_XXXというのがどこの部分から出てくるかさっぱり分かりません
どなたか教えていただけませんでしょうか
https://i.imgur.com/qfwq1aO.jpg
https://i.imgur.com/wmkpD5Y.jpg

724 :ご冗談でしょう?名無しさん:2021/10/11(月) 10:56:56.92 ID:???.net
物理の中でもマルチじゃん

725 :ご冗談でしょう?名無しさん:2022/03/06(日) 07:41:04.14 ID:???.net
E=mc^2は相対論では近似でしか求まらないのに対して、ルイス=トルマンの非ニュートン力学では厳密にE=mc^2が示せると読んだのですが、お互い結果が異なるためこの2つの理論は両立しないはずなので、ルイス=トルマンの非ニュートン力学の公理で相対論では偽になるものがあるはずなんですが、どれが偽になるんでしょうか?

726 :ご冗談でしょう?名無しさん:2022/03/06(日) 15:04:19.66 ID:URRCv+5k.net
mの定義が違うだけです

速度によって変化する相対論的質量を使って表現するのはもはや過去の遺物ですから、そのような解釈はやめた方が良いかと思います

727 :ご冗談でしょう?名無しさん:2022/03/06(日) 16:34:42.25 ID:/r9ovWbl.net
電気というのはクォーツのような微細な振動のことなんかな

728 :ご冗談でしょう?名無しさん:2022/03/06(日) 16:57:59.92 ID:???.net
クォーツは水晶のことだぞ

729 :ご冗談でしょう?名無しさん:2022/03/06(日) 21:13:44.43 ID:???.net
>>726
ほんとだ、細部まで読まないとわからないようにこっそり書いてあった
https://ja.m.wikipedia.org/wiki/%E3%83%AB%E3%82%A4%E3%82%B9%EF%BC%9D%E3%83%88%E3%83%AB%E3%83%9E%E3%83%B3%E3%81%AE%E9%9D%9E%E3%83%8B%E3%83%A5%E3%83%BC%E3%83%88%E3%83%B3%E5%8A%9B%E5%AD%A6
もしかして、このwikipediaの記事ゴミなのか…?それっぽく書いてあるけど本質的なことは全くわからねー

730 :ご冗談でしょう?名無しさん:2022/03/06(日) 21:15:52.57 ID:???.net
過去の遺物もなにも質量がローレンツスカラーでなかった時代なんて皆無だろ

731 :ご冗談でしょう?名無しさん:2022/03/06(日) 21:18:01.34 ID:gL9IdFVc.net
なんか前にもそういうこと言ってる人いましたけど、アインシュタインの原論文だと縦質量横質量とかいう相対論的質量の概念を用いて記述されてますよ

732 :ご冗談でしょう?名無しさん:2022/03/06(日) 21:19:06.15 ID:???.net
その後一切使われてない

733 :ご冗談でしょう?名無しさん:2022/03/06(日) 21:20:42.94 ID:gL9IdFVc.net
ですから、質量がローレンツスカラーでなかったことは”皆無”だという主張は間違ってます

今は使われないんだから、過去の遺物なのです

734 :ご冗談でしょう?名無しさん:2022/03/06(日) 21:21:57.10 ID:???.net
相対論的質量は質量ではない

735 :ご冗談でしょう?名無しさん:2022/03/06(日) 21:23:47.45 ID:gL9IdFVc.net
あなたの中ではそうなのでしょうね

普通の人は、相対論的だろうがなんだろうが、質量と名のつけば質量だと思います

だからいまだに啓蒙書等で速くなれば質量が大きくなるという表現が後を絶たないのですよ

736 :ご冗談でしょう?名無しさん:2022/03/06(日) 21:25:10.87 ID:???.net
素人の中で蔓延してるだけの誤りを正当化するな

737 :ご冗談でしょう?名無しさん:2022/03/06(日) 21:27:27.91 ID:gL9IdFVc.net
アインシュタインが相対論的質量の概念を持ち出した時点で、質量が増える、そのような解釈がなされ得たことは明らかです

その流れを汲んでいまだに質量が増えると表現されてるわけです

いい加減認めたらどうなんですか?

738 :ご冗談でしょう?名無しさん:2022/03/06(日) 21:29:50.32 ID:???.net
まあこんなとこでしか素人と話すことないからどうでも良いか

739 :ご冗談でしょう?名無しさん:2022/03/07(月) 00:40:33.06 ID:???.net
大学物理質問スレで何で素人を基準にしてるんだ?

740 :ご冗談でしょう?名無しさん:2022/03/07(月) 01:43:00.24 ID:???.net
むしろなぜ学部程度のスレの対象が素人でないと思ってるのか謎

741 :ご冗談でしょう?名無しさん:2022/03/07(月) 05:09:27.49 ID:???.net
受験のプロきどりで理系科目マスターだと学部程度の内容に特権を見い出したりするんだろ。

742 :ご冗談でしょう?名無しさん:2022/03/07(月) 14:12:11.34 ID:???.net
せめて中学生程度の理解力ないと

743 :ご冗談でしょう?名無しさん:2022/06/15(水) 04:36:44 ID:iyzPmPSD.net
たんに数学の問題とおもいますが
ミレニアム問題にでてくる
コンパクトな単純ゲージ群ってなにで、いくつありますか?
ゲージ群とゲージ理論がたくさんある場合、基礎的な力もたくさんあるんでしょうか

744 :ご冗談でしょう?名無しさん:2022/06/15(水) 10:18:11.05 ID:???.net
>>731
>アインシュタインの原論文だと縦質量横質量とかいう相対論的質量の概念を用いて記述されてますよ

慣性系で 質量x加速度=力 ( ma=F )  つまりニュートンの運動方程式を採用すると
ローレンツ変換では 縦質量と横質量というスカラーといえない質量が出て来る。

「もし力と加速度の定義を変えれば、これと違った値を得るのは当然である」
とアインシュタインの論文に書いてある。
現代では静止質量m0 をスカラーの質量と定義する。 運動方程式は dP/dt = F
(copyright)

745 :ご冗談でしょう?名無しさん:2022/06/16(木) 10:19:15.11 ID:e+irHx/W.net
宇宙はどんな3次元多様体何ですか?
最新の学説を教えてください。

746 :ご冗談でしょう?名無しさん:2022/06/16(木) 13:20:54.20 ID:???.net
マルチ

747 :ご冗談でしょう?名無しさん:2022/06/16(木) 16:53:34.70 ID:5vVNg+PA.net
コンパクトな単純ゲージ群を教えてください

748 :ご冗談でしょう?名無しさん:2022/06/16(木) 17:03:33.13 ID:4jccCNBm.net
わかりません

749 :ご冗談でしょう?名無しさん:2022/06/16(木) 17:28:58.36 ID:5vVNg+PA.net
エントロピー増大とダークエネルギーは関係があるんでしょうか
量子もつれが時空生成するらしいですが、量子もつれが起こる原因はなんでしょうか
量子もつれとダークエネルギーは関係ありますか

750 :ご冗談でしょう?名無しさん:2022/06/16(木) 18:12:20.39 ID:hl5xgR+j.net
>>744
こんなトコロにまで八兵衛さんキター!🤣

751 :ご冗談でしょう?名無しさん:2022/06/17(金) 13:17:25 ID:???.net
>>749
エントロピーと量子もつれは全てに関係する
量子もつれの原因は全てが単一の波動関数だから

752 :ご冗談でしょう?名無しさん:2022/06/18(土) 13:16:09.34 ID:???.net
オンサーガーの相反定理って何が凄いの?
適当に平均とったら係数行列が対称行列になりました
ってだけでなんでノーベル賞もらえたの?

753 :ご冗談でしょう?名無しさん:2022/06/18(土) 15:30:25.37 ID:???.net
どの理論もただの等式さ
違いは影響の大きさだけ

754 :ご冗談でしょう?名無しさん:2022/06/19(日) 00:43:48.93 ID:???.net
ブラックホールを中心に持たない銀河はありますか?

755 :ご冗談でしょう?名無しさん:2022/06/19(日) 01:40:41.98 ID:ufbZM2bu.net
銀河を中心に持つブラックホールが宇宙ですよ

756 :ご冗談でしょう?名無しさん:2022/06/19(日) 12:38:40.69 ID:???.net
宇宙がブラックホールなんて馬鹿を言ってる奴がまだ居るか

757 :ご冗談でしょう?名無しさん:2022/06/19(日) 13:04:40 ID:ufbZM2bu.net
じゃ、あなたは宇宙のお外に逃げ出せますか?
 光でも無理。つまり、宇宙はブラックホール

758 :ご冗談でしょう?名無しさん:2022/06/19(日) 13:13:53 ID:???.net
>>757
事象の地平線とごっちゃにしてんな、このアホ

759 :ご冗談でしょう?名無しさん:2022/06/19(日) 16:51:23.39 ID:???.net
特異点がどこかにある?

760 :ご冗談でしょう?名無しさん:2022/06/19(日) 18:59:19.41 ID:ufbZM2bu.net
特異点はありません

761 :ご冗談でしょう?名無しさん:2022/06/19(日) 19:56:32.01 ID:???.net
情報エントロピーは、電源に限らすどんな「品質」にでも定義できる

その定義と実際の品質評価がどの程度近いかは別問題だ。

762 :ご冗談でしょう?名無しさん:2022/06/19(日) 19:58:12.52 ID:???.net
>>761
スレ間違ってませんか?w

763 :ご冗談でしょう?名無しさん:2022/06/19(日) 20:10:40 ID:???.net
誤爆

764 :ご冗談でしょう?名無しさん:2022/06/20(月) 15:00:03 ID:X3bm0ism.net
中性子星の中って超流動になっているらしいと聞いて超低温現象の一種の超流動と類似性を感じて思ったんだけど、
超流動って低エントロピー現象なんじゃないか?
超低温実験室での超流動と中性子星の共通点は低エントロピーである事だ。
低温故の低エントロピーとエネルギーを放出し続けてきた成れの果ての低エントロピー

765 :ご冗談でしょう?名無しさん:2022/06/20(月) 15:26:54.94 ID:hJGhFS0b.net
>>764
中性子星は多分、超低音なんだよ

766 :ご冗談でしょう?名無しさん:2022/06/20(月) 16:17:45.75 ID:???.net
地球の尺度では超高温でも縮退した中性子には超低温なだけ

767 :ご冗談でしょう?名無しさん:2022/06/29(水) 15:27:57.73 ID:???.net
粒子って基本バラバラになろうとしてその時にエントロピーが増えるんだよね?
でも星の形成みたいに重力によって粒子が集まることもあるよね?
この時も周りの系を含めればエントロピーは増えてるの?

768 :ご冗談でしょう?名無しさん:2022/06/29(水) 18:35:55.71 ID:???.net
バラバラでも一様分布ならエントロピーは低い
重力で部分的に集まって不均一になればエントロピーは増える

769 :ご冗談でしょう?名無しさん:2022/06/29(水) 22:25:18.40 ID:wNWyDKFL.net
>>767
星の材料が一カ所に集まる時にエントロピーを持った光子を大量に宇宙に放出するから、
宇宙全体ではエントロピーは増大する。

770 :ご冗談でしょう?名無しさん:2022/06/30(木) 17:03:44.94 ID:???.net
単に放熱でいいんじゃ?

771 :ご冗談でしょう?名無しさん:2022/07/01(金) 09:56:04.59 ID:y3HEItjm.net
光子のエントロピーを語る時に光子の個数は重要だと思うんだ。
星が形成される時に放出される光子は星を形成する素粒子よりも多い筈だと言いたい。

772 :ご冗談でしょう?名無しさん:2022/07/01(金) 11:38:25.81 ID:???.net
当たり前

773 :ご冗談でしょう?名無しさん:2022/07/01(金) 14:23:49.07 ID:y3HEItjm.net
素粒子が広い空間を自由に行き来できるほどエントロピーは大きい。
素粒子が何かに拘束された場合は、光子等の新素粒子が生み出されて、
拘束された素粒子に代わって自由を得る。
例外は素粒子がブラックホールに拘束された場合

以下ポエム

素粒子は自由(広い空間)を求めて旅をする。
ブラックホールに飲み込まれた素粒子の自由はどこに消えてしまったのか?
きっと素粒子にとってブラックホールの中は元居た宇宙よりも広いに違いない。

774 :ご冗談でしょう?名無しさん:2022/07/01(金) 23:34:29.19 ID:???.net
ブラックホールも例外ではない

775 :ご冗談でしょう?名無しさん:2022/07/03(日) 10:25:39.31 ID:aBVEtSnB.net
エディントン限界とか言って、天体の光度って制限されているけど、
発光部位が宇宙にむき出しになっている降着円盤でもそれは成り立つのかな?

776 :ご冗談でしょう?名無しさん:2022/07/03(日) 12:44:33.75 ID:???.net
条件が違うから成り立つ訳ねーよ
クェーサーの光度は恒星よりはるかに大きい

777 :ご冗談でしょう?名無しさん:2022/07/03(日) 14:59:34.80 ID:0ZgkTrRY.net
>>762
それ登記888八兵衛さんだよ!

778 :ご冗談でしょう?名無しさん:2022/07/03(日) 15:07:59 ID:???.net
【鉛筆の棒を両手で握ってヘシ折るとき、
いつ折れるかは科学…つまり物理的には予想できない】
…って知ってた?オマイラ?
俺は知ってたけど、イマいち信じてなかった。だが
本日ゲラ〜先生の口から聞いたら、何故か素直に受け入れられたわ。

779 :ご冗談でしょう?名無しさん:2022/07/05(火) 00:13:40.84 ID:???.net
古いネタで自慢げ

780 :ご冗談でしょう?名無しさん:2022/07/05(火) 00:54:05.08 ID:EpQfYdDG.net
>>779
僻むな🐤

781 :ご冗談でしょう?名無しさん:2022/07/05(火) 13:37:14.19 ID:???.net
僻むw

782 :ご冗談でしょう?名無しさん:2022/07/10(日) 05:13:36.63 ID:cfCXfpCx.net
ヘルムホルツの自由エネルギーがよくわかんないです
等温かつ定積ってどういうことですか?
系にちっちゃい穴がついてて系を加熱したらそこから粒子が出ていくとかそういうことですか?

783 :ご冗談でしょう?名無しさん:2022/07/10(日) 09:42:17.14 ID:???.net
>>782
等温かつ定積でも圧力は変わっていいんだよ。
容器に穴が開いてちゃダメ。そこから粒子が出ていってどんどん広がってくから体積が増えることになるでしょう。

784 :ご冗談でしょう?名無しさん:2022/07/10(日) 09:43:25.17 ID:???.net
>>782
等温かつ定積でも圧力は変わっていいんだよ。
容器に穴が開いてちゃダメ。そこから粒子が出ていってどんどん広がってくから体積は増えることになるでしょう。

785 :ご冗談でしょう?名無しさん:2022/07/10(日) 15:04:28 ID:???.net
一成分の流体で物質量が変わらないときは
温度と体積が同じなら圧力も同じだろ

786 :ご冗談でしょう?名無しさん:2022/07/10(日) 15:37:22.97 ID:???.net
著しく表面積が大きくなったらどうだろうか?

787 :ご冗談でしょう?名無しさん:2022/07/10(日) 15:54:33.97 ID:???.net
>>786
それは熱力学の範囲ではない

元の話は,熱力学の自由エネルギーの話

788 :ご冗談でしょう?名無しさん:2022/07/10(日) 16:27:29 ID:???.net
で、何がわからんの?
等温かつ定積のままなら状態はまったく何も変わらないんじゃないのってことか?

そんなことはない。
2種類の気体を容器の中に入れてほっとけば化学反応が進む。
そのとき発熱や吸熱するかもしれんが外気と同じ温度になるまでほっとく。
最終的には容器の中の温度も体積も変わってないのに最初と全然違う状態になっている。

789 :ご冗談でしょう?名無しさん:2022/07/10(日) 18:50:06.63 ID:kGOZOpS6.net
そんなの反則です。同意できません

790 :ご冗談でしょう?名無しさん:2022/07/10(日) 19:22:22.53 ID:???.net
ごくごくあたりまえの話で反則になりうる要素がどこにも見当たらんのだが

791 :ご冗談でしょう?名無しさん:2022/07/10(日) 20:47:55 ID:???.net
マクスウェルの関係式って結局なんなの

792 :ご冗談でしょう?名無しさん:2022/07/10(日) 21:06:30 ID:???.net
エントロピー関連の測定不可能量を測定可能な量で置き換えるためのもの

793 :ご冗談でしょう?名無しさん:2022/07/11(月) 08:34:27.48 ID:???.net
熱力学って下手したら相対論や量子論より適用範囲広いけど何で?
例えば弦理論から熱力学を創発するって研究とかないの?

794 :ご冗談でしょう?名無しさん:2022/07/11(月) 09:20:23.06 ID:ARSNNSK/.net
温度は虚数時間。それで終わり

795 :ご冗談でしょう?名無しさん:2022/07/11(月) 13:08:18.57 ID:???.net
しょうもな

796 :ご冗談でしょう?名無しさん:2022/07/13(水) 10:00:28.95 ID:???.net
すまん初歩の初歩で申し訳ないんだけど
自然単位系で光速はc=1って置かれるけどさ
これって単位ごと1って置いてるの? それとも数値部分だけ1ってなるよう単位を調節してるの?
調べてると殆どが単位ごと、つまり3.0×10^8[m/s]=1に置いてるっぽいんだけどさ、もしそれが本当なら何でそんなことできるん?

797 :ご冗談でしょう?名無しさん:2022/07/13(水) 10:01:39.77 ID:???.net
次元解析を理解しろ

798 :ご冗談でしょう?名無しさん:2022/07/13(水) 10:09:16.07 ID:???.net
>>797
いや次元解析ぐらい分かる
自分の質問はなんで「数値」と「基底(単位)」の積を1にできるの?って話

799 :ご冗談でしょう?名無しさん:2022/07/13(水) 11:22:11.45 ID:???.net
なぜできないと思うのか謎
三(四)元単位系なんて人間が勝手に選んだものでしかないのに

800 :ご冗談でしょう?名無しさん:2022/07/13(水) 11:52:36.34 ID:89Rxmj8X.net
つまり、なんの客観性もないってこと?

801 :ご冗談でしょう?名無しさん:2022/07/13(水) 13:00:02.60 ID:???.net
>>796
各単位は本質的に同一である事を人類が理解する前にそれぞれ勝手に定義されたから
c などの換算定数が知られるまでは別物だったが
現在は同一が理解されてるので 1 とする事ができる

802 :ご冗談でしょう?名無しさん:2022/07/13(水) 13:08:15.42 ID:???.net
ここって割と意味ある質問があるな

803 :ご冗談でしょう?名無しさん:2022/07/13(水) 14:52:34.25 ID:ZdNe0cbP.net
すいません、下記の問題がわかりません

https://i.imgur.com/dbr1nBX.jpg
波の強度とはなんなのかっていうところからわかりません。
とりあえず1つ目の問題を考えてみました、思考過程は以下の通りです。
その前に自分の知識として、光の明るさは光子の個数に依存する、光や波のエネルギーは、振幅の2乗に比例するという知識は持っていて、高校範囲で合成波の波の式を和積の公式を用いて表すことはできます。
まず自分は光の強度の式からがxが、0,1,2…整数の時に光の強度が最大であることはわかりました。
また、T(s)の間光を出しているということは、t=0のときに絞りという名の光源から出た光子がt=Tにスクリーン中央、x=0に到達したんだと思いました。
ここで、絞りの位置がスクリーン上の原点x=0の正面にあり、
また、x=1のときは、この光がどのような波の式かはわからないから経路差うんぬんの話ではなく
光は絞りでおそらく解説して円形波を出してスクリーンに向けてあらゆる方向にt=0のとき
に光子が飛び出し、t=Tでx=1においては光子が強度が最大になるんだと思いました。

ここで質問です、
1つ目 上の状況は間違っていると思うので間違っている点を指摘してほしいです。
2つ目、光の強度  絞り  電磁波と電子波の違い この3つの用語について教えて欲しいです

3つ目 結局この問題のとき方がわからないです… ヒントだけでも欲しいです 1つ目の問題と言わず最後のグラフ書く問題までもし時間に余裕がある方がいらっしゃればぜひとも教えていただきたいですお願いしますm(._.)m
4年前くらいに物理が全くわからなくて物理のちょっとしたしつもんをする板でれっとうばあさんとくっくっくっくさんをはじめとした方々にお世話になりました、
その後高校卒業して浪人して理系の大学にいきました、
大学生なりたてなのともとから物理が苦手なのですがどなたか教えていただけるとと幸いです。
みなさんお忙しいところすいません、この問題の締め切りは4時間後です…なんとか間に合わせたいです

804 :ご冗談でしょう?名無しさん:2022/07/13(水) 15:06:22 ID:???.net
劣等婆とqqqの弟子はこのザマw

805 :ご冗談でしょう?名無しさん:2022/07/13(水) 16:26:51.04 ID:???.net
光が解説するらしいから聞いてみたら?

806 :ご冗談でしょう?名無しさん:2022/07/13(水) 16:29:10.01 ID:2KCoxOSW.net
>>803です
すいません、締め切りまであと1時間30です お忙しいところ申し訳ないのですがどなたか心優しいお方がいましたらご教授願いたいです

>>804
もともと理系に行くつもりなかったのですが今は理系に来てよかったと思えるようになりました

807 :ご冗談でしょう?名無しさん:2022/07/13(水) 16:30:49.26 ID:???.net
劣等婆とqqqに聞く奴に言うだけ無駄

808 :ご冗談でしょう?名無しさん:2022/07/13(水) 17:06:05.99 ID:o4L+6qb3.net
>>807
お忙しいとは思うのですがよければ解答を教えてほしいですどうかお願いします

809 :ご冗談でしょう?名無しさん:2022/07/13(水) 18:08:47.96 ID:???.net
1だけ開けたとき |ψ1|^2
2だけ開けたとき |ψ2|^2
個別に開けた累積 |ψ1|^2 + |ψ2|^2
同時に開けたとき |ψ1 + ψ2|^2

810 :ご冗談でしょう?名無しさん:2022/07/13(水) 20:06:33.67 ID:???.net
相間にモノを教えるなんて不可能だろ

811 :ご冗談でしょう?名無しさん:2022/07/14(木) 03:29:36 ID:???.net
科学史というか数学史の質問ですまん
磁気現象は古代から知られていたからファラデー→マクスウェル→アインシュタイン(や量子論)
の流れは理解できるんだけど
数学におけるカントールの出現があまりにも突然変異過ぎるように感じる
何か前兆ってあったの?

812 :ご冗談でしょう?名無しさん:2022/07/14(木) 06:18:07.88 ID:oIupX8mj.net
そりゃ実数論でぞ

813 :ご冗談でしょう?名無しさん:2022/07/14(木) 09:13:49.35 ID:???.net
>>811
ニュートン、ライプニッツに始まる近代解析学は応用優先でパラドックスを無視続けたが
19世紀になって数学の厳密化が始まり極限、無限大、連続などの定式化が必要になった。

814 :ご冗談でしょう?名無しさん:2022/07/14(木) 09:39:23.89 ID:oIupX8mj.net
最近の大学生はεδとかやらないし、そういう細かいことは気にしないでも構わない

815 :ご冗談でしょう?名無しさん:2022/07/14(木) 10:39:54.94 ID:???.net
カントールはクロネッカーやワイエルシュトラスの講義受けてたらしいから、ちょうど実数論が盛り上がってきてた時代か
クロネッカーは後に有限の立場になってしまったようだけど、人々が無限を気にし始めた頃だったからカントールも無限についてあれこれ考えるようになったんだろうね

816 :ご冗談でしょう?名無しさん:2022/07/14(木) 10:49:36.15 ID:oIupX8mj.net
対角線でゆーれかしたんだな

817 :ご冗談でしょう?名無しさん:2022/07/14(木) 11:38:14.06 ID:???.net
ゲーデルにもつながってる

818 :ご冗談でしょう?名無しさん:2022/07/14(木) 12:51:42.55 ID:oIupX8mj.net
バッハには?

819 :ご冗談でしょう?名無しさん:2022/07/14(木) 13:23:25 ID:???.net
共形場理論のprimary場の変換性がよくわからない(特にスケール変換について)
共形変換x'=λxが単なる座標変換なら、一般座標変換と同様に考えてスカラーの変換性は
Φ'(x')=Φ(x)
とする筈じゃん?
でも共形変換の元でprimary場は(特にスケール変換について)次のように変換するとしてる。
Φ'(x')=Φ(x)×(1/λ^2)^(Δ/2)
この余計な項は何? どういった視点で付け加えられてるの?

自分の予感では単位の変更がその余計な項として反映されてるんだと思う。つまり、
Φ'(x')[λL]=Φ(x)[L](左辺がスケール変換後、右辺が前)
⇒Φ'(x')[λL]=(1/λ)Φ(x)[λL]
⇒Φ'(x')=(1/λ)Φ(x)
でもなぜ座標変換によるスケール変換で単位にも影響が及んだか分からない。
『スケール変換を「スケール変換」と呼ぶ理由が、スケール変換を施すことが単位の変更を施すことと等価だから』という自分の認識が正しいならば、場にスケール変換を施す際は、「座標変換」としてか「単位の変更」としてかのどちらか一方を実行しないといけないと思う。(同時にやると二重にスケール変換を施すことになるので)
でも上の式は単位変換と座標変換を同時に行っているように見える。
一体どう言うこと? 何が起きてるの?

820 :ご冗談でしょう?名無しさん:2022/07/14(木) 19:31:44 ID:???.net
primary場の機能て何?

821 :ご冗談でしょう?名無しさん:2022/07/14(木) 19:45:44.91 ID:???.net
>>820
それって何かのヒントなの?
primary場だと相関関数の形が非常に制限されるだとしかあんまり良く分かってない

822 :ご冗談でしょう?名無しさん:2022/07/15(金) 13:29:39.21 ID:???.net
機能によっては変換で変わるだろ

823 :ご冗談でしょう?名無しさん:2022/07/15(金) 15:43:31.01 ID:???.net
>>816
どのあたり、すっ裸で駆け回ったの?🌸(・∀・)🌷☺

824 :ご冗談でしょう?名無しさん:2022/07/15(金) 17:09:14.91 ID:???.net
>>822
いやいや単なる座標変換なんだから物理的な要素である『単位』に影響を及ぼしちゃだめでしょ

825 :ご冗談でしょう?名無しさん:2022/07/15(金) 18:20:33.75 ID:???.net
微分演算子の座標変換は?

826 :ご冗談でしょう?名無しさん:2022/07/15(金) 18:35:32.37 ID:???.net
>>825
何も関係ない話やめてくれる?
共変ベクトルに対する変換と同じですよね
多様体側から見れば同じ大きさかつ接方向への微分ですが?

827 :ご冗談でしょう?名無しさん:2022/07/15(金) 21:16:35.33 ID:???.net
なんかのOHP発表用のPDFで見た覚えが。

828 :ご冗談でしょう?名無しさん:2022/07/15(金) 21:17:49.54 ID:Rk5ntQ9Q.net
アホばっかりスレ

829 :ご冗談でしょう?名無しさん:2022/07/15(金) 21:44:32.04 ID:???.net
スケール不変性vs共形不変性
https://www.jstage.jst.go.jp/article/butsuri/68/3/68_KJ00008584185/_pdf/-char/ja

830 :ご冗談でしょう?名無しさん:2022/07/15(金) 21:58:21.13 ID:???.net
>>829
疑問に答えるべく資料を引っ張ってきてくれたみたいで、その点に関してはありがとう。
でもざっと見た限り自分の疑問とは無関係みたいだ
自分はスケール変換と共形変換の違いに疑問を持ったのではなく、座標変換である筈の共形変換を施した際に、物理的な実体である『単位』にまで変換が及んでいるように見えることに疑問を覚えている。何か知らないだろうか?

831 :ご冗談でしょう?名無しさん:2022/07/15(金) 22:14:04.55 ID:Rk5ntQ9Q.net
単位ってなんじゃ?

832 :ご冗談でしょう?名無しさん:2022/07/15(金) 22:20:04.52 ID:???.net
>>831
>>819を見て頂ければ
[L]みたいに角括弧で括ってる部分が単位です

833 :ご冗談でしょう?名無しさん:2022/07/15(金) 22:23:21.41 ID:Rk5ntQ9Q.net
普通、次元というだろ。ばか

834 :ご冗談でしょう?名無しさん:2022/07/15(金) 22:26:07.22 ID:Rk5ntQ9Q.net
スカラー場は無次元量であるてでも思ってんだろうか

835 :ご冗談でしょう?名無しさん:2022/07/15(金) 22:40:46.99 ID:???.net
>>834
同じ意味だからいいでしょ
>>834
そもそも例に挙げてるスカラー場が無次元じゃないのにそんなこと思ってるわけないじゃん

836 :ご冗談でしょう?名無しさん:2022/07/15(金) 22:43:40.07 ID:Rk5ntQ9Q.net
なら当たり前じゃん

837 :ご冗談でしょう?名無しさん:2022/07/15(金) 22:50:53.26 ID:???.net
>>836
当たり前じゃなかったら疑問を持つ人間が現れるわけがない
疑問を持つ人間が現れる以上当たり前ではない
「当たり前」と嘯く人間は本当は対して理解してないのに理解したふりをして自分を誇示したいだけ

もう一度、質問を繰り返そう

『スケール変換を「スケール変換」と呼ぶ理由が、スケール変換を施すことが単位の変更を施すことと等価だから』という自分の認識が正しいならば、場にスケール変換を施す際は、「座標変換」としてか「単位の変更」としてかのどちらか一方を実行しないといけないと思う。(同時にやると二重にスケール変換を施すことになるので)
でも上の式は単位変換と座標変換を同時に行っているように見える。
一体どう言うこと?何が起きてるの?

838 :ご冗談でしょう?名無しさん:2022/07/15(金) 22:51:46.47 ID:???.net
>>837
訂正
『当たり前じゃなかったら疑問を持つ人間が現れるわけがない』⇒『当たり前だったら疑問を持つ人間が現れるわけがない』

839 :ご冗談でしょう?名無しさん:2022/07/15(金) 22:52:06.09 ID:Rk5ntQ9Q.net
>>837
>単位の変更を
意味不明
単に、場の次元に見合った変換をするだけだよ

840 :ご冗談でしょう?名無しさん:2022/07/15(金) 23:32:21.00 ID:ICKIndPA.net
>>839
要点はですね、[m]や[cm]みたいな単位は、見かけの量ではなく物理的な長さの基底なんだから、見かけの変換である座標変換の影響は受けない筈だよね?って事。
場の次元に見合った変換って何? 何で次元も影響を受けるの? 多様体の言葉で説明してもらえるとありがたいんだけど。

841 :ご冗談でしょう?名無しさん:2022/07/15(金) 23:44:38.71 ID:???.net
単位と次元区別できてなくて草

842 :ご冗談でしょう?名無しさん:2022/07/15(金) 23:54:04.18 ID:???.net
>>841
よくよく考えたら基底(単位)の長さを変えても次元は変わらんな
次元と単位ってやっぱり別物だし、単位って言葉使ってて正解じゃねーか
まあこの認識の修正があっても質問内容は何も変わらないんだが

843 :ご冗談でしょう?名無しさん:2022/07/16(土) 13:56:34.40 ID:???.net
>>826
何を関係ない方に行ってるんだ?

844 :ご冗談でしょう?名無しさん:2022/07/16(土) 15:39:59.95 ID:???.net
>>843
日本語が意味不明

845 :ご冗談でしょう?名無しさん:2022/07/17(日) 12:58:25.17 ID:???.net
自覚できんか

846 :ご冗談でしょう?名無しさん:2022/07/19(火) 12:08:07.28 ID:veNJdSfC.net
単位系は難しい

847 :ご冗談でしょう?名無しさん:2022/07/19(火) 19:46:38.28 ID:???.net
アフリカでは1分に60秒が過ぎている
は物理学的には嘘、みたいなネタって他に何がある?

848 :ご冗談でしょう?名無しさん:2022/07/19(火) 19:51:43.89 ID:???.net
1分=60秒 は定義だからしょうがない

849 :ご冗談でしょう?名無しさん:2022/07/19(火) 19:57:06.69 ID:???.net
一般相対論を考慮すると日本の1分≠アフリカの60秒だから
アフリカでは1分に60秒が過ぎているってネットミームは物理学的には進次郎構文ではない
という話だが

それでこういうの他にないのかね

850 :ご冗談でしょう?名無しさん:2022/07/19(火) 20:28:14.66 ID:I/0MRJjs.net
>>848
それって、きっちり測定して確かめたの?

851 :ご冗談でしょう?名無しさん:2022/07/20(水) 01:04:07.28 ID:???.net
定義を測定とかアホや

852 :ご冗談でしょう?名無しさん:2022/07/20(水) 04:12:09 ID:gCyMbP93.net
量子力学の演算子の交換関係について質問なんですけど
位置演算子とスピンのz成分の交換関係とかってどうなってるんですか?

(なんか調べても、「位置」と「運動量」、「角運動量の大きさ」と「角運動量のz成分」、「スピンのx成分」と「スピンのy成分」、
みたいなのは山ほど解説が出てくるんですが、これら以外の組み合わせについては調べた範囲で見当たらなかったです・・・)

853 :ご冗談でしょう?名無しさん:2022/07/20(水) 04:29:06 ID:???.net
>>851
バカみたいな君も繰りこみ理論みたいな測定しないと値が無限大に飛んでっちゃう理論知ってるよね?

854 :ご冗談でしょう?名無しさん:2022/07/20(水) 06:15:38.89 ID:IiViKeQx.net
>>852
位置はこの世、スピンはあの世のもの。つまり両者は無関係だから可換

855 :ご冗談でしょう?名無しさん:2022/07/20(水) 12:50:30.26 ID:???.net
>>852
大抵は
「スピンは内部自由度であり、粒子の位置や運動量など外部自由度とは無関係」
と書いてるが、これって可換なのさ

856 :ご冗談でしょう?名無しさん:2022/07/20(水) 13:09:05.10 ID:IiViKeQx.net
ややっこしいのは、スッピンを含めないと角運動量は保存しない。なんじゃそれ???

857 :ご冗談でしょう?名無しさん:2022/07/20(水) 15:27:14.52 ID:???.net
計算してみればええねん

858 :ご冗談でしょう?名無しさん:2022/07/20(水) 16:27:00.88 ID:???.net
角運動量保存則は角運動量の合計が保存するのさ

859 :ご冗談でしょう?名無しさん:2022/07/20(水) 22:04:10 ID:???.net
自分で永久機関(笑)を考えて、自分で論破する遊びやってたら
自分じゃ論破できない代物が出来ちゃったんだけど誰か助けてw
まあ壊れないは無理な上で第二種的な動きをする物

求む、情報熱力学ガチ勢w

860 :ご冗談でしょう?名無しさん:2022/07/22(金) 03:47:29.43 ID:???.net
教えて下さい。

自由落下してる大きな箱の中に
AとBがいます。

AからBへボールをまっすぐ投げたら
AB間をまっすぐ飛んでBへ行くはずです。
これを地上から見たらボールは放物線を描いているはずです。

次にAからBへレーザーを発射したとします。
このレーザーはどういう飛び方をしますか?

よろしくお願い致します。

861 :ご冗談でしょう?名無しさん:2022/07/22(金) 03:59:16.97 ID:???.net
外力がなくても
角運動量保存則が成立しない場合があると聞きました。
つまり、角運動量が保存しない場合です。

それは、内力同士の作用反作用が同一直線上にない場合だそうです。
作用反作用が同一直線上にないとは、どんな場合でしょうか。

作用反作用は必ず同一直線上ではないのでしょうか。
確かに、作用反作用が同一直線上になければ偶力となって回転が始まり
角運動量がない状態からある状態になってしまいますね。保存しません。

これもよろしくお願い致します。

862 :ご冗談でしょう?名無しさん:2022/07/22(金) 04:00:28.03 ID:LFC5Nft4.net
さすが大学物理らしい質問やね

863 :ご冗談でしょう?名無しさん:2022/07/22(金) 04:12:17.87 ID:???.net
>>860
放物線

>>861
微小電流要素と磁荷間に働く力(ビオ・サバールの法則)とか

864 :ご冗談でしょう?名無しさん:2022/07/22(金) 04:37:38.48 ID:???.net
>>863
天才あらわる

865 :ご冗談でしょう?名無しさん:2022/07/22(金) 04:55:28.17 ID:???.net
>>861
現実では方位磁石と閉電流回路との間に働く力とか。
方位磁石のN極S極に働く力と閉電流回路に働く力は同一線上にはない。
この系では他の力つまり外力がないからと言って角運動量は保存しない。

外力がない系だから必ず角運動量が保存されると思い込んでいると大間違い。
専門書のほとんどはその点に気づいていなかったりする。
まず、作用反作用は常に同一線上にあると思い込んでいる人間がほとんど。

ワクチンを打った人間がまずそう。
自分で考えないから常に角運動量は保存されると信じ切っている。
打てば打つほど免疫低下して感染爆発、どんどん記録更新中。

自分で調べて1回も打たなかった人間の大勝利。
1回も打っていなかったら暴露しても感染がなかなか成立しないし、
感染してもすぐに治るし、そういう人間はまずイベルメクチンを持っているから
すぐに飲んですぐに治している。

ワクチンに騙されて打ってしまった人間は変異ウイルスにどんどん感染して
重症化しやすいから気を付けるように。

私からは以上だな。

866 :ご冗談でしょう?名無しさん:2022/07/22(金) 05:00:50.76 ID:???.net
>>865
なるほど
素粒子物理学でも角運動量保存則でオオポカやってそう

867 :ご冗談でしょう?名無しさん:2022/07/22(金) 05:17:56.83 ID:???.net
>>865
バイデンは3回打って感染したってよ。
https://www.nikkei.com/article/DGXZQOGN21DLZ0R20C22A7000000/

重症化を防ぐ飲み薬の服用を始めたって、何のためにワクチン打ったんだよw
ホントにデタラメすぎるウンコワクチンだわwww

868 :ご冗談でしょう?名無しさん:2022/07/22(金) 08:57:56.22 ID:???.net
>>867
おまえ大学出てないだろ?

869 :ご冗談でしょう?名無しさん:2022/07/22(金) 09:28:41 ID:???.net
久しぶりに相間・量間qqqの自作自演のデタラメが始まったか

当然、閉じた系(外力なし)では物理学の基本法則である全角運動量は保存される。

見かけ上保存しないように見える場合、磁性体のスピンを無視しているなどだ。

870 :ご冗談でしょう?名無しさん:2022/07/22(金) 13:10:26.19 ID:???.net
保存則の破れが見つかった場合は
必ず見落としが見つかるのが物理の歴史

871 :858:2022/07/22(金) 18:08:19.49 ID:???.net
>>869 は登記のアホか。
場が角運動量持つって書けばいいのによ。

872 :ご冗談でしょう?名無しさん:2022/07/25(月) 06:53:26.50 ID:42FpfPfP.net
>>865
ほう、お前よく分かってるのうー

理系のくせにワクチン打ったアホは
角運動量保存の条件をまったく理解しておらんだろうな。

くっくっく

873 :ご冗談でしょう?名無しさん:2022/07/25(月) 09:56:51.01 ID:cnLXqN70.net
ワクチンも感染も変わらんやんけ失敗作やなかったらw
なら毒性を低くしてるほうがいいわw失敗作じゃなかったらw

874 :ご冗談でしょう?名無しさん:2022/07/25(月) 09:59:07.61 ID:cnLXqN70.net
m-RNAちゅうても、生ウイルスも同じ働きするやんけw
増殖するんやからよw

875 :ご冗談でしょう?名無しさん:2022/07/25(月) 09:59:37.72 ID:i12RpAn/.net
オミクロンは人工的に改変されて野に放たれた弱毒ワクチン

876 :ご冗談でしょう?名無しさん:2022/07/25(月) 12:53:41.30 ID:???.net
嘘振りまく奴は醜悪な心を曝け出す

877 :ご冗談でしょう?名無しさん:2022/07/25(月) 13:24:59.72 ID:i12RpAn/.net
オミクロン株の異常
https://pbs.twimg.com/card_img/1549166316111675394/9RPHMjL_?format=jpg&name=medium

878 :ご冗談でしょう?名無しさん:2022/07/25(月) 14:25:00.45 ID:???.net
>>872

角運動量保存の条件 = 作用反作用が同じ直線上にあること

これ、理解せずに大間違いやってる素粒子論文いっぱいありそう

879 :ご冗談でしょう?名無しさん:2022/07/25(月) 14:27:39.81 ID:???.net
>>872
今みたいな悲惨な感染状況になると予言してたよね、
打っちゃいけないと大反対してた研究者たちは。
打たなくて良かったよ本当に。

880 :ご冗談でしょう?名無しさん:2022/07/25(月) 15:23:09.94 ID:???.net
>>878
作用と反作用は常に同一直線上にあると思い込んでいる例。
「作用 反作用 直線」で検索すると無数に出てくる。

https://ryebourbon.xsrv.jp/action-reaction/
https://www.yukimura-physics.com/entry/dyn-f20
https://www2.hamajima.co.jp/~tenjin/labo/rikatan201610p38-41.pdf
https://study-z.net/11273/3

図のように方位磁石と閉電流回路があれば(図では分かりにくいが方位磁石の面は閉電流回路の面に垂直であり、
NSの直線上に閉電流回路の面があるとする)
方位磁石のN極に働く力、S極に働く力、閉電流回路の各4辺に働く力は
すべて同一直線上にはないことはビオサバールの法則から明らか。

この状態から始めると、方位磁石と閉電流回路の両方に角運動量が発生、4辺をよくよく考えれば分かるけど
方位磁石と閉電流回路は同じ回転方向になるので打ち消さず角運動量は保存されない。

ここまで理解してる人間はほぼ皆無。
上のお粗末なサイト見れば分かるよね。本当にアホばかりで驚異です。
https://o.5ch.net/1yyqj.png

881 :ご冗談でしょう?名無しさん:2022/07/25(月) 15:33:28.91 ID:???.net
>>880
https://ja.wikipedia.org/wiki/%E9%81%8B%E5%8B%95%E3%81%AE%E7%AC%AC3%E6%B3%95%E5%89%87

さすがにwikiには直線上とか同じ作用線上とか書いてないな。残念。

882 :ご冗談でしょう?名無しさん:2022/07/25(月) 15:52:46.71 ID:Ybz6toD6.net
>>872
きゃー〜〜Qちゃーん!😃💕
待ってたよ〜〜〜!👋😜✨
安心して〜!ボクわあんな得たいの知れないインチキ毒物打ってないよ〜〜〜!
縁の薄い知り合いなんか、疑うことなく素直に打って
さんざん苦しみぬいた末に旅立ったよ〜!😭
一言相談してくれたら…
仮にも理科系にゲソつけた奴であんなの打つ奴いたら、そいつは
最低限の論理的思考できない時点でサイエンス失格やわ🤣

883 :ご冗談でしょう?名無しさん:2022/07/25(月) 15:57:27.28 ID:???.net
>>882
ここの連中は案外打ってなさそうだわ

打ってたらムキになって発作起こすからな

884 :ご冗談でしょう?名無しさん:2022/07/25(月) 16:01:30.20 ID:Ybz6toD6.net
>>865
「自分で調べる」必要すらなかった。日本(世界)の報道、流し見
してるだけで「…ななななんやこいつらいったいナニをやってるんや?」
と沸き起こる疑問と違和感と不信感!あかんわ、こいつら本気で世界の
人口十億分の1に大削減する気満々やわ!はよビル芸の巨大別荘手入れせんと…🔥

885 :ご冗談でしょう?名無しさん:2022/07/25(月) 16:16:18.38 ID:???.net
忘れた頃に荒らしにくるな

886 :ご冗談でしょう?名無しさん:2022/07/25(月) 17:35:17.82 ID:ouMLVhdr.net
>>885
凄く大事で大切なことやんけ!👋😜✨
これ以上人口減らされて堪るか!😖💦

887 :ご冗談でしょう?名無しさん:2022/07/25(月) 17:55:13.06 ID:i12RpAn/.net
今の大流行は、ワクチンよりマスクの方が問題なんだよ。オミクロンさんにはワクチンは向こうだし関係あらへん。日本人だけ常時マスクしてるやろ、そのため異物(バイキン)が入らないため、自然免疫の活動レベルが低下してしまってて、軟弱ウイルスに接しただけで易々と感染してしまうんだな。

888 :ご冗談でしょう?名無しさん:2022/07/25(月) 17:56:28.68 ID:???.net
qqq と絵文字は反ワク、馬鹿2匹お揃いで

889 :ご冗談でしょう?名無しさん:2022/07/25(月) 17:58:26.77 ID:64wvOLdD.net
マスクで免疫低下てwww

890 :ご冗談でしょう?名無しさん:2022/07/25(月) 18:39:35.52 ID:cnLXqN70.net
安部ガンではガチで免疫低下するけどなw

891 :ご冗談でしょう?名無しさん:2022/07/25(月) 19:57:09.86 ID:???.net
>>880
>作用と反作用は常に同一直線上にあると思い込んでいる例。

検索したら
ほぼ全員がそう思っててビックリしたわ。

rotH=iを本当に理解してたら
電流と磁石ではそうならないって常識だよな。

こりゃ、マジで量子論と素粒子論はデタラメだわ。
こんな連中が推してて本書いたりしてんだから絶対にデタラメ理論に決まってる。

超納得したわ。

892 :ご冗談でしょう?名無しさん:2022/07/25(月) 20:13:01.37 ID:cOMm0z6Q.net
馬鹿の発狂自演にはウンザリ

893 :ご冗談でしょう?名無しさん:2022/07/25(月) 21:11:54.81 ID:???.net
場が運動量、角運動量持つって必ず習うのに。
ホントqqqは馬鹿過ぎ。

894 :ご冗談でしょう?名無しさん:2022/07/25(月) 21:18:13.72 ID:???.net
リーマン積分再発見して得意になってたな。
数学板にも書き込んでたとか。

895 :ご冗談でしょう?名無しさん:2022/07/26(火) 01:34:20.75 ID:???.net
>>880
ググったらマジでそう書いてる馬鹿ばっかだな。
作用反作用は一直線上にあるとかそんな大嘘がまかり通っててあり得ないだろ。
ワクチン打つ馬鹿ばっかなのがよく分かる。

896 :ご冗談でしょう?名無しさん:2022/07/26(火) 01:44:08.24 ID:???.net
むしろ遠隔作用論者qqq的には困ったことになるんだが、まあ気付かんだろうな馬鹿だから

897 :ご冗談でしょう?名無しさん:2022/07/26(火) 01:49:09.95 ID:???.net
質問です。
図のように物体を引っ張っても床の摩擦力が大きいため動かないとします。

引っ張る力と摩擦力は同じ作用線上にありませんが、これで釣り合っているとしていいのでしょうか。
同じ作用線上にないのに高校物理では疑問に思わないのでしょうか。

前から疑問に思っていましたが、上で衝撃的な話が出たので、この際すっきり解決したいです。
https://o.5ch.net/1yyup.png

898 :ご冗談でしょう?名無しさん:2022/07/26(火) 01:57:24.47 ID:???.net
はいはいいつもの手口ね

899 :ご冗談でしょう?名無しさん:2022/07/26(火) 02:39:12.97 ID:???.net
>>897
作用と反作用が同一作用線上にない例ばかりで笑える。
高校や大学の物理で同一作用線上にあると教えている教師や教授が存在したら切腹もんだな。

900 :ご冗談でしょう?名無しさん:2022/07/26(火) 04:53:59.90 ID:???.net
>>897
作用・反作用は同じ作用線上にないのが普通だったのか。

同じ作用線上にあるって教えてる奴やサイトとかどうすんのよ。

確かに切腹もんだな。書いてる教科書もあるんじゃないのか。

901 :ご冗談でしょう?名無しさん:2022/07/26(火) 05:00:00.45 ID:???.net
>>897
これはひっかけ問題。
よく考えればおかしいことに気づく。
まあ、反射的に作用反作用だと思い込んでしまう例。
高校教師なら考え込ませることは出来るな。そのまま何がおかしいのか
分からない教師もいそうだが。

902 :ご冗談でしょう?名無しさん:2022/07/26(火) 05:02:40.33 ID:???.net
>>899
>>900

この2人は見事にひっかかったな。

903 :ご冗談でしょう?名無しさん:2022/07/26(火) 05:05:57.79 ID:???.net
>>880
これは素晴らしいね。
教科書に絶対に載せるべき例。

904 :ご冗談でしょう?名無しさん:2022/07/26(火) 06:03:17 ID:khLwqqGp.net
>>880
念のために言っておいてやるが
1つの磁荷と
1つの電流素片との間でちゃんと考えろよ。どちらから計算しても
ちゃんと作用反作用の法則が成り立つ、しかし同一直線上にはないからな。

左図のように
磁荷mが作る磁界Hが
電流素片Idsに及ぼす力はベクトルとして
F=Ids×B=Ids×μH=Ids×μ・m・r/4πμr^3=mIds×r/4πr^3・・・(1)

この逆で右図のように
電流素片Idsが作る磁界H’が
磁荷mに及ぼす力はベクトルとして
F’=mH’=mIds×r’/4πr’^3・・・(2)

r=ーr’なので
(1)=ー(2)となり、大きさは同じで向きは反対だから
ちゃんと作用反作用の法則が成り立っておるわけだ。
ところがFとF’は反平行であり同一直線上にはないのだな。

このように磁荷と電流素片の組み合わせなら
それだけで作用反作用の法則が成り立つのだ。

ところが電流素片と電流素片の組み合わせだとそうはならず、
2つの閉電流全体で計算して合算しないと
作用反作用の法則が成り立たんのはお前らも知っておるだろうとおりだ。

じゃあな
アホザルども

くっくっく
sssp://o.5ch.net/1yyvq.png

905 :ご冗談でしょう?名無しさん:2022/07/26(火) 06:14:10.30 ID:???.net
圧倒的すぎて神すぎる

知らなかったことばかりなり。。。

906 :ご冗談でしょう?名無しさん:2022/07/26(火) 06:17:54.97 ID:???.net
>>901
答え教えてくんなまし。。。

907 :ご冗談でしょう?名無しさん:2022/07/26(火) 07:54:50.20 ID:FRglcImG.net
物体を引っ張る力の反作用は物体から引っ張らられる力であって摩擦力ではない

908 :ご冗談でしょう?名無しさん:2022/07/26(火) 08:24:50.48 ID:UU7p2tyV.net
それじゃ力が釣り合わない

909 :ご冗談でしょう?名無しさん:2022/07/26(火) 09:34:27.29 ID:FRglcImG.net
>>897
作用と反作用を含めて力をすべて書いてみろ

910 :ご冗談でしょう?名無しさん:2022/07/26(火) 11:33:26.21 ID:???.net
物理板なのに力の釣り合いと作用反作用の関係の区別がついてないってマジ?

911 :ご冗談でしょう?名無しさん:2022/07/26(火) 12:11:03.99 ID:???.net
qqqは区別がついてない

912 :ご冗談でしょう?名無しさん:2022/07/26(火) 12:47:46.22 ID:???.net
しょせん相間だし

913 :ご冗談でしょう?名無しさん:2022/07/26(火) 14:07:11.62 ID:???.net
>>897
(1)
>物体を引っ張っても床の摩擦力が大きいため動かないとします。
(2)
>引っ張る力と摩擦力は同じ作用線上にありませんが、これで釣り合っているとしていいのでしょうか。

静止してる前提条件なのだから、物体(剛体)に作用する全ての合力は釣り合ってる
間違いは、(2)の主張だと誰でもそこまでは判る
>引っ張る力と摩擦力は同じ作用線上にありません・・・
に拘ってるが、2つだけの力によるつり合いでしか成り立たないのだよ!

床から物体に作用する力と力のモーメントを無視してるから矛盾してるように見えるだけ
さらに、頭がいい人なら逆に
(未知の)摩擦力が引っ張る力に平行で同じ大きさでなければならないことが解かる。
(copyright)

914 :ご冗談でしょう?名無しさん:2022/07/26(火) 14:54:27.40 ID:FRglcImG.net
重力は?w

915 :ご冗談でしょう?名無しさん:2022/07/26(火) 15:56:32.98 ID:???.net
>>904
くっくっくはたまに現れるときには
記述がすべてノーベル賞級だから恐れ入る。

あとの人間はほとんど便所の落書きレベル。

916 :ご冗談でしょう?名無しさん:2022/07/26(火) 16:04:37.49 ID:???.net
>>907
そういうことだね。

では、>>897は何がおかしいのだろうか。

何か抜けているのだろうか。>>913はそれらしいこと書いてるけど本当に分かっているのだろうか。

917 :ご冗談でしょう?名無しさん:2022/07/26(火) 16:13:21.58 ID:???.net
999からは何もかもが間違ってると正当な科学で間違い探しするのは無理と学べる

918 :ご冗談でしょう?名無しさん:2022/07/26(火) 16:20:13.83 ID:???.net
>>904
2つの電流素片間では作用反作用が成り立たないというのは大学物理で教えられるけど、
そんなふうに磁石と電流素片でどちらから計算しても一致して作用反作用が成り立つというのはテキストにも書いてないよね。

本当に勉強になるし、相対論と量子論が虚構だという主張もきっと事実なんだろうね。
ワクチン打ってるバカ人間の圧倒的多さ見れば納得できるよ。

919 :ご冗談でしょう?名無しさん:2022/07/26(火) 16:28:46.83 ID:???.net
>>916
>>913は正解だろ
ただ本人が真に理解してるかどうかはちょっと怪しいな
図が書けないところが特に怪しい
なんか適当に書いてたまたま当たってるだけじゃないかと

920 :ご冗談でしょう?名無しさん:2022/07/26(火) 16:29:47.42 ID:???.net
お前はワクチン打つなよ
絶対だぞ

921 :ご冗談でしょう?名無しさん:2022/07/26(火) 16:39:48.61 ID:???.net
>>881
ホントだ。
wikiは間違い多いけど、作用反作用は同一作用線上にあるとは書いてないのは感動した。
これ書いた人は偉い。

検索したら作用反作用は必ずしも同一作用線上にあるとは限らないって正しいこと書いてるサイトが
まったくといっていいくらい見つからないんだけど、やはり相対論と量子論はエセ科学ということか。
こういう間抜けな人間たちが支持してるんだからそうに決まってるな。

922 :ご冗談でしょう?名無しさん:2022/07/26(火) 16:41:24.47 ID:???.net
>>920
もう若い人間ほど打ってないんだが、、、
接種率見てみろよ

ジジババだけだぞ、まだ打ってんのは、、、

923 :ご冗談でしょう?名無しさん:2022/07/26(火) 16:48:19.64 ID:???.net
>>921
そのwikiはせいぜい90点だろ
重心回りの回転を記述していないからな

作用反作用が同一直線上にない場合は、内力でも回転が発生するからだ

924 :ご冗談でしょう?名無しさん:2022/07/26(火) 16:53:36.41 ID:???.net
>>920
打ったら感染しやすく重症化もしやすく
サル痘にもインフルにもRSにも梅毒にもエイズにも感染しやすくなるのに
打つわけないだろ

まったくアホかいな
ツイッターで世界中の動画や写真や論文や記事やデータ見てみろ

925 :ご冗談でしょう?名無しさん:2022/07/26(火) 16:54:24.00 ID:???.net
qqqは遠隔作用説の狂信者だから、電磁場(と相対論)が嘘だとあら捜してる
それが見つかると湧いて自作自演を始める。

>>913 の説明から解かるように、2つの力だけの作用反作用とその作用線説では
電磁力の相互作用は説明できない。
簡単に言えば、
互いに相手が空間に形成した電磁場との近接作用を介した作用反作用になる。

926 :ご冗談でしょう?名無しさん:2022/07/26(火) 16:58:36.15 ID:???.net
>>924
世界中でいろんなウイルスや奇病が流行り出したのは
完全にワクチンのせいだよね。

927 :ご冗談でしょう?名無しさん:2022/07/26(火) 18:58:01.29 ID:6EAndNU/.net
状態密度と電子比熱係数の間の単位換算で死にそう
比較対象の状態密度がeV^-1atom^-1で電子比熱係数がJK^-2mol^-1で計算していくとatomとKmol/Jの換算がないとできないぽい
わかる人います?

928 :ご冗談でしょう?名無しさん:2022/07/26(火) 19:04:56.08 ID:???.net
次元が違う別の量だから単位の換算はできない
自由電子模型でフェルミ面上の状態密度から比熱を求めるなら1molに何個の原子があるかを考えればいいだけ

929 :ご冗談でしょう?名無しさん:2022/07/26(火) 19:06:33.91 ID:???.net
1 eV = 1.602 × 10^-19 J
atmが何かわからんが、1molの粒子数は6.02×10^23 個

930 :ご冗談でしょう?名無しさん:2022/07/26(火) 20:56:02 ID:???.net
atmだったら気圧だろ

931 :ご冗談でしょう?名無しさん:2022/07/26(火) 20:57:53 ID:UU7p2tyV.net
お金の出し入れのできるとこ

932 :ご冗談でしょう?名無しさん:2022/07/26(火) 21:29:09.28 ID:I4z4AMB5.net
>>888
だ〜か〜ら〜、タイタニックこっそり取り替えて
沈没させて保険金で大儲けしたのはデブすげえ豚(ゴイム)
…まぁ、ボクわ…🐤

933 :ご冗談でしょう?名無しさん:2022/07/26(火) 21:31:12.11 ID:???.net
>>897
ひっぱる力と摩擦力だけでは左回りのモーメントが発生してしまい釣り合わない。
重力と垂直抗力が抜けている。

よくあるモデルでは重心に重力、垂直抗力も摩擦力も1点のみで作用するとしているが、
現実には垂直抗力も摩擦力も接触面全体に発生している。

動かない、つまり釣り合っているとはそれらのモーメントすべて合わせて零になっているということ。
物体に働く摩擦力のモーメントはどのような分布でも左回りになるので、垂直抗力の分布が
図のように左側へ偏ってそれらのモーメントが右回りになっていることになる。

要は、垂直抗力のモーメントが発生してトータルのモーメントは零になり、
合力も零だから物体は動かないという状態になっているということ。

摩擦が弱ければ物体は滑り出し、
摩擦が強くて滑らないなら強く引っ張ると左回りに回転してしまう。これは垂直抗力のモーメントが負けるということ。

引っ張る力と摩擦力はどちらも物体に働く作用力であって、作用反作用ではない。
https://o.5ch.net/1yz85.png

934 :ご冗談でしょう?名無しさん:2022/07/26(火) 21:42:53.44 ID:???.net
>>933
こういう現実の物理学を知らない教えられない馬鹿たちによって
虚構である相対論と量子論が広まったんだよな。

ワクチンによってそれが完璧に実証されたようなものだわ。
人口の大多数は馬鹿であり、ウソをウソと見抜けない馬鹿たち。
相対論も量子論もまったく見抜けないから簡単に広まった。

935 :ご冗談でしょう?名無しさん:2022/07/26(火) 21:53:16.61 ID:???.net
>>934
ワクチン見てたら
本当にそう思うよ。
バカがこんなに多数派だとは
思わなかった。

本当に怖い。

936 :ご冗談でしょう?名無しさん:2022/07/26(火) 22:20:08.37 ID:UU7p2tyV.net
>>934
見識ですな。すばらしい

937 :ご冗談でしょう?名無しさん:2022/07/27(水) 13:18:14.13 ID:???.net
>>931
なるほろ

938 :ご冗談でしょう?名無しさん:2022/07/27(水) 20:10:13.77 ID:RBWKzAhG.net
>>937
キャッシュ・ディスペンサ〜言え😂

939 :ご冗談でしょう?名無しさん:2022/07/27(水) 22:09:49.27 ID:???.net
>>933
そういうことね
あんた、くっくっく氏じゃないの?

940 :ご冗談でしょう?名無しさん:2022/07/27(水) 22:11:43.81 ID:???.net
でも>>897っておもしろい引っ掛け問題だね
確かに即答できる教師はいなさそう

941 :ご冗談でしょう?名無しさん:2022/07/28(木) 00:08:35.34 ID:jBtQuGKo.net
全然つまらない。

942 :ご冗談でしょう?名無しさん:2022/07/28(木) 07:15:59.20 ID:???.net
垂直抗力が重心通ると思ったら大間違いってことか。
確かに強く引くと物体の後方が床から浮いて
接触箇所が前のほうに片寄るよなあ。

これは勉強になったよ。

943 :ご冗談でしょう?名無しさん:2022/07/28(木) 08:26:12.83 ID:cbyR0qr5.net
アホかw

944 :ご冗談でしょう?名無しさん:2022/07/28(木) 13:51:48 ID:BNoCXtlX.net
添付した図のような四要素模型に応力σ0を加えたとき、無限時間後の変形速度はどのようになりますか?
バネの弾性率を上からG1,G2,ダッシュポットの粘度を上からη2,η3として
ひずみを上からγ1,γ2,γ3とします

https://image.slideserve.com/1345329/slide28-n.jpg

945 :ご冗談でしょう?名無しさん:2022/07/28(木) 14:27:53.69 ID:???.net
,ダッシュポットってなんすか

946 :ご冗談でしょう?名無しさん:2022/07/29(金) 06:39:08.18 ID:???.net
>>944
ダッシュポットは速度に比例して反力が生じる要素=ダンパーだから、無限時間後なら速度成分は無くなってる=ゼロでは?

947 :ご冗談でしょう?名無しさん:2022/08/03(水) 18:45:04.76 ID:aaMaUbWm.net
ここって熱力学もいいですか?

948 :ご冗談でしょう?名無しさん:2022/08/03(水) 22:27:22.65 ID:nF6RbpJe.net
ダッシュポットの中の熱力学をどうぞ

949 :ご冗談でしょう?名無しさん:2022/08/06(土) 06:56:48.13 ID:???.net
 いちお法学なのに、工場に入れられてそりゃふてくされてるようにみえるだろうさ

950 :ご冗談でしょう?名無しさん:2022/08/15(月) 00:14:04.58 ID:???.net
二重振り子の計算がようわからん

951 :ご冗談でしょう?名無しさん:2022/08/15(月) 11:14:41.41 ID:???.net
詳解力学演習でも読めば

952 :ご冗談でしょう?名無しさん:2022/08/15(月) 12:52:46.15 ID:29+s4CHd.net
いやです

953 :ご冗談でしょう?名無しさん:2022/08/16(火) 15:44:01.17 ID:???.net
ランジュバン方程式で二重振り子やってみたけど
もっと頭の中スッキリする解き方ないの?
式の途中、正直何やってるのかわからないから、こういう流れ作業って感じで解いてるだけだった

954 :ご冗談でしょう?名無しさん:2022/08/16(火) 15:54:50.34 ID:GxM7mxzJ.net
まあランジュバン方程式よりラグランジュ方程式使うほうがスッキリするだろうな

955 :ご冗談でしょう?名無しさん:2022/08/16(火) 17:24:29.66 ID:???.net
なんで確率微分方程式?

956 :ご冗談でしょう?名無しさん:2022/08/16(火) 18:35:08.51 ID:???.net
すまんラグランジュだった

957 :ご冗談でしょう?名無しさん:2022/08/16(火) 22:52:43.49 ID:PTc/HRp7.net
「マッカ―リサイモン物理化学」で物理化学を勉強しています。
定常状態のシュレーディンガー方程式の1次元の箱の中の粒子を考えた際、波動関数は単なる正弦波となりますが
エネルギーがハミルトニアン演算子をsinに作用させた時に固有値(定数)となるのと同様に、
運動量を運動量の演算子を波動関数(sin)に作用させて固有値としてくくりだすことはできないのでしょうか?
普通に微分するとうまくできないような気がするのですが・・・ 

958 :ご冗談でしょう?名無しさん:2022/08/16(火) 23:27:07.35 ID:???.net
有限区間で正弦波になる状態は運動量の固有状態ではない

959 :ご冗談でしょう?名無しさん:2022/08/17(水) 00:45:22.23 ID:t4DelD1b.net
解答ありがとうございます。
上述の状態はむしろEに対して固有値が得ることができる特別な状態というふうに理解しました。
数学的な知識が不足していて変形できていないのかなとも思っていましたがそうではないのですね。

960 :ご冗談でしょう?名無しさん:2022/08/17(水) 09:07:36.49 ID:???.net
むしろとかじゃなくて一般にエネルギー固有関数は運動量の同時固有関数ではない

961 :ご冗談でしょう?名無しさん:2022/08/17(水) 11:09:29.38 ID:L30p2CS7.net
ムニちゃんポポだのんのおじさんなんだいっ

962 :ご冗談でしょう?名無しさん:2022/08/19(金) 15:59:22.23 ID:???.net
ピカー

963 :ご冗談でしょう?名無しさん:2022/08/21(日) 00:37:44.83 ID:???.net
xyz方向や接線法線方向に分解したがりさんなんだけどラグランジュの方がいいんかね
なんかよくわからんけど解けるのが解せない

964 :ご冗談でしょう?名無しさん:2022/08/21(日) 11:44:19.01 ID:???.net
物理の一般的原理だからさ

965 :ご冗談でしょう?名無しさん:2022/08/21(日) 16:46:46.81 ID:???.net
ホロノーム拘束とかそっちの抽象論

966 :ご冗談でしょう?名無しさん:2022/08/22(月) 13:17:26.35 ID:???.net
非ホロノーム系って言いにくいね

967 :ご冗談でしょう?名無しさん:2022/08/24(水) 00:40:18.99 ID:???.net
ラグランジュもハミルトンも結局なんなのかよくわかってない
細かいこと気にしたら負けかな
気にして突き詰めるべきなんだろうけど

968 :ご冗談でしょう?名無しさん:2022/08/24(水) 13:16:39.26 ID:???.net
固有名詞に人名を使っただけだから
難しく考えることないよ

969 :ご冗談でしょう?名無しさん:2022/08/24(水) 19:37:21.47 ID:???.net
まぁ数学科の範囲だわな

970 :ご冗談でしょう?名無しさん:2022/08/26(金) 15:38:57.05 ID:???.net
等温過程ってどういうこと?
過程だからずーーっと一定の温度で変化すること?
それとも初めと終わりさえ等温なら過程は問わない?
ヘルムホルツの自由エネルギーで等温定積みたいな言葉出てきて
「それだとなにも変わらなくね?」って思った

971 :ご冗談でしょう?名無しさん:2022/08/26(金) 16:22:37.11 ID:???.net
準静的なら常に温度一定

972 :ご冗談でしょう?名無しさん:2022/08/27(土) 09:03:52.01 ID:???.net
常に温度一定なのか
なら等温定積な過程でヘルムホルツの自由エネルギーの変化量が0以下になるってなんなんだ…
気体がパンパンに詰まった箱に穴開ければ系の中の粒子が減っていって圧力とエントロピー変化してヘルムホルツの自由エネルギーが下がりますよってこと?
気体抜ければエネルギー下がるなんて当たり前のことの様にも思えるけども

973 :ご冗談でしょう?名無しさん:2022/08/27(土) 09:07:17.99 ID:???.net
いやでも気体抜ければそれはそれで体積変化してることになるか…
定積な化学変化すればヘルムホルツの自由エネルギーが減るとか?

974 :ご冗談でしょう?名無しさん:2022/08/27(土) 13:28:31.61 ID:???.net
等温だからって熱の出入りがないとは限らん

975 :ご冗談でしょう?名無しさん:2022/08/28(日) 09:47:02.46 ID:???.net
>>973
化学変化に限らず、状態変化とかもね

976 :ご冗談でしょう?名無しさん:2022/08/28(日) 13:07:01.91 ID:???.net
ムニちゃんポポたのんのおじさんなんだいっ!

977 :ご冗談でしょう?名無しさん:2022/08/28(日) 17:13:08.90 ID:???.net
>>972
例えば
壁で仕切られた2室に気体が入っていて壁を取り除く

978 :ご冗談でしょう?名無しさん:2022/08/28(日) 22:15:34.75 ID:???.net
あーなるほど
気体が化学反応もせずただ混合するだけで等温過程でできる仕事の最大量が減るってなんか面白いな

979 :ご冗談でしょう?名無しさん:2022/08/29(月) 13:11:04.66 ID:???.net
それを利用する発電てのもあったな
海水による塩分濃度差発電とか

980 :ご冗談でしょう?名無しさん:2022/08/29(月) 13:20:03.81 ID:???.net
円錐振り子のハミルトニアンで
r,θ,φを変数にして解いたんだけど
何で微分したらr' θ' φ' が求められるの?

981 :ご冗談でしょう?名無しさん:2022/08/30(火) 12:44:56.64 ID:???.net
質問が意味不明すぎ付

982 :ご冗談でしょう?名無しさん:2022/08/30(火) 23:10:03.79 ID:???.net
解く前に分かってることが
解いたのに分からんとは
不思議だね

983 :ご冗談でしょう?名無しさん:2022/08/31(水) 06:11:23.42 ID:kjGWcKRM.net
ムーニーちゃんしんぷのおじさんだい

984 :ご冗談でしょう?名無しさん:[ここ壊れてます] .net
自分がミジメでないんかね

985 :ご冗談でしょう?名無しさん:2022/09/02(金) 09:41:26.88 ID:???.net
ムニちゃーんポポ

986 :ご冗談でしょう?名無しさん:2022/09/02(金) 13:02:33.12 ID:???.net
ミジメやね

987 :ご冗談でしょう?名無しさん:2022/09/03(土) 21:18:06.51 ID:rICLEgyr.net
大学の物理学と高校の物理と中学の理科と小学校の理科と幼稚園のお遊戯との
違いを説明してください。

988 :ご冗談でしょう?名無しさん:2022/09/04(日) 11:12:54.78 ID:???.net
何の意味がある?

989 :ご冗談でしょう?名無しさん:[ここ壊れてます] .net
みんな同じでしょ。どうせ役にたたないし

990 :ご冗談でしょう?名無しさん:2022/09/04(日) 11:48:20.39 ID:???.net
やってる側の頭の程度が違う

991 :ご冗談でしょう?名無しさん:2022/09/04(日) 18:44:27.96 ID:4vygGdkF.net
趣味で物理の勉強続けてる無能サラリーマンです
GriffithsのIntroduction to Elementary Particlesの6章で、ファインマン図から散乱のamplitude(M)を計算する所なのですが
https://imgur.com/rSU7viJ
CM(center of momentum)フレームにおいて(6.56), (6.57)式の最後の変換(=-1p^2(1-cosθ), =-1p^2(1+cosθ))がどうやって出てくるのか教えて頂けるでしょうか?
Fig.6.10からこれが導けるという事だと思うのですが…

992 :ご冗談でしょう?名無しさん:[ここ壊れてます] .net
タダの内積計算

993 :ご冗談でしょう?名無しさん:2022/09/05(月) 12:47:40.37 ID:???.net
>>989
幼稚園のお遊戯の恩恵を受けた奴が…
恩知らずめ!

994 :982:2022/09/12(月) 17:53:52.12 ID:Ga4CDQCq.net
俺的には・・・
大学の物理学は体系的に基礎をやる
高校の物理はとりあえず若いうちに詰め込んどけ的にやる
中学の理科は多少、数学的に計算させてみる
小学校の理科は日常経験の整理
幼稚園のお遊戯は遊びやけんかで力学を体で知る

995 :ご冗談でしょう?名無しさん:2022/09/13(火) 17:58:10.95 ID:???.net
高校もけっこう基礎やってた気がするが
中学理科で計算てあったか?
高圧放電の実験で痺れさせられた事しか覚えてない
教師自身も痺れてたはずなのに教師の手が触れると痺れて逃げれないなんて
ホントに人間か?

996 :ご冗談でしょう?名無しさん:2022/09/19(月) 18:59:57.06 ID:mGAhqXE/.net
>中学理科で計算てあったか?
速度=移動距離÷時間
仕事=力×移動距離
ぐらいはやる

997 :ご冗談でしょう?名無しさん:2022/09/20(火) 20:20:02.34 ID:aUSOPv7K.net
フランクリンは凧揚げやって雷撃受けても平気。
ライデン瓶に雷を閉じ込めた。
だから雷電という日本語もある。

998 :ご冗談でしょう?名無しさん:2022/09/20(火) 20:29:35.12 ID:l0Ixp/+I.net
売電さんは初電気屋ですか?

999 :ご冗談でしょう?名無しさん:2022/09/20(火) 22:51:40.89 ID:???.net
フランクリンの真似した奴
死んでんのな

1000 :ご冗談でしょう?名無しさん:2022/09/22(木) 10:36:02.15 ID:???.net
フランクリンの凧揚げは嘘だったが定説

1001 :ご冗談でしょう?名無しさん:2022/09/22(木) 13:29:31.73 ID:???.net
納得
不死身すぎると思った

1002 :ご冗談でしょう?名無しさん:2022/10/14(金) 10:22:30.89 ID:???.net
あれ不死身だったんか

1003 :ご冗談でしょう?名無しさん:2022/10/14(金) 10:22:43.96 ID:???.net
意外だな

1004 :ご冗談でしょう?名無しさん:2022/10/14(金) 10:23:28.15 ID:???.net
うーゆ

1005 :ご冗談でしょう?名無しさん:2022/10/14(金) 10:23:34.06 ID:???.net
あっ!

1006 :2ch.net投稿限界:Over 1000 Thread
2ch.netからのレス数が1000に到達しました。

総レス数 1006
260 KB
掲示板に戻る 全部 前100 次100 最新50
read.cgi ver.24052200